210
Solutions Manual for Basic Engineering Mathematics C. K. Chan, C. W. Chan and K. F. Hung August 21, 2008

AMA SolutionsManual

Embed Size (px)

DESCRIPTION

Solution

Citation preview

Page 1: AMA SolutionsManual

Solutions Manual for Basic Engineering Mathematics

C. K. Chan, C. W. Chan and K. F. Hung

August 21, 2008

Page 2: AMA SolutionsManual

Contents

1 Complex Numbers 1

2 Linear Algebra 13

3 Infinite series, Power series and Fourier series 41

4 Partial Differentiation 53

5 Multiple Integrals 83

6 Vector Calculus 125

7 Differential Equations 157

8 Laplace and Fourier Transformations 177

9 Partial Differential Equations 191

i

Page 3: AMA SolutionsManual

Chapter 1

Complex Numbers

(1) z = 2− 3i, |z| =√

22 + 32 =√

13.

(2) By definition,1

z=

(

a

a2 + b2

)

+ i

( −b

a2 + b2

)

. Therefore,

z · 1z

= (a + ib)

(

a

a2 + b2+ i

−b

a2 + b2

)

= 1 + 0i = 1.

(3) (a)3 + i

2− i=

3 + i

2− i× 2 + i

2 + i=

(6− 1) + i(3 + 2)

22 + 12=

5 + 5i

5= 1 + i.

(b)2 +√

3i

1− 4i=

2 +√

3i

1− 4i× 1 + 4i

1 + 4i==

(

2− 4√

3)

+(

8 +√

3)

i

17

(c)1− i

1 +√

3i=

1− i

1 +√

3i× 1−

√3i

1−√

3i=

(

1−√

3)

−(

1 +√

3)

i

4

(4) Let z = a + ib and w = c + id. We have (z) = (a− ib) = a + ib = z. Since (z + w) =

(a + c) + i(b + d) = a+c−i(b+d) and z+w = (a−ib)+(c−id), we have (z + w) = z+w.

Moreover, observe that

z · w = (ac− bd) + i(ad + bc) = (ac− bd)− i(ad + bc)

and

z · w = (a− ib)(c− id) = (ac− bd) + i(−bc− ad).

We thus conclude that z · w = z · w. Finally, the last result follows from the identity

|z · w|2 = (z · w)(z · w) = z · w · z · w = zz · ww = |z|2 |w|2 .

(5) If z = a+ ib, then z + z = (a+ ib)+(a− ib) = 2a = 2 Re z and z− z = (a+ ib)− (a− ib) =

1

Page 4: AMA SolutionsManual

CHAPTER 1. COMPLEX NUMBERS

2ib = 2i Im z. Since |Re z|2 = a2 ≤ a2 + b2 = |z|2, we may take square root to obtain

|Re z| ≤ |z|.

(6) |z + w|2 = (z+w)(z + w) = (z+w)(z+w) = z z+w w+w z+z w = |z|2+|w|2+w z+w z =

|z|2 + |w|2 + 2 Re(w z). Since the earlier results together imply Re(w z) ≤ |Re(w z)| ≤|w z| = |w| |z| = |w| |z|, we obtain

|z + w|2 ≤ |z|2 + |w|2 + 2 |z| |w| = (|z|+ |w|)2.

The triangle inequality thus follows from taking square root of the above inequality.

(7) (a) Distance between(

12 , 1)

and the center =

(

12 − 0

)2+ (1− 1)2 = 1

2

Thus, 12 + i lies inside the circle.

(b) Distance between(

1, i2

)

and the center =

(1− 0)2 +(

12 − 1

)2=

√5

2 > 1

Thus, 1 + i2 lies outside the circle.

(c) Distance between(

12 ,

√2

2 i)

and the center =

(

12 − 0

)2+(√

22 − 1

)2= 0.57947

Thus, 12 +

√2

2 i lies inside the circle.

(d) Distance between(

−12 ,√

3i)

and the center =

(

−12 − 0

)2+(√

3− 1)2

= 0.88651

Thus, −12 +√

3i lies inside the circle.

(8) For z = cos θ + i sin θ, we have z · z = cos2 θ + sin2 θ = 1. Therefore,

1

z= z = cos θ − i sin θ. Therefore, z +

1

z= z + z = 2 cos θ. Furthermore, DeMoivre’s

Theorem implies z2 = cos 2θ + i sin 2θ and therefore z2 +1

z2= z2 + (z)2 = 2 cos 2θ.

(9) (a) (3+4i)2−2(x− iy) = x+ iy ⇒ −7+24i = 3x− iy. Therefore, x = −73 and y = −24.

(b) Note that

(

1 + i

1− i

)2

= −1 and1

x + iy=

x− iy

x2 + y2. Therefore,

(

1 + i

1− i

)2

+1

x + iy= 1 + i⇒ −1 +

(

x

x2 + y2

)

− i

(

y

x2 + y2

)

= 1 + i.

Therefore, we havex

x2 + y2= 2 and

y

x2 + y2= −1. Solving these equations and

noting that x and y can not be both equal to zero, we conclude that x = 25 and

y = −15 .

(c) (3 − 2i)(x + iy) = 2(x − 2iy) + 2i − 1 ⇒ x + 2y = −1 and −2x + 7y = 2. Solving

these equations, one obtains x = −1 and y = 0.

2

Page 5: AMA SolutionsManual

CHAPTER 1. COMPLEX NUMBERS

(10) Since√

3 + i = 2(

cos π6 + i sin π

6

)

and 1− i =√

2(

cos 7π4 + i sin 7π

4

)

, we conclude that

(√

3 + i)4

(1− i)3=

24(

cos 4π6 + i sin 4π

6

)

(√2)3 (

cos 21π4 + i sin 21π

4

)

= 4√

2

(

cos55π

12− i sin

55π

12

)

= 4√

2

(

cos7π

12− i sin

12

)

.

(11) (a) Since−1− i

√3

2= cos 4π

3 + i sin 4π3 , one concludes from DeMoivre’s Theorem that

(

−1− i√

3

2

)3

=

(

cos4π

3+ i sin

3

)3

= cos12π

3+ i sin

12π

3= 1.

(b)√

3− i = 2(

cos 11π6 + i sin 11π

6

)

1 + i =√

2(

cos π4 + i sin π

4

)

(√3− i

)4(1 + i)10

=

[

2

(

cos11π

6+ i sin

11π

6

)]4[√

2(

cosπ

4+ i sin

π

4

)]10

= 29

[

cos

(

4× 11π

6+ 10× π

4

)

+ i sin

(

4× 11π

6+ 10× π

4

)]

= 29

(

cos59π

6+ i sin

59π

6

)

= 29

(

cos11π

6+ i sin

11π

6

)

= 256√

3− 256i

(c) 1−√

3i = 2(

cos 5π3 + i sin 5π

3

)

1 +√

3i = 2(

cos π3 + i sin π

3

)

(

1−√

3i)4 (

1 +√

3i)8

=

[

2

(

cos5π

3+ i sin

3

)]4[

2(

cosπ

3+ i sin

π

3

)]8

= 212

[

cos

(

4× 5π

3+ 8× π

3

)

+ i sin

(

4× 5π

3+ 8× π

3

)]

= 212

(

cos28π

3+ i sin

28π

3

)

= 212

(

cos4π

3+ i sin

3

)

= −2048− 2048√

3i

(12) Since −32 = 32 (cos π + i sinπ) and 8i = 8(

cos π2 + i sin π

2

)

, the 5th roots of −32 are given

3

Page 6: AMA SolutionsManual

CHAPTER 1. COMPLEX NUMBERS

by

zk = 2

[

cos(2k + 1

5π) + i sin(

2k + 1

5π)

]

, k = 0, 1, 2, 3, 4;

while the cubic roots of 8i are given by

zk = 2

[

cos(4k + 1

6π) + i sin(

4k + 1

6π)

]

, k = 0, 1, 2.

(13) (a)

(

−8 + 8√

3i)1/4

=

[

16

(

−1

2+

√3

2i

)]1/4

=

16

[

cos

(

2kπ +2π

3

)

+ i sin

(

2kπ +2π

3

)]1/4

= 2

[

cos

(

2+

π

6

)

+ i sin

(

2+

π

6

)]

, where k = 0, 1, 2 and 3

z0 = 2(

cos π6 + i sin π

6

)

=√

3 + i

z1 = 2(

cos 2π3 + i sin 2π

3

)

= −1 +√

3i

z2 = 2(

cos 7π6 + i sin 7π

6

)

= −√

3− i

z3 = 2(

cos 5π3 + i sin 5π

3

)

= 1−√

3i

(b)

(

−32 + 32√

3i)1/6

=

[

64

(

−1

2+

√3

2i

)]1/6

=

64

[

cos

(

2kπ +2π

3

)

+ i sin

(

2kπ +2π

3

)]1/6

= 2

[

cos

(

3+

π

9

)

+ i sin

(

3+

π

9

)]

, where k = 0, 1, ..., 5

z0 = 2(

cos π9 + i sin π

9

)

= 1.8794 + 0.68404i

z1 = 2(

cos 4π9 + i sin 4π

9

)

= 0.34730 + 1.9696i

z2 = 2(

cos 7π9 + i sin 7π

9

)

= −1.5321 + 1.2856i

z3 = 2(

cos 10π9 + i sin 10π

9

)

= −1.8794− 0.68404i

z4 = 2(

cos 13π9 + i sin 13π

9

)

= −0.34730− 1.9696i

4

Page 7: AMA SolutionsManual

CHAPTER 1. COMPLEX NUMBERS

z5 = 2(

cos 16π9 + i sin 16π

9

)

= 1.5321− 1.2856i

(14) If z = x + iy, thenz

z=

z × z

z × z=

(z)2

|z|2.

If z → 0 along the real axis, one has z = x + i0 and thereforez

z→ 1. On the other hand,

if z → 0 along the imaginary axis, then z = 0 + iy andz

z→ −1. We thus conclude that

limz→0

z

zdoes not exist. When z0 6= 0, lim

z→z0

z

z=

z0

z0.

(15) (a) From DeMoivre’s theorem, we have

(cos x + i sinx)5 = cos 5x + i sin 5x.

On the other hand, by binomial theorem, we also have

(cos x + i sinx)5 =5∑

k=0

(

5

k

)

cosk x (i sinx)5−k

Therefore, comparing the real and imaginary parts yields

cos 5x = cos5 x− 10 cos3 x sin2 x + 5 cos x sin4 x,

sin 5x = sin5 x− 10 cos2 x sin3 x + 5 cos4 x sinx

(b)

cos x cos 5x = cos6 x− 10 cos4 x sin2 x + 5 cos2 x sin4 x

sinx sin 5x = sin6 x− 10 cos2 x sin4 x + 5 cos4 x sin2 x

Adding these equations yields

cos x cos 5x + sin x sin 5x = sin6 x + cos6 x− 5(cos2 x sin4 x + cos4 x sin2 x)

cos 4x = sin6 x + cos6 x− 5 cos2 x sin2 x

Therefore,

sin6 x + cos6 x = cos 4x + 5 sin2 x cos2 x = cos 4x +5

4sin2 2x

5

Page 8: AMA SolutionsManual

CHAPTER 1. COMPLEX NUMBERS

Since cos 4x = 1− 2 sin2 2x, we have

sin6 x + cos6 x = cos 4x +5

8(1− cos 4x) =

3

8cos 4x +

5

8=

3 cos 4x + 5

8

(16) Using the identityz

z=

z2

|z|2for any non-zero complex no. z, we have

1 + sin x + i cos x

1 + sin x− i cos x=

(1 + sin x + i cos x)2

|1 + sin x + i cos x|2

=(1 + sin x)2 − cos2 x + 2i cos x(1 + sinx)

(1 + sin x)2 + cos2 x

=

(

1 + 2 sinx + sin2 x− cos2 x)

+ 2i cos x(1 + sinx)

2 (1 + sin x)

= sinx + i cos x

= cos(π

2− x) + i sin(

π

2− x)

= ei(π2−x).

DeMoivre’s Theorem now gives

(

1 + sin x + i cos x

1 + sin x− i cos x

)n

=[

ei(π2−x)]n

= ein(π2−x)

= cos n(π

2− x) + i sinn(

π

2− x).

(17) If the given number is a + bi, we are looking for a number x + yi such that

(x + yi)2 = a + bi.

This is equivalent to the system of equations

x2 − y2 = a and 2xy = b.

From these equations we obtain

(x2 + y2)2 = (x2 − y2)2 + 4x2y2 = a2 + b2.

Hence we must have

x2 + y2 =√

a2 + b2,

6

Page 9: AMA SolutionsManual

CHAPTER 1. COMPLEX NUMBERS

where the square root is positive or zero. Together with the equation x2− y2 = a we find

x2 =a +√

a2 + b2

2and y2 =

−a +√

a2 + b2

2.

These equation yield, in general, two opposite value for x and two for y. But these values

cannot be combined arbitrarily, for the equation 2xy = b is not a consequence of these

equation. We must therefore be careful to select x and y so that their product has the

sign of b. This leads to the general equation

√a + bi = ±

(

r + a

2+ i(sgn b)

r − a

2

)

where r =√

a2 + b2.

(18) (a) Let w = z2. Then the given equation becomes w2 − 2w + 4 = 0. The quadratic

formula implies

z2 = w =2±√

4− 16

2= 1±

√3i.

Case z2 = 1 +√

3i: In polar form, 1 +√

3i = 2(

cos π3 + i sin π

3

)

. Therefore,

z =√

2

(

cos

( π3 + 2kπ

2

)

+ i sin

( π3 + 2kπ

2

))

, k = 0, 1.

Equivalently, the square root formula with r = 2 and sgn b = 1 implies

z = ±(

2 + 1

2+ i

2− 1

2

)

= ±(

3

2+ i

1

2

)

.

Case z2 = 1−√

3i: In polar form, 1−√

3i = 2(

cos −π3 + i sin −π

3

)

. Therefore,

z =√

2

(

cos

(−π3 + 2kπ

2

)

+ i sin

(−π3 + 2kπ

2

))

, k = 0, 1.

Equivalently, the square root formula with r = 2 and sgn b = −1 implies

z = ±(

2 + 1

2− i

2− 1

2

)

= ±(

3

2− i

1

2

)

.

(b) Let w = z3. Then the equation becomes w2 + 2w + 2 = 0. Solving the equation

yields

z3 = w =−2±

√4− 8

2= −1± i.

7

Page 10: AMA SolutionsManual

CHAPTER 1. COMPLEX NUMBERS

Case z3 = −1 + i: In polar form, −1 + i =√

2(

cos 3π4 + i sin 3π

4

)

. Therefore,

z = 21/6

(

cos

(

3π4 + 2kπ

3

)

+ i sin

(

3π4 + 2kπ

3

))

, k = 0, 1, 2.

Case z3 = −1− i: In polar form, −1− i =√

2(

cos −3π4 + i sin −3π

4

)

. Therefore,

z = 21/6

(

cos

(

−3π4 + 2kπ

3

)

+ i sin

(

−3π4 + 2kπ

3

))

, k = 0, 1, 2.

(c) Let w = z2. Then the given equation becomes w2 + 4w + 16 = 0. The quadratic

formula implies

z2 = w =−4±

√16− 64

2= −2± 2

√3i.

Case z2 = −2 + 2√

3i: In polar form, 1 +√

3i = 4(

cos 2π3 + i sin 2π

3

)

. Therefore,

z = 2

(

cos

(

2π3 + 2kπ

2

)

+ i sin

(

2π3 + 2kπ

2

))

, k = 0, 1.

Equivalently, the square root formula with r = 4 and sgn b = 1 implies

z = ±(

4− 2

2+ i

4 + 2

2

)

= ±(

1 + i√

3)

.

Case z2 = −2− 2√

3i: In polar form, −2− 2√

3i = 4(

cos −2π3 + i sin −2π

3

)

. There-

fore,

z = 2

(

cos

(

−2π3 + 2kπ

2

)

+ i sin

(

−2π3 + 2kπ

2

))

, k = 0, 1.

Equivalently, the square root formula with r = 4 and sgn b = −1 implies

z = ±(

4− 2

2− i

4 + 2

2

)

= ±(

1− i√

3)

.

(d) z4 = −1 = cos π + i sinπ. Therefore,

z = cos

(

π + 2kπ

4

)

+ i sin

(

π + 2kπ

4

)

, k = 0, 1, 2, 3.

(19) (a) ei π2 = cos π

2 + i sin π2 = i

8

Page 11: AMA SolutionsManual

CHAPTER 1. COMPLEX NUMBERS

(b) 4e−i π2 = 4

[

cos(

−π2

)

+ i sin(

−π2

)]

= −4i

(c) 8ei 7π3 = 8

(

cos 7π3 + i sin 7π

3

)

= 4 + 4√

3i

(d) 2e−i 3π4 = 2

[

cos(

−3π4

)

+ i sin(

−3π4

)]

= −√

2−√

2i

(e) 6ei 2π3 eiπ = 6ei 5π

3 = 6(

cos 5π3 + i sin 5π

3

)

= 3− 3√

3i

(f) ei π4 e−iπ = e−i 3π

4 =[

cos(

−3π4

)

+ i sin(

−3π4

)]

= −√

22 −

√2

2 i

(20) Let z = x + iy and define ez = ex (cos y + i sin y).

(a) If z = x + 0i, ez = ex (cos 0 + i sin 0) = ex

(b) |ez| =√

(ex cos y)2 + (ex sin y)2 = ex. In particular∣

∣eiy∣

∣ = e0 = 1.

(c) ez1+z2 = e(x1+x2)+i(y1+y2) = ex1+x2 (cos (y1 + y2) + i sin (y1 + y2)) = ez1ez2 .

(d) |ez| = ex 6= 0 for any z ∈ C. Therefore, ez 6= 0. Furthermore,

1

ez=

1

ex (cos y + i sin y)= e−x (cos y − i sin y) = e−x+i(−y) = e−z.

(e) Repeat application of (c) and (d), or by induction, yields enz = (ez)n for any integer

n.

(f)

d

dteiwt =

d

dtcos wt + i

d

dtsinwt = −w sinwt + iw cos wt = iw (cos wt + i sinwt)

= iweiwt.

(21) Since ei2πkt = cos (2πkt) + i sin (2πkt), the cosine function is an even function and the

sine function is an odd function, we have

n∑

k=−n

ei2πkt =n∑

k=−n

cos (2πkt) + in∑

k=−n

sin (2πkt) = 1 + 2n∑

k=1

cos (2πkt)

Recall that

1 + z + z2 + · · ·+ zn =1− zn+1

1− z.

9

Page 12: AMA SolutionsManual

CHAPTER 1. COMPLEX NUMBERS

Then

1 +n∑

k=1

(

eiθ)k

=1−

(

eiθ)n+1

1− (eiθ)=

1−(

ei(n+1)θ)

1− (eiθ)

=1− cos(n + 1)θ − i sin(n + 1)θ

1− cos θ − i sin θ

=1− cos(n + 1)θ − i sin(n + 1)θ

(1− cos θ)2 + sin2 θ

=[1− cos(n + 1)θ − i sin(n + 1)θ] [1− cos θ − i sin θ]

2(1− cos θ)

Comparing the real part of this equation yields

1 + cos θ + cos 2θ + · · ·+ cosnθ =(1− cos(n + 1)θ)(1− cos θ) + sin(n + 1)θ sin θ

4 sin2(θ/2).

Simplifying the right hand side of the equation, we have

(1− cos(n + 1)θ)(1− cos θ) + sin(n + 1)θ sin θ

4 sin2(θ/2)

=(1− cos(n + 1)θ)(2 sin2(θ/2)) + sin(n + 1)θ(2 sin(θ/2) cos(θ/2))

4 sin2(θ/2)

=2 sin2(θ/2)− 2 sin(θ/2)[cos(n + 1)θ sin(θ/2)− sin(n + 1)θ cos(θ/2)]

4 sin2(θ/2)

=1

2+

sin[(2n + 1)θ/2]

2 sin(θ/2)

Hencen∑

k=1

cos (kθ) = −1

2+

sin[(2n + 1)θ/2]

2 sin(θ/2).

Finally, by letting θ = 2πt, we have

n∑

k=−n

ei2πkt = 1 + 2n∑

k=1

cos (2πkt) = 1 + 2

[

−1

2+

sin[(2n + 1)πt]

2 sinπt

]

=sin[(2n + 1)πt]

sinπt.

Integrating this result yields

∫ 1

0

sin[(2n + 1)πt]

sinπtdt =

∫ 1

0

[

1 + 2n∑

k=1

cos (2πkt)

]

dt = 1 + 2n∑

k=1

∫ 1

0cos (2πkt) dt = 1.

(22) Suppose w is a root of the polynomial P (z) = a0 +a1z +a2z2 + · · ·+anzn, i.e., P (w) = 0.

10

Page 13: AMA SolutionsManual

CHAPTER 1. COMPLEX NUMBERS

Then

P (w) = a0 + a1w + a2w2 + · · ·+ anwn = a0 + a1w + a2w2 + · · ·+ anwn

= a0 + a1w + a2w2 + · · ·+ anwn = P (w) = 0.

(23) Let z = x + yi, then

z − 3

z + 3

= 2

=⇒ |z − 3| = 2 |z + 3|=⇒ |(x− 3) + yi| = 2 |(x + 3) + yi|

=⇒√

(x− 3)2 + y2 = 2

(x + 3)2 + y2

=⇒ (x− 3)2 + y2 = 4[

(x + 3)2 + y2]

=⇒ x2 + 10x + y2 + 9 = 0

=⇒ (x + 5)2 + y2 = 16

(24) Let z = x + yi, then

|z + 3|+ |z − 3| = 10

=⇒ |(x + 3) + yi|+ |(x− 3) + yi| = 10

=⇒√

(x + 3)2 + y2 +

(x− 3)2 + y2 = 10

=⇒ (x + 3)2 + y2 + 2

(x + 3)2 + y2

(x− 3)2 + y2 + (x− 3)2 + y2 = 100

=⇒[

(x + 3)2 + y2] [

(x− 3)2 + y2]

=(

41− x2 − y2)2

=⇒ 64x2 + 100y2 = 1600

=⇒ x2

25+

y2

16= 1

(25) If z = a + bi and |z| = 1, then a2 + b2 = 1.

w =z − 1

z + 1=

(a + bi)− 1

(a + bi) + 1=

(a− 1) + bi

(a + 1) + bi=

(a− 1) + bi

(a + 1) + bi

(a + 1)− bi

(a + 1)− bi

=(a− 1) (a + 1) + b2 + 2bi

(a + 1)2 + b2=

(

a2 + b2)

− 1 + 2bi

(a + 1)2 + b2

=(1)− 1 + 2bi

(a + 1)2 + b2=

2bi

(a + 1)2 + b2

Therefore, w is a purely imaginary number.

11

Page 14: AMA SolutionsManual
Page 15: AMA SolutionsManual

Chapter 2

Linear Algebra

(1) (a) A =

0 −1 −2

1 0 −1

2 1 0

(b) A =

1 12

13

2 1 23

3 32 1

(c) A =

0 −1 −212 0 −1

223

13 0

(d) A =

e e2 e3

e2 e4 e6

e3 e6 e9

(2) The entry at the i-th row and the j-th column of BTAT is given by the product of i-th

row of BT =[

b1i b2i · · · bni

]

and the j-th column of AT =[

aj1 aj2 · · · ajn

]Twhich

is equal to∑

bki ajk. On the other hand, the entry at the i-th row and the j-th column

of (AB)T is the same as the the entry at the j-th row and i-th column of AB, and is

therefore given by∑

ajk bki.

If A−1 exists, the above result ⇒ AT(

A−1)T

=(

A−1A)T

= IT = I.

(3) (a)

BT =[

A(

ATA)−1

AT]T

=(

AT)T[

(

ATA)−1]T

AT = A[

(

ATA)T]−1

AT

= A(

ATA)−1

AT = B

13

Page 16: AMA SolutionsManual

CHAPTER 2. LINEAR ALGEBRA

(b)

B2 =[

A(

ATA)−1

AT]2

=[

A(

ATA)−1

AT] [

A(

ATA)−1

AT]

= A(

ATA)−1 (

ATA) (

ATA)−1

AT = A(

ATA)−1

AT = B

(4) (a) Let A =

[

1 0

1 1

]

and B =

[

1 1

0 1

]

.

(A + B)2 =

([

1 0

1 1

]

+

[

1 1

0 1

])2

=

[

5 4

4 5

]

A2 + 2AB + B2 =

[

1 0

1 1

]2

+ 2

[

1 0

1 1

][

1 1

0 1

]

+

[

1 1

0 1

]2

=

[

4 4

4 6

]

(b) Let A =

[

1 0

1 1

]

and B =

[

1 1

0 1

]

.

(A + B) (A−B) =

([

1 0

1 1

]

+

[

1 1

0 1

])([

1 0

1 1

]

−[

1 1

0 1

])

=

[

1 −2

2 −1

]

A2 −B2 =

[

1 0

1 1

]2

−[

1 1

0 1

]2

=

[

0 −2

2 0

]

(c) Let C =

[

0 1

0 0

]

6= 0.

C2 =

[

0 1

0 0

][

0 1

0 0

]

=

[

0 0

0 0

]

(5) (a)

2 6 −4 10

4 1 3 −2

1 −3 4 −7

r1 ←→ r3−−−−−−→

1 −3 4 −7

4 1 3 −2

2 6 −4 10

(b)

1 −3 4 −7

4 1 3 −2

2 6 −4 10

−4r1 + r2 −→ r2−−−−−−−−−−−−→

1 −3 4 −7

0 13 −13 26

2 6 −4 10

(c)

1 −3 4 −7

0 13 −13 26

2 6 −4 10

12r3 −→ r3−−−−−−−→

1 −3 4 −7

0 13 −13 26

1 3 −2 5

(6)

2x1 − x2 + 3x3 − 2x4 = 1 · · · · · · (1)

x2 − 2x3 + 3x4 = 2 · · · · · · (2)

4x3 + 3x4 = 3 · · · · · · (3)

4x4 = 4 · · · · · · (4)

14

Page 17: AMA SolutionsManual

CHAPTER 2. LINEAR ALGEBRA

From (4), x4 = 1.

From (3), 4x3 + 3 (1) = 3 =⇒ x3 = 0.

From (2), x2 − 2 (0) + 3 (1) = 2 =⇒ x2 = −1.

From (1), 2x1 − (−1) + 3 (0)− 2 (1) = 1 =⇒ x1 = 1.

(7)

1 2 1 3

3 −1 −3 −1

2 3 1 4

−3r1 + r2 −→ r2−−−−−−−−−−−−→

1 2 1 3

0 −7 −6 −10

2 3 1 4

−2r1 + r3 −→ r3−−−−−−−−−−−−→

1 2 1 3

0 −7 −6 −10

0 −1 −1 −2

r2 ←→ r3−−−−−−→

1 2 1 3

0 −1 −1 −2

0 −7 −6 −10

−7r2 + r3 −→ r3−−−−−−−−−−−−→

1 2 1 3

0 −1 −1 −2

0 0 1 4

−r2 −→ r2−−−−−−−→

1 2 1 3

0 1 1 2

0 0 1 4

−r3 + r2 −→ r2−−−−−−−−−−−→

1 2 1 3

0 1 0 −2

0 0 1 4

−r3 + r1 −→ r1−−−−−−−−−−−→

1 2 0 −1

0 1 0 −2

0 0 1 4

−2r2 + r1 −→ r1−−−−−−−−−−−−→

1 0 0 3

0 1 0 −2

0 0 1 4

=⇒ x =

x1

x2

x3

=

3

−2

4

(8)

A→

0 1 2

0 −2 −4

1 1 1

0 2 4

0 1 2

0 0 0

1 1 1

0 0 0

1 0 −1

0 1 2

0 0 0

0 0 0

= R.

Therefore, the augmented matrix [A |0 ] of Ax = 0 is reduced to [R |0 ]. Rx = 0 is of

the form

x1 −x3 = 0

x2 + 2x3 = 0, giving x1 = x3 and x2 = −2x3. Therefore, we have

x =[

t −2t t]T

where t is an arbitrary scalar as the solution.

(9) (a) With A =

1 −2 1

2 1 8

1 −12 −11

and b =

1

3

−1

, we may reduce the augmented matrix

15

Page 18: AMA SolutionsManual

CHAPTER 2. LINEAR ALGEBRA

of Ax = b as follows:

[A |b ] =

1 −2 1

2 1 8

1 −12 −11

1

3

−1

1 −2 1

0 5 6

0 −10 −12

1

1

−2

1 −2 1

0 1 65

0 0 0

115

0

.

We thus have

x1 −2x2 + x3 = 1

x2 + 65x3 = 1

5

, giving x2 = 15 − 6

5x3 and x1 = 1 + 2x2− x3.

Putting x3 = t, one obtains x2 = 15 − 6

5 t and x1 = 1+2(

15 − 6

5 t)

− t = 75 − 17

5 t. (b) With

A =

3 −2 2

2 1 −3

1 −10 18

and b =

4

5

−8

,

[A |b ] =

3 −2 2

2 1 −3

1 −10 18

4

5

−8

0 28 −52

0 21 −39

1 −10 18

28

21

−8

0 28 −52

0 1 −137

1 −10 18

28

1

−8

1 −10 18

0 1 −137

0 0 0

−8

1

0

1 0 −47

0 1 −137

0 0 0

2

1

0

.

We thus obtain x =[

2 + 47 t 1 + 13

7 t t]T

, where t is an arbitrary scalar.

(c) In this problem, A =

1 1 p

1 p 1

p 1 1

, b =

1

p

p2

.

[A |b ] =

1 1 p

1 p 1

p 1 1

1

p

p2

1 1 p

0 p− 1 1− p

0 1− p 1− p2

1

p− 1

p2 − p

1 1 p

0 p− 1 1− p

0 0 (1− p)(p + 2)

1

p− 1

p2 − 1

.

There are 3 cases:

16

Page 19: AMA SolutionsManual

CHAPTER 2. LINEAR ALGEBRA

(i) If p = 1, the above augmented matrix is reduced to

1 1 1

0 0 0

0 0 0

1

0

0

. Hence the sys-

tem of linear equations is reduced to one single equation x1 + x2 + x3 = 1. Thus

x =[

1− s− t s t]T

, where t and s are arbitrary scalars.

(ii) If p = −2, the augmented matrix becomes

1 1 −2

0 −3 3

0 0 0

1

−3

3

. Hence the correspond-

ing system of linear equations is inconsistent.

(iii) For p 6= 1, p 6= −2, the augmented matrix may be reduced to

1 1 p

0 1 −1

0 0 1

1

1−1−p2+p

.

We conclude that the corresponding linear system has one and only one solution given by

x =[

(1+p)2

2+p1

2+p −1+p2+p

]T.

(10) (a)

1 1 1 1 1 1

−1 −1 0 0 1 −1

−2 −2 0 0 3 1

0 0 1 1 3 3

1 1 2 2 4 4

−r1 + r5 −→ r5

2r1 + r3 −→ r3

r1 + r2 −→ r2−−−−−−−−−−−−−→

1 1 1 1 1 1

0 0 0 0 2 0

0 0 2 2 5 3

0 0 1 1 3 3

0 0 1 1 3 3

r2 ←→ r5−−−−−−→

1 1 1 1 1 1

0 0 1 1 3 3

0 0 2 2 5 3

0 0 1 1 3 3

0 0 0 0 2 0

−r2 + r4 −→ r4

−2r2 + r3 −→ r3−−−−−−−−−−−−−−→

1 1 1 1 1 1

0 0 1 1 3 3

0 0 0 0 −1 −3

0 0 0 0 0 0

0 0 0 0 2 0

r4 ←→ r5

−r3 −→ r3−−−−−−−−−→

1 1 1 1 1 1

0 0 1 1 3 3

0 0 0 0 1 3

0 0 0 0 2 0

0 0 0 0 0 0

−2r3 + r4 −→ r4−−−−−−−−−−−−→

1 1 1 1 1 1

0 0 1 1 3 3

0 0 0 0 1 3

0 0 0 0 0 −6

0 0 0 0 0 0

(b) The system of linear equations is inconsistent. If it is consistent, then there are

three non-zero equations with five variables. Therefore, the system has infinitely

many solutions.

17

Page 20: AMA SolutionsManual

CHAPTER 2. LINEAR ALGEBRA

(11)

1 1 3 0

1 −2 0 0

−2 3 −1 0

−1 2 0 0

2 −3 1 0

−2r1 + r5 −→ r5

r1 + r4 −→ r4

2r1 + r3 −→ r3

−r1 + r2 −→ r2−−−−−−−−−−−−−−→

1 1 3 0

0 −3 −3 0

0 5 5 0

0 3 3 0

0 −5 −5 0

−15r5 −→ r5

13r4 −→ r4

15r3 −→ r3

−13r2 −→ r2

−−−−−−−−−−→

1 1 3 0

0 1 1 0

0 1 1 0

0 1 1 0

0 1 1 0

−r2 + r5 −→ r5

−r2 + r4 −→ r4

−r2 + r3 −→ r3−−−−−−−−−−−−−→

1 1 3 0

0 1 1 0

0 0 0 0

0 0 0 0

0 0 0 0

−r2 + r1 −→ r1−−−−−−−−−−−−−→

1 0 2 0

0 1 1 0

0 0 0 0

0 0 0 0

0 0 0 0

The reduced system of equations is :

x1 + 2x3 = 0 · · · · · · (1)

x2 + x3 = 0 · · · · · · (2)

Let x3 = t, then from (2), x2 = −x3 = −t.

From (1), x1 = −2x3 = −2t.

Therefore, x =

x1

x2

x3

=

−2t

−t

t

.

(12)

1 1 −1 1

2 3 a 3

1 a 3 2

−r1 + r3 −→ r3

−2r1 + r2 −→ r2−−−−−−−−−−−−−−→

1 1 −1 1

0 1 a + 2 1

0 a− 1 4 1

− (a− 1) r2 + r3 −→ r3−−−−−−−−−−−−−−−−−→

1 1 −1 1

0 1 a + 2 1

0 0 − (a + 3) (a− 2) − (a− 2)

(a) If

− (a + 3) (a− 2) = 0

− (a− 2) = 0, then the system is consistent with infinitely many solu-

tion. Therefore, a = 2.

(b) If − (a + 3) (a− 2) 6= 0, then the system is consistent with one and only one solution.

Therefore, a 6= 2 or a 6= −3.

(c) If

− (a + 3) (a− 2) = 0

− (a− 2) 6= 0, then the system is inconsistent. Therefore, a = −3.

(13)

1 −1 1 2

3 1 4a− 1 2

2 a a + 1 2

−2r1 + r3 −→ r3

−3r1 + r2 −→ r2−−−−−−−−−−−−−−→

1 −1 1 2

0 4 4a− 4 −4

0 a + 2 a− 1 −2

14r2 −→ r2−−−−−−−→

18

Page 21: AMA SolutionsManual

CHAPTER 2. LINEAR ALGEBRA

1 −1 1 2

0 1 a− 1 −1

0 a + 2 a− 1 −2

− (a + 2) r2 + r3 −→ r3−−−−−−−−−−−−−−−−−→

1 −1 1 2

0 1 a− 1 −1

0 0 − (a + 1) (a− 1) a

(a) If

− (a + 1) (a− 1) = 0

a = 0, then the system is consistent with infinitely many so-

lutions. However, the above system has no solution. Therefore, the system is not

possible to have infinitely mant solutions.

(b) If − (a + 1) (a− 1) 6= 0, then the system is consistent with one and only one solution.

Therefore, a 6= 1 or a 6= −1.

(c) If

− (a + 1) (a− 1) = 0

a 6= 0, then the system is inconsistent. Therefore, a = 1 or

a = −1.

(14) The augmented matrix

1 1 −1

−a −1 a

a2 1 −a

b1

b2

b3

may be reduced to

1 1 −1

0 a− 1 0

0 1− a2 a(a− 1)

b1

b2 + ab1

b3 − a2b1

———(*)

(a) If a 6= 0, 1, (*) is reduced to

1 1 −1

0 1 0

0 0 1

b1

b2+ab1a−1

b3+(a+1)b2+ab1a(a−1)

, which implies consis-

tency of the linear system. On the other hand, if the system is consistent for any

b1, b2 and b3, then it follows from (*) that a 6= 0, 1.

(b) If a = 0, (*) becomes

1 1 −1

0 1 0

0 0 0

b1

b3

b2 + b3

. It thus follows that the system is

consistent if and only if b2 + b3 = 0.

(c) If a = 1 and b = 0, (*) becomes

1 1 −1

0 0 0

0 0 0

0

0

0

. As such, we have one single

equation x1 + x2 − x3 = 0, and x =[

s− t t s]T

.

19

Page 22: AMA SolutionsManual

CHAPTER 2. LINEAR ALGEBRA

(15)

[A |b ] =

1 3 −2 5

5 8 −5 12

3 16 −11 28

b1

b2

b3

1 3 −2 5

0 −7 5 −13

0 7 −5 13

b1

b2 − 5b1

b3 − 3b1

1 3 −2 5

0 1 −57

137

0 0 0 0

b1

5b1−b27

b3 + b2 − 8b1

.

Therefore, the system is consistent if and only if b3 + b2−8b1 = 0. When b = [1 4 4]T ,

the system is reduced to one with 2 linear equations in 4 unknowns:

x1 +3x2 −2x3 +5x4 = 1

x2 −57x3 +13

7 x4 = 17

.

Therefore, solutions are x =[

− t7 + 4s

7 + 47

5t7 − 13s

7 + 17 t s

]T.

(16)

1 1 1

α β γ

β γ α

γ α β

3

1

1

1

1 1 1

α β γ

β γ α

α + β + γ α + β + γ α + β + γ

3

1

1

3

1 1 1

α β γ

β γ α

0 0 0

3

1

1

3− 3(α + β + γ)

1 1 1

0 β − α γ − α

0 γ − β α− β

0 0 0

3

1− 3α

1− 3β

3− 3(α + β + γ)

.

Therefore, if α+β+γ 6= 1, then the system is inconsistent . Assume now that α+β+γ = 1.

20

Page 23: AMA SolutionsManual

CHAPTER 2. LINEAR ALGEBRA

The augmented matrix now takes the form

1 1 1

0 β − α γ − α

0 γ − β α− β

0 0 0

3

1− 3α

1− 3β

0

.

Case (i) If α = β = γ = 13 , then the system is reduced to a single equation x1+x2+x3 = 3.

Therefore, it has infinitely many solutions.

Case (ii) If α = β 6= γ, then the above matrix may be reduced to

1 1 1

0 1 0

0 0 1

0 0 0

31−3βγ−β1−3αγ−α

0

.

In this case, the system has one and only one solution. Similar, we have the same

conclusion if β = γ 6= α or if α = γ 6= β.

Case (iii) If α, β and γ are all distinct, then

1 1 1

0 β − α γ − α

0 γ − β α− β

0 0 0

3

1− 3α

1− 3β

0

1 1 1

0 1 γ−αβ−α

0 0 δ

0 0 0

31−3αβ−α

η

0

,

where δ =α2 + β2 + γ2 − αβ − βγ − γα

α− βand η = 1− 3β +

(1− 3α)(β − γ)

β − α.

Since δ =(α− β)2 + (β − γ)2 + (γ − α)2

2(α− β)6= 0, the system of linear equations has a unique

solution.

21

Page 24: AMA SolutionsManual

CHAPTER 2. LINEAR ALGEBRA

(17) (a)

1 a b

0 2 c

0 0 −1

1 0 0

0 1 0

0 0 1

1 a b

0 1 c2

0 0 −1

1 0 0

0 12 0

0 0 1

1 a b

0 1 c2

0 0 1

1 0 0

0 12 0

0 0 −1

1 a b

0 1 0

0 0 1

1 0 0

0 12

c2

0 0 −1

1 a 0

0 1 0

0 0 1

1 0 b

0 12

c2

0 0 −1

1 0 0

0 1 0

0 0 1

1 −a2 b− ac

2

0 12

c2

0 0 −1

Therefore,

1 a b

0 2 c

0 0 −1

−1

=

1 −a2 b− ac

2

0 12

c2

0 0 −1

.

(b)

3 1 2

−1 3 −4

4 0 5

1 0 0

0 1 0

0 0 1

0 10 −10

−1 3 −4

0 12 −11

1 3 0

0 1 0

0 4 1

0 1 −1

−1 3 −4

0 12 −11

110

310 0

0 1 0

0 4 1

0 1 −1

−1 0 −1

0 0 1

110

310 0

− 310

110 0

−1210

410 1

0 1 0

−1 0 0

0 0 1

−1110

710 1

−1510

510 1

−1210

410 1

1 0 0

0 1 0

0 0 1

1510 − 5

10 −1

−1110

710 1

−1210

410 1

.

22

Page 25: AMA SolutionsManual

CHAPTER 2. LINEAR ALGEBRA

(c)

−2 3 −1

−1 2 −1

−6 9 −4

1 0 0

0 1 0

0 0 1

0 −1 1

1 −2 1

0 −3 2

1 −2 0

0 −1 0

0 −6 1

0 1 −1

1 0 −1

0 0 1

−1 2 0

−2 3 0

3 0 −1

0 1 0

1 0 0

0 0 1

2 2 −1

1 3 −1

3 0 −1

1 0 0

0 1 0

0 0 1

1 3 −1

2 2 −1

3 0 −1

.

(18) Let ∆ = αδ − βγ.

Case (1) If α 6= 0, a simple calcultion shows that

[

α β

γ δ

]

→[

1 βα

γ δ

]

→[

1 βα

0 ∆α

]

.

If ∆ 6= 0, then the last matrix may be reduced to I by two more elementary row operations,

which implies that A is nonsingular. If ∆ = 0, then the reduced row echelon form of A

is of the form

[

1 βα

0 0

]

, meaning that A is singular.

Case (2) Suppose α = 0. When ∆ 6= 0, then both γ and β are non-zero so that

A =

[

0 β

γ δ

]

→[

γ δ

0 β

]

→[

1 δγ

0 β

]

→[

1 δγ

0 1

]

→ I.

If ∆ = 0, then either γ = 0 or β = 0. Hence either A =

[

0 β

0 δ

]

or A =

[

0 0

γ δ

]

.

Therefore, A is singular in both cases.

23

Page 26: AMA SolutionsManual

CHAPTER 2. LINEAR ALGEBRA

(19)

1 1 1

1 2 4

1 3 9

1 0 0

1 1 0

0 0 1

σ1→

1 1 1

0 1 3

1 3 9

1 0 0

−1 1 0

0 0 1

σ2→

1 1 1

0 1 3

0 2 8

1 0 0

−1 1 0

−1 0 1

σ3→

1 1 1

0 1 3

0 0 2

1 0 0

−1 1 0

1 −2 1

σ4→

1 1 1

0 1 3

0 0 1

1 0 0

−1 1 012 −1 1

2

σ5→

1 1 1

0 1 0

0 0 1

1 0 0

−52 4 −3

212 −1 1

2

σ6→

1 1 0

0 1 0

0 0 1

12 1 −1

2

−52 4 −3

212 −1 1

2

σ7→

1 0 0

0 1 0

0 0 1

3 −3 1

−52 4 −3

212 −1 1

2

.

For 1 ≤ j ≤ 7, let Ej be the elementary matrix obtained by applying σj to I. Let

Fj = E−1j . Since E7E6E5E4E3E2E1A = I, we conclude that A−1 = E7E6E5E4E3E2E1.

Therefore,

A = (E7E6E5E4E3E2E1)−1 = F1F2F3F4F5F6F7.

All Fj can be written down easily. For example,

F1 =

1 0 0

−1 1 0

0 0 1

−1

=

1 0 0

1 1 0

0 0 1

.

(20) (a) −2r1 + r2 −→ r2 : E1 =

1 0 0

−2 1 0

0 0 1

r2 + r3 −→ r3 : E2 =

1 0 0

0 1 0

0 1 1

−r3 −→ r3 : E3 =

1 0 0

0 1 0

0 0 −1

24

Page 27: AMA SolutionsManual

CHAPTER 2. LINEAR ALGEBRA

(b)

1 2 1

0 1 2

0 0 1

= E3E2E1A

=⇒

1 2 1

0 1 2

0 0 1

−1

= (E3E2E1A)−1 = A−1E−11 E−1

2 E−13

=⇒

1 2 1

0 1 2

0 0 1

−1

E3E2E1 = A−1E−11 E−1

2 E−13 E3E2E1 = A−1

A−1 =

1 −2 3

0 1 −2

0 0 1

1 0 0

0 1 0

0 0 −1

1 0 0

0 1 0

0 1 1

1 0 0

−2 1 0

0 0 1

=

11 −5 −3

−6 3 2

2 −1 −1

(21) det(A) =

1 2 3 4

0 −4 −8 −12

0 −8 −16 −24

13 14 15 16

=

1 2 3 4

0 −4 −8 −12

0 0 0 0

13 14 15 16

= 0.

(22) (a) i. x =

1 2 3

3 8 11

1 5 7

−1

−1

1

1

=

12

12 −1

−5 2 −172 −3

2 1

−1

1

1

=

−1

6

−4

.

ii. ∆ =

1 2 3

3 8 11

1 5 7

= 2, ∆1 =

−1 2 3

1 8 11

1 5 7

= −2, ∆2 =

1 −1 3

3 1 11

1 1 7

= 12,

∆3 =

1 2 −1

3 8 1

1 5 1

= −8.

Therefore, x1 =∆1

∆= −1, x2 =

∆2

∆= 6, x3 =

∆3

∆= −4.

25

Page 28: AMA SolutionsManual

CHAPTER 2. LINEAR ALGEBRA

(b) i. A−1 =

1 0 1 0

1 1 0 0

0 0 1 1

0 1 1 1

−1

=

0 1 1 −1

0 0 −1 1

1 −1 −1 1

−1 1 2 −1

x =

x1

x2

x3

x4

= A−1b =

0 1 1 −1

0 0 −1 1

1 −1 −1 1

−1 1 2 −1

1

1

1

0

=

2

−1

−1

2

ii. detA =

1 0 1 0

1 1 0 0

0 0 1 1

0 1 1 1

= −1, detA1 =

1 0 1 0

1 1 0 0

1 0 1 1

0 1 1 1

= −2,

detA2 =

1 1 1 0

1 1 0 0

0 1 1 1

0 0 1 1

= 1, detA3 =

1 0 1 0

1 1 1 0

0 0 1 1

0 1 0 1

= 1,

detA4 =

1 0 1 1

1 1 0 1

0 0 1 1

0 1 1 0

= −2

Therefore, x1 = ∆A1

∆A= −2

−1 = 2, x2 = ∆A2

∆A= −1, x3 = ∆A3

∆A= −1 and

x4 = ∆A4

∆A= 2.

(c) i. A−1 =

1 1 1 1

1 1 0 0

1 0 1 1

0 1 0 1

−1

=

−1 1 1 0

1 0 −1 0

2 −1 −1 −1

−1 0 1 1

x =

x1

x2

x3

x4

= A−1b =

−1 1 1 0

1 0 −1 0

2 −1 −1 −1

−1 0 1 1

1

1

1

1

=

1

0

−1

1

ii. ∆A =

1 1 1 1

1 1 0 0

1 0 1 1

0 1 0 1

= −1, ∆A1 =

1 1 1 1

1 1 0 0

1 0 1 1

1 1 0 1

= −1

26

Page 29: AMA SolutionsManual

CHAPTER 2. LINEAR ALGEBRA

∆A2 =

1 1 1 1

1 1 0 0

1 1 1 1

0 1 0 1

= 0, ∆A3 =

1 1 1 1

1 1 1 0

1 0 1 1

0 1 1 1

= 1

∆A4 =

1 1 1 1

1 1 0 1

1 0 1 1

0 1 0 1

= −1

Therefore, x1 = ∆A1

∆A= −1

−1 = 1, x2 = ∆A2

∆A= 0, x3 = ∆A3

∆A= −1 and x4 =

∆A4

∆A= 1.

(23) (a) By Kirchoff’s Law, one obtains the circuit equations

i1 −i2 −i3 = 0

(R1 + R2 + R3) i1 + R4i2 = E1 + E2

R4i2 −R5i3 = E2

.

Cramer’s rule then yields

i1 =−E1(R4 + R5)− E2R5

∆, i2 =

−E1R5 − E2(R1 + R2 + R3 + R5)

and

i3 =−E1R4 + E2(R1 + R2 + R3)

∆,

where ∆ = −R1R4 −R1R5 −R2R4 −R2R5 −R3R4 −R3R5 −R4R5.

(b) Using Kirchoff’s Law, currents in the circuit satisfy

i1 −i2 −i3 = 0

R2i2 −R3i3 = 0

(R1 + R4) i1 + R3i3 = −E0

.

Using Cramer’s rule, we obtain

i3 = −E0R2

R, i2 = −E0R3

Rand i1 = −E0(R2 + R3)

R,

where R = R1R2 + R1R3 + R2R3 + R2R4 + R3R4.

(24) Let x1, x2 and x3 be the numbers of batches of Milky, Extra Milky and Supreme respec-

27

Page 30: AMA SolutionsManual

CHAPTER 2. LINEAR ALGEBRA

tively. We therefore obtain

x1 + x2 + x3 = 6

x1 + 2x2 + 3x3 = 14

x1 + 4x2 + 9x3 = 36

.

Therefore x1 = 1, x2 = 2, x3 = 3.

(25) x1, x2, x3, x4 satisfy

−x1 + x4 = 100

x1 −x2 = 300

−x3 + x4 = 500

x2 −x3 = 100

.

Solving the system, we obtain x1 = x4 − 100, x2 = x4 − 400, x3 = x4 − 500, where x4 is

any integer ≥ 500.

(26) (a) The matrices are

[

0.8 0.37

0.2 0.63

]

and

[

0.75 0.15

0.25 0.85

]

. The combined network is repre-

sented by

[

0.75 0.15

0.25 0.85

][

0.8 0.37

0.2 0.63

]

=

[

0.63 0.372

0.37 0.628

]

.

(b)

[

v

w

]

=

[

0.75 0.15

0.25 0.85

][

0.8 0.37

0.2 0.63

][

x

y

]

=

[

0.63x + 0.372y

0.37x + 0.628y

]

.

(27) The system may be re-written as

R3 + R4 + R6 −R3 −R4

−R3 R2 + R3 + R5 −R5

−R4 −R5 R1 + R4 + R5

i1

i2

i3

=

E0

0

0

.

Using Cramer’s rule, one has i2 =∆2

∆and i3 =

∆3

∆, where ∆ is the determinant of the

coefficient matrix,

∆2 = E0 (R1R3 + R3R4 + R3R5 + R4R5) ,

∆3 = E0 (R2R4 + R3R4 + R3R5 + R4R5) .

Therefore, i2 = i3 ⇒ ∆2 = ∆3 ⇒ R1R3 = R2R4.

28

Page 31: AMA SolutionsManual

CHAPTER 2. LINEAR ALGEBRA

(28) Consider the homogeneous systems in 3 unknowns

a1 b1 c1

a2 b2 c2

a3 b3 c3

x1

x2

x3

=

0

0

0

and the non-homogeneous system in 2 unknowns

a1 b1

a2 b2

a3 b3

[

y1

y2

]

=

c1

c2

c3

If y =[

α β]T

is a solution of the nonhomogeneous system, then x =[

α β −1]T

is

a solution of the homogeneous system. Therefore, consistency of the nonhomogeneous

system implies existence of nontrivial solutions of the homogeneous system, which also

implies the coefficient matrix of the homogeneous system has zero determinant, i.e.,

det

a1 b1 c1

a2 b2 c2

a3 b3 c3

= 0.

(29) Elementary row operations give

1 3 −2 5

4 1 3 −2

−1 3 −4 7

0

0

0

1 0 1 −1

0 1 −1 2

0 0 0 0

0

0

0

.

Solutions of the homogeneous system are[

−s + t s− 2t s t]T

, where s and t are arbi-

trary scalars. Taking s = 3, t = 1, we have

−2

1

4

−1

+

3

1

3

+ 3

−2

3

−4

+

5

−2

7

= 0.

Hence the given vectors are linearly dependent.

(30) Since ‖u± v‖2 = 〈u± v,u± v〉 = 〈u,u〉 + 〈v,v〉 ± 2 〈u,v〉, we deduce that ‖u + v‖2 +

‖u− v‖2 = 2 〈u, u〉+ 2 〈v,v〉 = 2 ‖u‖2 + 2 ‖v‖2.

29

Page 32: AMA SolutionsManual

CHAPTER 2. LINEAR ALGEBRA

(31)

w =

(〈v,u〉‖u‖2

)

u =(−4)× 1 + 5× (−2) + 6× 3

12 + (−2)2 + 32

1

−2

3

=

27

−47

67

.

Furthermore,

〈v −w,u〉 =

[

−4 5 6]T−[

27 −4

767

]T,[

1 −2 3]T⟩

=

[

−307

397

367

]T,[

1 −2 3]T⟩

= −30

7− 78

7+

108

7= 0.

(32) The jth row of QT is the same as the transpose of the jth column of Q. Therefore, the

entry at the jth row and the kth column of the product QTQ is just∑n

i=1 qij qik. This

implies (a)⇔ (b). The equivalence of (a) and (c) may be similarily proved by considering

the product QQT .

(33) det (A− λI) =

−2− λ 2 −3

2 1− λ −6

−1 0 −λ

= −λ3 − λ2 + 9λ + 9 = − (λ + 3) (λ− 3) (λ + 1)

Therefore, the eigenvalues of A are λ = 3, λ = −1 and λ = −3.

(34)

det (A− λI) =

1− λ 2 1

0 3− λ 1

0 5 −1− λ

= −λ3 + 3λ2 + 6λ− 8

= − (λ− 1) (λ + 2) (λ− 4)

Therefore, the eigenvalues of A are λ = 1, λ = 4 and λ = −2.

(35) When λ = 0, (A− λI)v = 0 =⇒

3 −1 −2 0

2 0 −2 0

2 −1 −1 0

1 0 −1 0

0 1 −1 0

0 0 0 0

and v =

1

1

1

r, where r ∈ R.

30

Page 33: AMA SolutionsManual

CHAPTER 2. LINEAR ALGEBRA

When λ = 1, (A− λI)v = 0 =⇒

2 −1 −2 0

2 −1 −2 0

2 −1 −2 0

1 −12 −1 0

0 0 0 0

0 0 0 0

and v =

12s + t

s

t

=

12

1

0

s +

1

0

1

t, where s, t ∈ R.

Thus, the eigenvectors are

1

1

1

,

12

1

0

and

1

0

1

.

Therefore, D =

0 0 0

0 1 0

0 0 1

and P =

1 12 1

1 1 0

1 0 1

.

AP =

3 −1 −2

2 0 −2

2 −1 −1

1 12 1

1 1 0

1 0 1

=

0 12 1

0 1 0

0 0 1

PD =

1 12 1

1 1 0

1 0 1

0 0 0

0 1 0

0 0 1

=

0 12 1

0 1 0

0 0 1

= AP

(36) When λ = 2, (A− λI)v = 0 =⇒

0 1 −1 0

1 0 −1 0

1 1 −2 0

1 0 −1 0

0 1 −1 0

0 0 0 0

and v =

1

1

1

r, where r ∈ R.

When λ = 1, (A− λI)v = 0 =⇒

1 1 −1 0

1 1 −1 0

1 1 −1 0

1 1 −1 0

0 0 0 0

0 0 0 0

and v =

−s + t

s

t

=

−1

1

0

s +

1

0

1

t, where s, t ∈ R.

Thus, the eigenvectors are

1

1

1

,

−1

1

0

and

1

0

1

.

31

Page 34: AMA SolutionsManual

CHAPTER 2. LINEAR ALGEBRA

Therefore, D =

2 0 0

0 1 0

0 0 1

and P =

1 −1 1

1 1 0

1 0 1

.

AP =

2 1 −1

1 2 −1

1 1 0

1 −1 1

1 1 0

1 0 1

=

2 −1 1

2 1 0

2 0 1

PD =

1 −1 1

1 1 0

1 0 1

2 0 0

0 1 0

0 0 1

=

2 −1 1

2 1 0

2 0 1

= A

(37) (a) The characteristic polynomial

f(λ) = det

−1− λ 2 −3

2 2− λ −6

−1 −2 1− λ

= −λ3 + 2λ2 + 20λ + 24 = (6− λ) (λ + 2)2 .

Eigenvalues are λ = 6 and λ = −2(double root).

(i) For λ = −2, (A − λI) =

1 2 −3

2 4 −6

−1 −2 3

. We use Gaussian elimination to solve

the homogeneous system

1 2 −3

2 4 −6

−1 −2 3

v = 0 and conclude that v1 = [3 0 1]T and

v2 = [−2 1 0]T are corresponding eigenvectors.

(ii) For λ = 6, (A− λI) =

−7 2 −3

2 −4 −6

−1 −2 −5

. Use Gaussian elimination to solve the homo-

geneous system

−7 2 −3

2 −4 −6

−1 −2 −5

v = 0 to obtain v3 = [−1 − 2 1]T as an eigenvector.

Finally, P =

3 −2 −1

0 1 −2

1 0 1

diagonalizes A, i.e., P−1AP =

−2 0 0

0 −2 0

0 0 6

.

32

Page 35: AMA SolutionsManual

CHAPTER 2. LINEAR ALGEBRA

(b)

f(λ) = det

0− λ −2 0

−2 1− λ −2

0 −2 2− λ

= −λ3 + 3λ2 + 6λ− 8 = (1− λ)(λ + 2)(λ− 4).

Eigenvalues are λ = 1, λ = −2, λ = 4.

(i) For λ = 1, (A − λI) =

−1 −2 0

−2 0 −2

0 −2 1

. Solving (A − λI)v = 0, we obtain v1 =

[−2 1 2]T as a corresponding eigenvector.

(ii) For λ = −2, (A − λI) =

2 −2 0

−2 3 −2

0 −2 4

. Gaussian elimination when applied to the

homogeneous system (A−λI)v = 0 gives v2 = [2 2 1]T as a corresponding eigenvector.

(iii) For λ = 4, (A − λI) =

−4 −2 0

−2 −3 −2

0 −2 −2

. We now solve the homogeneous system

(A − λI)v = 0 and obtain the third eigenvector v3 = [1 − 2 2]T . Therefore, P =

−2 2 1

1 2 −2

2 1 2

diagonalizes A, i.e., P−1AP =

1 0 0

0 −2 0

0 0 4

.

(c)

f(λ) = det

4− λ 2 2

2 1− λ −4

2 −4 1− λ

= −λ3 + 6λ2 + 15λ− 100 = − (λ + 4) (λ− 5)2 .

Eigenvalues of A are λ = 5 (double root), −4.

For λ = 5, (A − λI) =

−1 2 2

2 −4 −4

2 −4 −4

. We thus obtain two linearly independent

eigenvectors v1 = [2 0 1]T and v2 = [2 1 0]T by solving the homogeneous system

(A− λI)v = 0.

For λ = −4, (A−λI) =

8 2 2

2 5 −4

2 −4 5

. Therefore, v3 = [1 − 2 − 2]T is a correspond-

33

Page 36: AMA SolutionsManual

CHAPTER 2. LINEAR ALGEBRA

ing eigenvector.

Furthermore, putting P =

2 2 1

0 1 −2

1 0 −2

, we have P−1AP =

5 0 0

0 5 0

0 0 −4

.

(38) (i) (a) det (A− λI) =

2− λ 3

−1 6− λ

= λ2 − 8λ + 15 = (λ− 3) (λ− 5) = 0

Thus, the eigenvalues of A are λ = 3 and λ = 5.

When λ = 3, (A− λI)v = 0 =⇒[

−1 3 0

−1 3 0

]

→[

1 −3 0

0 0 0

]

and v =

[

3

1

]

s, where s ∈ R.

When λ = 5, (A− λI)v = 0 =⇒[

−3 3 0

−1 1 0

]

→[

1 −1 0

0 0 0

]

and v =

[

1

1

]

t, where t ∈ R.

(b) Using the eigenvalues and eigenvectors, Therefore,

D =

[

3 0

0 5

]

and P =

[

3 1

1 1

]

AP =

[

2 3

−1 6

][

3 1

1 1

]

=

[

9 5

3 5

]

PD =

[

3 1

1 1

][

3 0

0 5

]

=

[

9 5

3 5

]

= AP

(c) Dn =

[

λn1 0

0 λn2

]

P−1=

[

3 1

1 1

]−1

=

[

12 −1

2

−12

32

]

A6 − 6A4 + 11A2

= PD6P−1 − 6PD4P−1 + 11PD2P−1 = P(

D6 − 6D4 + 11D2)

P−1

=

[

3 1

1 1

]([

36 0

0 56

]

− 6

[

34 0

0 54

]

+ 11

[

32 0

0 52

])[

12 −1

2

−12

32

]

=

[

−5562 17712

−5904 18054

]

34

Page 37: AMA SolutionsManual

CHAPTER 2. LINEAR ALGEBRA

(ii) (a)

det (A− λI) =

−1− λ 2 0

1 1− λ 0

0 0 1− λ

= −λ3 + λ2 + 3λ− 3

= − (λ− 1)(

λ−√

3)(

λ +√

3)

= 0

Thus, the eigenvalues of A are λ = 1, λ =√

3, and λ = −√

3.

When λ = 1, (A− λI)v = 0 =⇒ v =

0

0

1

r, where r ∈ R.

When λ =√

3, (A− λI)v = 0 =⇒ v =

√3− 1

1

0

s, where s ∈ R.

When λ = −√

3, (A− λI)v = 0 =⇒ v =

−√

3− 1

1

0

t, where t ∈ R.

(b) Using the eigenvalues and eigenvectors, Therefore,

D =

1 0 0

0√

3 0

0 0 −√

3

and P =

0√

3− 1 −√

3− 1

0 1 1

1 0 0

AP =

−1 2 0

1 1 0

0 0 1

0√

3− 1 −√

3− 1

0 1 1

1 0 0

=

0 3−√

3 3 +√

3

0√

3 −√

3

1 0 0

PD =

0√

3− 1 −√

3− 1

0 1 1

1 0 0

1 0 0

0√

3 0

0 0 −√

3

=

0 3−√

3 3 +√

3

0√

3 −√

3

1 0 0

35

Page 38: AMA SolutionsManual

CHAPTER 2. LINEAR ALGEBRA

(c) P−1=

0√

3− 1 −√

3− 1

0 1 1

1 0 0

−1

=

0 0 1√3

63+

√3

6 0

−√

36

3−√

36 0

A6 − 6A4 + 11A2

= P(

D6 − 6D4 + 11D2)

P−1

=

0√

3− 1 −√

3− 1

0 1 1

1 0 0

6 0 0

0 6 0

0 0 6

0 0 1√3

63+

√3

6 0

−√

36

3−√

36 0

=

6 0 0

0 6 0

0 0 6

(39)

det (A− λI) =

4− λ 2 0 4

0 2− λ −1 0

0 0 3− λ 3

0 4 0 7− λ

= λ4 − 16λ3 + 89λ2 − 194λ + 120

= (λ− 1) (λ− 4) (λ− 5) (λ− 6) = 0.

Thus, the fourth eigenvalue of A is λ = 5.

When λ = 5, (A− λI)v = 0 =⇒

−1 2 0 4 0

0 −3 −1 0 0

0 0 −2 3 0

0 4 0 2 0

1 0 0 −3 0

0 1 0 12 0

0 0 1 −32 0

0 0 0 0 0

. There-

fore, v =

3

−12

32

1

s, where s ∈ R. and, the eigenvector associated with λ = 5 is

6

−1

3

2

.

36

Page 39: AMA SolutionsManual

CHAPTER 2. LINEAR ALGEBRA

(40)

det (A− λI) =

1− λ 2 0 2

0 −2− λ −1 0

0 0 3− λ 3

0 0 0 −1− λ

= λ4 − λ3 − 7λ2 + λ + 6

= (λ− 1) (λ− 3) (λ + 2) (λ + 1) = 0

Thus, the eigenvalues of A are λ = 1, λ = 3, λ = −1, and λ = −2.

When λ = 1, v =

1

0

0

0

. When λ = 3, v =

1

1

−5

0

. When λ = −1, v =

7

−3

3

−4

.

When λ = −2, v =

2

−3

0

0

.

(41) Assume A is a matrix with eigenvalue λ and eigenvector x, i.e., Ax = λx.

A2x = A (Ax) = A (λx) = λ (Ax) = λ (λx) = λ2x

(42) (a) For A =

[

0.7 0.4

0.3 0.6

]

, f(λ) = (λ− 0.3)(λ− 1). Therefore, eigenvalues are λ1 = 0.3

and λ2 = 1, with eigenvectors v1 =[

1 −1]T

and v2 =[

4 3]T

respectively.

Taking P =

[

1 4

−1 3

]

, we have A = P

[

0.3 0

0 1

]

P−1. Therefore,

Ak = P

[

0.3k 0

0 1

]

P−1 =

[

1 4

−1 3

][

0.3k 0

0 1

][

37 −4

717

17

]

=1

7

[

4 + 3× 0.3k 4− 4× 0.3k

3− 3× 0.3k 3 + 4× 0.3k

]

.

(b) If the Red Party is in power now, the probability that it is to be in power after k

elections is[

1 0]

Ak

[

1

0

]

=4 + 3× 0.3k

7.

If the Red Party is not in power now, the probability that it is to be in power after

k elections is[

0 1]

Ak

[

0

1

]

3− 3× 0.3k

7.

37

Page 40: AMA SolutionsManual

CHAPTER 2. LINEAR ALGEBRA

When k →∞, the respective probabilities are4

7and

3

7.

(43) (a)

[

1 2

1 0

][

an−1

an−2

]

=

[

an−1 + 2an−2

an−1

]

=

[

an

an−1

]

for n ≥ 3.

(b) A =

[

1 2

1 0

]

has eigenvalues λ1 = 2 and λ2 = −1, with eigenvectors v1 =

[

2

1

]

and v2 =

[

−1

1

]

respectively. Therefore, one obtains

A =

[

2 −1

1 1

][

2 0

0 −1

][

2 −1

1 1

]−1

and

Ak =

[

2 −1

1 1

][

2k 0

0 (−1)k

][

13

13

−13

23

]

=1

3

[

2k+1 + (−1)k 2k+1 − 2× (−1)k

2k − (−1)k 2k + 2× (−1)k

]

.

(c)

[

an

an−1

]

=

[

1 2

1 0

][

an−1

an−2

]

=

[

1 2

1 0

]2 [

an−2

an−3

]

= . . . =

[

1 2

1 0

]n−2 [

a2

a1

]

Therefore, an =2n − (−1)n

3for n ≥ 3.

(44) (a)

Eigenvalues Eigenvectors Orthonormal eigenvectors

λ = 3[

1 1 1]T [

1√3

1√3

1√3

]T

λ = 6[

1 1 −2]T [

1√6

1√6− 2√

6

]T

λ = 8[

1 −1 0]T [

1√2− 1√

20]T

Therefore, Q =

1√3

1√6

1√2

1√3

1√6

−1√2

1√3

−2√6

0

and D =

3 0 0

0 6 0

0 0 8

(b)

Eigenvalues Eigenvectors

λ = 7 (double root)[

1 −2 0]T

and[

0 2 1]T

λ = −2[

2 1 −2]T

38

Page 41: AMA SolutionsManual

CHAPTER 2. LINEAR ALGEBRA

[

1 −2 0]T

and[

0 2 1]T

are not orthogonal to each other, although each of them

is orthogonal to[

2 1 −2]T

. Using Gram Schmidt Process, one obtains 3 orthonor-

mal eigenvectors w1 =[

1√5− 2√

50]T

, w2 =[

43√

52

3√

55

3√

5

]T, w3 =

[

23

13 −2

3

]T.

Therefore, Q =

1√5

43√

523

− 2√5

23√

513

0 53√

5−2

3

and D =

7 0 0

0 7 0

0 0 −2

39

Page 42: AMA SolutionsManual
Page 43: AMA SolutionsManual

Chapter 3

Infinite series, Power series and

Fourier series

(1) (a) Convergent, Comparison with1

n3/2.

∑ 1√

n(n2 + 1)=∑ 1√

n3 + n≤∑ 1√

n3.

(b) Divergent, Comparison with1

n.

∑ π/4

n≤∑ tan−1 n

n

(c) Convergent, Comparison with1

n2.

∑ lnn

n3≤∑ n

n3=∑ 1

n2

(d) Convergent, Comparison with1

n3.

1

(n + 1)(n + 2)(n + 3)≤ 1

n3

(e) Convergent, ratio test

((n+1)!)2

(2(n+1))!

(n!)2

(2n)!

=(2n)!((n + 1)!)2

(2(n + 1))!(n!)2=

(n + 1)2

(2n + 1)(2n + 2)=

n + 1

4n + 2→ 1

4

41

Page 44: AMA SolutionsManual

CHAPTER 3. INFINITE SERIES, POWER SERIES AND FOURIER SERIES

(f) Convergent, ratio test

(n+1)!(n+1)n+1

n!nn

=(n + 1)!nn

n!(n + 1)n+1=

nn

(n + 1)n=

1(

n+1n

)n =1

(

1 + 1n

)n →1

e

(g) Convergent, ratio test

xn+1

(n+1)n+1

xn

nn

=xn+1nn

xn(n + 1)n+1=

xnn

(n + 1)n+1=

nn

(n + 1)n

x

n + 1→ 0

(h) Divergent, integral test

∫ ∞

a

dx

x lnx ln(lnx)= [ln(ln(lnx))]∞a

(i) Convergent, integral test

∫ ∞

a

en

9 + e2ndx =

[

1

3tan−1

(

ex

3

)]∞

a

(j) Divergent, integral test

∫ ∞

a

2ln(ln n)

n lnndx =

[

2ln(ln x)

ln 2

]∞

a

(2) (a) Conditionally.

The series is not absolutely convergent because x < sin−1 x for 0 < x ≤ 1 , i.e.

1

n≤ sin−1

(

1

n

)

.

Since sin−1 x is an increasing continuous function, we have

sin−1

(

1

n

)

> sin−1

(

1

n + 1

)

n→∞−−−→ 0.

By Leibniz test, the series converges conditionally.

(b) Absolutely

(−1)nn3

en

=∑ n3

en,

which converges by ratio test.

42

Page 45: AMA SolutionsManual

CHAPTER 3. INFINITE SERIES, POWER SERIES AND FOURIER SERIES

(c) Absolutely

(−1)n

2 + 2n + n2

=∑ 1

2 + 2n + n2≤∑ 1

n2,

which converges by Comparison test.

(d) Conditionally

(−1)n

n lnn

=∑ 1

n lnn,

which diverges by Integral test. Since x lnx is an increasing function for x > 1,

1

n lnn>

1

(n + 1) ln(n + 1)

n→∞−−−→ 0.

By Leibniz test, the series converges conditionally.

(3) Consider the k-th partial sum:

sk =k∑

n=1

1

(4n− 1)(4n + 3)=

k∑

n=1

[

1

4(4n− 1)− 1

4(4n + 3)

]

=

(

1

12− 1

28

)

+

(

1

28− 1

44

)

+ · · ·+(

1

4(4n− 1)− 1

4(4n + 3)

)

=1

12− 1

4(4k + 3)

→ 1

12

(4) (a) [−1, 1), 0 < s ≤ 1; [−1, 1], s > 1

R = limn→∞

an

an+1

= limn→∞

1ns

1(n+1)s

= limn→∞

(

1 +1

n

)s∣∣

= 1

At x = 1, the series becomes∑ 1

ns which converges if s > 1 and diverges if s ≤ 1.

At x = −1, the series becomes the alternating series∑ (−1)n

ns , which converges

absolutely if s > 1 and conditionally if s ≤ 1.

(b) (−e, e)

R = limn→∞

an

an+1

= limn→∞

n!nn

(n+1)!(n+1)n+1

= limn→∞

(

n + 1

n

)n

= e

43

Page 46: AMA SolutionsManual

CHAPTER 3. INFINITE SERIES, POWER SERIES AND FOURIER SERIES

At x = e, the series becomes∑ n!en

nn , which diverges because

an =n!en

nn=

n!

nn

(

1 + n + · · ·+ nn

n!+ · · ·

)

> 1

and therefore an does not converges to 0.

Similarly, at x = −e, the series becomes the alternating series∑ (−1)nn!en

nn , which

diverges because bn = (−1)nn!en

nn does not converge to 0.

(c) (−1, 1)

R = limn→∞

an

an+1

= limn→∞

n

n + 1

= 1

At x = −1 or x = 1, the terms of the series are (−1)nn and n respectively, and do

not converge to 0.

(d) (−2, 0]

R = limn→∞

an

an+1

= limn→∞

(−1)n−1

n(−1)n

n+1

= limn→∞

n + 1

n= 1

Translating to the left by 1 units, we are interested in the interval (−2, 0)

At x = −2, the series becomes∑ (−1)2n−1

n = −∑ 1n , which is the negative harmonic

series and diverges.

At x = 0, the series becomes∑ (−1)n−1

n , which is the alternating harmonic series and

converges.

(e) (−4, 0)

R = limn→∞

an

an+1

= limn→∞

n2

2n

(n+1)2

2n+1

= limn→∞

2n2

(n + 1)2= 2

Translating to the left by 2 units, we are interested in the interval (−4, 0).

At x = −4 or x = 1, the terms of the series are (−1)2n2 and n2 respectively, and do

not converge to 0.

(5) The Maclaurin Series of a function f(x) is given by

f(x) = f(0) + f ′(0)x +f ′′(0)

2!x2 +

f ′′′(0)

3!x3 +

f (4)(0)

4!x4 + · · ·

(a) 1− 2x + 2x2 − 43x3 + 2

3x4

Note:

e−2x =∞∑

n=0

(−2x)n

n!=

∞∑

n=0

(−2)n

n!xn

44

Page 47: AMA SolutionsManual

CHAPTER 3. INFINITE SERIES, POWER SERIES AND FOURIER SERIES

(b) 2x− 43x3

Note

sin(2x) =∞∑

n=0

(−1)n(2x)2n+1

(2n + 1)!

(c) x2 + x3 + x4

2! + x5

3!

Note

x2ex = x2∞∑

n=0

xn

n!=

∞∑

n=0

xn+2

n!

(d) ln 3 + 23x− 2

9x2 + 881x3 − 4

81x4

Note

ln(3 + 2x) = ln(1 + 2(1 + x)) =∞∑

n=1

(−1)n−1

n[2(1 + x)]n

(6) (a) Let z = ax. Then

ez =

∞∑

n=0

zn

n!=

∞∑

n=0

(ax)n

n!

(b) Let z = x2 . Then

cos z =∞∑

n=0

(−1)nz2n

(2n)!=

∞∑

n=0

(−1)nx2n

4n(2n)!

(c)

ex = 1 + x +x2

2!+

x3

3!+ · · ·+ xn

n!+ · · ·

e−x = 1− x +x2

2!− x3

3!+ · · ·+ xn

n!+ · · ·

cosh x =ex + e−x

2=

∞∑

n=0

x2n

(2n)!

(d)

ln |1 + x| = x− 1

2x2 +

1

3x3 − 1

4x4 + · · ·+ (−1)n−1

nxn + · · ·

ln |1− x| = −x +1

2x2 − 1

3x3 +

1

4x4 + · · ·+ (−1)n

nxn + · · ·

1

2ln

1 + x

1− x

=1

2(ln |1 + x| − ln |1− x|) =

∞∑

n=1

x2n−1

2n− 1

45

Page 48: AMA SolutionsManual

CHAPTER 3. INFINITE SERIES, POWER SERIES AND FOURIER SERIES

(7) (a)∑∞

n=1(−1)n+1

n (x− 1)n

lnx = ln(1 + (x− 1)) =∞∑

n=1

(−1)n+1

n(x− 1)n

(b)∑∞

n=0(−1)(x + 1)n

1

x= − 1

1− (x + 1)= −

∞∑

n=0

(x + 1)n =∞∑

n=0

(−1)(x + 1)n

(c)∑∞

n=0(−1)nπ2n

(2n)!

(

x− 12

)2n

sinπx = cos

[

π

(

x− 1

2

)]

=∞∑

n=0

(−1)n

(2n)!

[

π

(

x− 1

2

)]2n

(8) (a) Since |x| is an even function, all bn = 0.

a0 =1

π

∫ π

−π|x| dx =

1

π

∫ π

0xdx +

1

π

∫ 0

−π−xdx = π

an =1

π

∫ π

−π|x| cos nxdx = 2

1

π

∫ π

0x cos nxdx =

2

π

−1 + cos πn

n2=

2

π

(−1)n − 1

n2

Hence,

f (x) =π

2+

2

π

∞∑

n=1

(−1)n − 1

n2cos nx

Note: Choose f = x, g′ = cos nx and use integration by parts, we get

x cos nxdx = xsinnx

n−∫

sinnx

ndx = x

sinnx

n+

cos nx

n2

(b)

f (x) = sin2 x− 2 cos3 x =1

2(1− cos 2x)− 2

(

cos2 x)

cos x

=1

2(1− cos 2x)− (1 + cos 2x) cos x

=1

2(1− cos 2x)− cos x− cos 2x cos x

=1

2(1− cos 2x)− cos x− 1

2(cos 3x + cos x)

=1

2− 3

2cos x− 1

2cos 2x− 1

2cos 3x

46

Page 49: AMA SolutionsManual

CHAPTER 3. INFINITE SERIES, POWER SERIES AND FOURIER SERIES

Note:

cos 2x = 1− 2 sin2 x = 2 cos2 x− 1

cos mx cos nx =1

2[cos (m + n)x + cos (m− n)x]

(c)

a0 =1

π

∫ π

0sin tdt =

2

π

For n 6= 1,

bn =1

π

∫ π

0sin t sinntdt

=1

π

∫ π

0−1

2[cos (1 + n) t− cos (1− n) t] dt

=

[

−1

2

sin (1 + n) t

1 + n+

1

2

sin (1− n) t

1− n

0

= 0

For n = 1,

b1 =1

π

∫ π

0sin t sin tdt =

1

∫ π

0(1− cos 2t) dt =

[

1

(

t− 1

2sin 2t

)]π

0

=1

2

an =1

π

∫ π

0sin t cos ntdt =

1

π

∫ π

0

1

2[sin (1 + n) t + sin (1− n) t] dt

=

[

1

π

(

−1

2

cos (1 + n) t

1 + n− 1

2

cos (1− n) t

1− n

)]π

0

= − 1

π

cos πn + 1

n2 − 1= − 1

π

(−1)n + 1

n2 − 1

Hence, if n is even, i.e. n = 2m, a2m = − 1π

24m2−1

if n is odd an = 0

f (x) =1

π+

1

2sin t− 1

π

∞∑

m=1

2

4m2 − 1cos 2mt

Note:

sin mx sinnx = −1

2[cos (m + n)x− cos (m− n)x]

sinmx cos nx =1

2[sin (m + n)x + sin (m− n)x]

47

Page 50: AMA SolutionsManual

CHAPTER 3. INFINITE SERIES, POWER SERIES AND FOURIER SERIES

(9) (a)

a0 =2

π

∫ π

0x (π − x) dx =

π2

3

an =2

π

∫ π

0x (π − x) cos nxdx = −2

cos πn + 1

n2= −2

(−1)n + 1

n2

bn =2

π

∫ π

0x (π − x) sinnxdx = − 4

π

cos πn− 1

n3=

4

π

1− (−1)n

n3

The half range cosine series of f (x) is

f (x) =π2

6− 2

∞∑

n=1

(−1)n + 1

n2cos nx

=π2

3− 2

∞∑

m=1

(−1)2m + 1

4m2cos 2mx

=π2

6−

∞∑

m=1

1

m2cos 2mx

The half range sine series of f (x) is

f (x) =4

π

∞∑

n=1

1− (−1)n

n3sinnx =

8

π

∞∑

m=1

1

(2m− 1)3sin (2m− 1) x

(b)

a0 =4

π

∫ π/2

0cos xdx =

4

π

an =4

π

∫ π/2

0cos x cos 2nxdx = − 4

π

cos πn

4n2 − 1=

4

π

(−1)n−1

4n2 − 1

f (x) =2

π+

4

π

∞∑

m=1

(−1)n−1

4n2 − 1cos 2nx

(10) (a) Since |sinx| is an even function, all bn = 0.

a0 =1

π

∫ π

−π|sinx| dx =

1

π

∫ π

0sinxdx +

1

π

∫ 0

−π− sinxdx =

4

π

an =1

π

∫ π

−π|sinx| cos nxdx = 2

1

π

∫ π

0sinx cos nxdx = − 2

π

cos πn + 1

−1 + n2

=2

π

1 + (−1)n

n2 − 1

48

Page 51: AMA SolutionsManual

CHAPTER 3. INFINITE SERIES, POWER SERIES AND FOURIER SERIES

Hence,

f (x) =2

π− 4

π

∞∑

m=1

1

4m2 − 1cos 2mx

(b)

a0 =1

π

∫ π

−πexdx =

1

π

(

eπ − e−π)

an =1

π

∫ π

−πex cos nxdx

=1

π

[(

n2

n2 + 1

)(

ex sinnx

n− ex cos nx

n2

)]π

−π

=1

π

[

n2

n2 + 1

(

eπ cos nπ

n2− e−π cos nπ

n2

)

]

=1

π

[

(−1)n

n2 + 1

(

eπ − e−π)

]

bn =1

π

∫ π

−πex sinnxdx

=

[

ex sinnx− n2

n2 − 1

(

ex sinnx

n− ex cos nx

n2

)]π

0

= −n

π

[

(−1)n

n2 + 1

(

eπ − e−π)

]

f (x) =1

(

eπ − e−π)

+∞∑

n=1

1

π

[

(−1)n

n2 + 1

(

eπ − e−π)

cos nx

]

+∞∑

n=1

n

π

[

(−1)n

n2 + 1

(

eπ − e−π)

sinnx

]

=1

π

(

eπ − e−π)

[

1

2+

∞∑

n=1

(−1)n

n2 + 1(cos nx− n sinnx)

]

=2 sinhπ

π

[

1

2+

∞∑

n=1

(−1)n

n2 + 1(cos nx− n sinnx)

]

ex cos nxdx = ex sinnx

n−∫

ex sinnx

ndx

= ex sinnx

n+ ex cos nx

n2+

(

−ex cos nx

n2

)

dx

49

Page 52: AMA SolutionsManual

CHAPTER 3. INFINITE SERIES, POWER SERIES AND FOURIER SERIES

Therefore,∫

ex cos nxdx =

(

n2

n2 − 1

)(

ex sinnx

n− ex cos nx

n2

)

ex sinnx

ndx = ex sinnx

n−∫

ex cos nxdx

= ex sinnx−(

n

n2 − 1

)(

ex sin nx

n− ex cos nx

n2

)

Note:

sinhx =ex − e−x

2

(11)

a0 =1

π

∫ π

0(x− π)2 dx +

1

π

∫ 2π

ππ2dx =

1

3π2 + π2 =

4

3π2

an =1

π

∫ π

0(x− π)2 cos nxdx +

1

π

∫ 2π

ππ2 cos nxdx

=2

π

− sinπn + πn

n3+

[

πsin nx

n

]2π

π

=2

n2

bn =1

π

∫ π

0(x− π)2 sinnxdx +

1

π

∫ 2π

ππ2 sinnxdx

=1

π

2 cos πn− 2 + n2π2

n3−[

πcos nx

n

]2π

π

=2

π

(−1)n − 1

n3+

π

n− π

n+

(−1)n π

n

=2

π

(−1)n − 1

n3+

(−1)n π

n

f (x) =2

3π2 +

∞∑

n=1

2

n2cos nx +

∞∑

n=1

(

2

π

(−1)n − 1

n3+

(−1)n π

n

)

sinnx

50

Page 53: AMA SolutionsManual

CHAPTER 3. INFINITE SERIES, POWER SERIES AND FOURIER SERIES

Put x = 0

π2 =2

3π2 + 2

∞∑

n=1

1

n2

π2

3= 2

∞∑

n=1

1

n2

∞∑

n=1

1

n2=

π2

6

Put x = π,

π2

2=

1

2

(

f(

π−)+ f(

π+))

=2π2

3+ 2

∞∑

n=1

cos nπ

n2

−π2

6= 2

∞∑

n=1

(−1)n

n2

∞∑

n=1

(−1)n−1

n2=

π2

12

Note:

(x− π)2 cos nxdx = (x− π)2sinnx

n−∫

(2x− 2π)sinnx

ndx

= (x− π)2sinnx

n+ (2x− 2π)

cos nx

n2−∫

(

−2cos nx

n2

)

dx

= (x− π)2sinnx

n+ (2x− 2π)

cos nx

n2+ 2

sin nx

n3∫

(x− π)2 sinnxdx = − (x− π)2cos nx

n+

(

(2x− 2π)cos nx

n

)

dx

= − (x− π)2cos nx

n+ (2x− 2π)

sin nx

n2−∫

2sinnx

n2dx

= − (x− π)2cos nx

n+ (2x− 2π)

sin nx

n2− 2

cos nx

n3

(12) (a)

a0 =1

π

∫ 0

−π−1dx = −1

an =1

π

∫ 0

−π− cos nxdx = −sinπn

πn= 0

bn =1

π

∫ 0

−π− sinnxdx =

1− cos πn

πn=

1− (−1)n

πn

51

Page 54: AMA SolutionsManual

CHAPTER 3. INFINITE SERIES, POWER SERIES AND FOURIER SERIES

f (x) = −1

2+

∞∑

n=1

1− (−1)n

πnsinnx

(b) Since x2 is an even function, all bn = 0.

a0 =

∫ 1

−1x2dx =

2

3

an =

∫ 1

−1x2 cos nπxdx =

[

x2 sinnπx

nπ+ 2x

cos nπx

n2π2+ 2

sinnπx

n3π3

]1

−1

=(−1)n 4

n2π2

Therefore,

f (x) =1

3+

∞∑

n=1

4

n2π2(−1)n cos nπx

Note: Choose f = x2 and g′ = cos nπx,

x2 cos nπxdx = x2 sinnπx

nπ−∫

2xsinnπx

nπdx.

Choose f = 2x and g′ = sinnπx,

2xsinnπx

nπdx = −2x

cos nπx

n2π2−∫

(

−2cos nπx

n2π2

)

dx

= −2xcos nπx

n2π2− 2

sinnπx

n3π3

Hence,∫

x2 cos nπxdx = x2 sin nπx

nπ+ 2x

cos nπx

n2π2+ 2

sin nπx

n3π3

52

Page 55: AMA SolutionsManual

Chapter 4

Partial Differentiation

(1) f (x, y) = 2x−3yx+y

Suppose that (x, y) approaches (0, 0) along the x-axis. Then

limx→0y=0

f (x, y) = limx→0

2x

x= 2.

Suppose that (x, y) approaches (0, 0) along the y-axis. Then

limy→0x=0

f (x, y) = limx→0

−3y

y= −3.

Since the limits are not equal, then the limit does not exist.

(2) w =√

x2 + y2

∂w∂x =

∂(√

x2+y2)

∂x =(

12

) (

x2 + y2)− 1

2 (2x) = x√x2+y2

∂w∂y = y√

x2+y2

Using polar coordinates of the point (x, y), i.e., x = r cos θ and y = r sin θ,

limr→0

r=√

x2+y2

x√

x2 + y2= lim

r→0

r cos θ√

(r cos θ)2 + (r sin θ)2= cos θ

and

limr→0

r=√

x2+y2

y√

x2 + y2= lim

r→0

r sin θ√

(r cos θ)2 + (r sin θ)2= sin θ.

53

Page 56: AMA SolutionsManual

CHAPTER 4. PARTIAL DIFFERENTIATION

Since both of the limits are dependent on θ, then there is no unique value for the derivatives

for w at (0, 0). Therefore, the derivatives do not exist.

(3) By the product formula, we have

∂w

∂t= e−

x2

4t

[

−1

2t−

32 +

x2

4t−

52

]

=∂2w

∂x2.

(4) (a) ∂w∂x = ex cos y + 2x + y, ∂w

∂y = −ex sin y + x − 2y, ∂2w∂x2 = ex cos y + 2, ∂2w

∂y2 =

−ex cos y − 2y, ∂2w∂y∂x = −ex sin y + 1 = ∂2w

∂x∂y .

(b) ∂w∂x = 2x

x2+y2−z2 , ∂w∂y = 2y

x2+y2−z2 , ∂w∂z = −2z

x2+y2−z2 , ∂2w∂x2 = 2(y2−x2−z2)

(x2+y2−z2)2, ∂2w

∂y2 =

2(x2−y2−z2)

(x2+y2−z2)2, ∂2w

∂z2 = −2(x2+y2+z2)

(x2+y2−z2)2, ∂2w

∂x∂y = −4xy

(x2+y2−z2)2, ∂2w

∂z∂y = 4yz

(x2+y2−z2)2, ∂2w

∂z∂x =4zx

(x2+y2−z2)2.

(c)

∂w

∂x= e2x+y cos (x− y) + 2e2x+y sin (x− y) = e2x+y [cos (x− y) + 2 sin (x− y)]

∂w

∂y= −e2x+y cos (x− y) + e2x+y sin(x− y) = e2x+y [sin(x− y)− cos (x− y)]

∂2w

∂x2

= 2e2x+y [cos (x− y) + 2 sin (x− y)] + e2x+y [− sin (x− y) + 2 cos (x− y)]

= e2x+y [4 cos (x− y) + 3 sin (x− y)]

∂2w

∂y2= e2x+y [sin (x− y)− cos (x− y)] + e2x+y [− cos (x− y)− sin (x− y)]

= −2e2x+y cos (x− y)

∂2w

∂x∂y= 2e2x+y [sin (x− y)− cos (x− y)] + e2x+y [cos (x− y) + sin (x− y)]

= e2x+y [3 sin (x− y)− cos (x− y)]

∂2w

∂y∂x

= e2x+y [cos (x− y) + 2 sin (x− y)] + e2x+y [sin (x− y)− 2 cos (x− y)]

= e2x+y [3 sin (x− y)− cos (x− y)]

54

Page 57: AMA SolutionsManual

CHAPTER 4. PARTIAL DIFFERENTIATION

(d) ∂w∂x = 1√

x2+2y2

(

12

) (

x2 + 2y2)−1/2

(2x) = xx2+2y2

∂w∂y = 1√

x2+2y2

(

12

) (

x2 + 2y2)−1/2

(4y) = 2yx2+2y2

∂2w∂x2 =

(x2+2y2)−x(2x)

(x2+2y2)2= 2y2−x2

(x2+2y2)2

∂2w∂y2 =

2(x2+2y2)−2y(4y)

(x2+2y2)2= 2x2−4y2

(x2+2y2)2

∂2w∂x∂y = −2y(2x)

(x2+2y2)2= −4xy

(x2+2y2)2

∂2w∂y∂x = −x(4y)

(x2+2y2)2= −4xy

(x2+2y2)2

(e)

∂w

∂x= 2e2x−y2

sin(x + y) + e2x−y2

cos(x + y) = e2x−y2

[2 sin(x + y) + cos(x + y)]

∂w

∂y= −2ye2x−y2

sin(x + y) + e2x−y2

cos(x + y) = e2x−y2

[cos(x + y)− 2y sin(x + y)]

∂2w

∂x2

= 2e2x−y2

[2 sin(x + y) + cos(x + y)] + e2x−y2

[2 cos(x + y)− sin(x + y)]

= e2x−y2

[4 cos (x + y) + 3 sin (x + y)]

∂2w

∂y2= −2ye2x−y2

[cos(x + y)− 2y sin(x + y)]

+e2x−y2

[− sin(x + y)− 2 sin(x + y)− 2y cos (x + y)]

= e2x−y2 [−4y cos (x + y) +(

4y2 − 3)

sin (x + y)]

∂2w

∂x∂y

= 2e2x−y2

[cos(x + y)− 2y sin(x + y)] + e2x−y2

[− sin(x + y)− 2y cos(x + y)]

= e2x−y2

[(2− 2y) cos (x + y)− (4y + 1) sin (x + y)]

∂2w

∂y∂x

= −2ye2x−y2

[2 sin(x + y) + cos(x + y)] + e2x−y2

[2 cos(x + y)− sin(x + y)]

= e2x−y2

[(2− 2y) cos (x + y)− (4y + 1) sin (x + y)] =∂2w

∂x∂y

(f) ∂w∂x = yx ln y + yexy ln y

55

Page 58: AMA SolutionsManual

CHAPTER 4. PARTIAL DIFFERENTIATION

∂w∂y = xyx−1 + xexy ln y + 1

yexy

∂2w∂x2 = (ln y)2 yx + y2exy ln y

∂2w∂y2 = x (x− 1) yx−2 + x2exy ln y + 2x

y exy − 1y2 exy

∂2w∂x∂y = yx−1 + xyx−1 ln y + exy ln y + xyexy ln y + exy

∂2w∂y∂x = xyx−1 ln y + yx−1 + exy (ln y + 1) + xyexy ln y = ∂2w

∂x∂y

(5) (a) ∂w∂x = sin(2y − 3z2), ∂w

∂y = 2x cos(2y − 3z2), ∂w∂z = −6xz cos(2y − 3z2).

(b) ∂w∂x = 2Ax + Dy + Fz, ∂w

∂y = 2By + Dx + Ez, ∂w∂z = 2Cz + Ey + Fx.

(c) ∂w∂x = −(2y)−sin x)

(y+cos x)2= 2y sin x

(y+cos x)2, ∂w

∂y = 2(y+cos x)−(2y)(1)

(y+cos x)2= 2 cos x

(y+cos x)2.

(d) ∂w∂x = yex2+y2

+ 2x2yex2+y2

, ∂w∂y = xex2+y2

+ 2xy2ex2+y2

.

(e) ∂w∂x =

(x2+2y2)−(x−y)(2x)

(x2+2y2)2= x2+2y2−2x2+2xy

(x2+2y2)2= 2y2−x2+2xy

(x2+2y2)2,

∂w∂y =

(x2+2y2)(−1)−(x−y)(4y)

(x2+2y2)2= −x2−2y2−4xy+4y2

(x2+2y2)2= 2y2−x2−4xy

(x2+2y2)2.

(6) 1R = 1

R1+ 1

R2+ 1

R3= R2R3+R1R3+R1R2

R1R2R3

R = R1R2R3

R2R3+R1R3+R1R2

∂R∂R1

= (R2R3+R1R3+R1R2)(R2R3)−(R1R2R3)(R3+R2)

(R2R3+R1R3+R1R2)2= (R2R3)2

(R2R3+R1R3+R1R2)2,

∂R∂R2

= (R1R3)2

(R2R3+R1R3+R1R2)2, ∂R

∂R3= (R1R2)2

(R2R3+R1R3+R1R2)2.

(7) (a) a2 = c2 + b2 − 2bc cos A =⇒ cos A = c2+b2−a2

2bc or A = cos−1(

c2+b2−a2

2bc

)

∂∂a (cos A) = ∂

∂a

(

c2+b2−a2

2bc

)

=⇒ − sinA∂A∂a = − a

bc =⇒ ∂A∂a = a

bc sin A ,

∂A∂b = c2−a2−b2

2b2c sin A, ∂A

∂c = b2−a2−c2

2bc2 sin A.

(b) sin Aa = sin B

b = sin Cc =⇒ a = b sin A

sin B

∂a∂A = b cos A

sin B , ∂a∂B = −b sin A cos B

(sin B)2.

(8) (a) ∂w∂x = 2xy + 2y2 sinxy + y cos x, ∂w

∂y = x2 − 2 cos xy + 2xy sin xy + sinx,

∂2w∂x2 = 2y + 2y3 cos xy − y sinx, ∂2w

∂y2 = 4x sin xy − 2x2y cos xy,

∂2w∂x∂y = ∂

∂x

(

∂w∂y

)

= 2x + 4y sin xy + 2xy2 cos xy + cos x,

∂2w∂y∂x = ∂

∂y

(

∂w∂x

)

= 2x + 4y sinxy + 2xy2 cos xy + cos x.

(b) ∂w∂x = x

x2+y2 , ∂w∂y = y

x2+y2 , ∂2w∂x2 = y2−x2

(x2+y2)2, ∂2w

∂y2 = x2−y2

(x2+y2)2,

∂2w∂x∂y = ∂

∂x

(

∂w∂y

)

= −2xy

(x2+y2)2, ∂2w

∂y∂x = ∂∂y

(

∂w∂x

)

= −2xy

(x2+y2)2.

(c) ∂w∂x = yxy−1, ∂w

∂y = xy lnx, ∂2w∂x2 = y (y − 1) xy−2, ∂2w

∂y2 = (lnx)2 xy

∂2w∂x∂y = ∂

∂x

(

∂w∂y

)

= xy−1(1 + y lnx), ∂2w∂y∂x = ∂

∂y

(

∂w∂x

)

= xy−1(1 + y lnx).

56

Page 59: AMA SolutionsManual

CHAPTER 4. PARTIAL DIFFERENTIATION

(d) w = x sin2 y + exy = 12x(1− cos 2y) + exy

∂w∂x = sin2 y + yexy = 1

2(1− cos 2y) + yexy, ∂w∂y = x sin 2y + xexy,

∂2w∂x2 = y2exy, ∂2w

∂y2 = 2x cos 2y + x2exy,

∂2w∂x∂y = ∂

∂x

(

∂w∂y

)

= sin 2y + exy + xyexy,

∂2w∂y∂x = ∂

∂y

(

∂w∂x

)

= sin 2y + exy + xyexy.

(e) ∂w∂x =

(y2+sin x+1)(2y−2x)−(2xy−x2)(cos x)

(y2+sin x+1)2, ∂w

∂y =(y2+sin x+1)(2x)−(2xy−x2)(2y)

(y2+sin x+1)2,

∂2w∂x2 =

2 cos2 x(2xy−x2)(y2+sin x+1)3

+sin x(2xy−x2)+4(x−y) cos x

(y2+sin x+1)2− 2

y2+sin x+1,

∂2w∂y2 =

8y2(2xy−x2)(y2+sin x+1)3

+ 2x2−12xy

(y2+sin x+1)2,

∂2w∂x∂y = ∂

∂x

(

∂w∂y

)

=4y cos x(2xy−x2)

(y2+sin x+1)3− (4y2−4xy)+2x cos x

(y2+sin x+1)2+ 2

y2+sin x+1,

∂2w∂y∂x = ∂

∂y

(

∂w∂x

)

=4y cos x(2xy−x2)

(y2+sin x+1)3− (4y2−4xy)+2x cos x

(y2+sin x+1)2+ 2

y2+sin x+1.

(9) f (x, y, z) = cos(

2xy + z2)

fx = −2y sin(2xy + z2)

fxy = −2[

sin(2xy + z2) + 2xy cos(2xy + z2)]

= −2 sin(2xy + z2)− 4xy cos(2xy + z2)

fxyz = −4z cos(2xy + z2) + 8xyz sin(2xy + z2)

fz = −2z sin(2xy + z2)

fzz = −4z2 cos(2xy + z2)− 2 sin(2xy + z2)

fzzx = 8yz2 sin(2xy + z2)− 4y cos(2xy + z2)

(10) ∂w∂x = −x

(x2+y2+z2)32

, ∂2w∂x2 = 2x2−y2−z2

(x2+y2+z2)52

, ∂w∂y = −y

(x2+y2+z2)32

, ∂2w∂y2 = 2y2−x2−z2

(x2+y2+z2)52

,

∂w∂z = −z

(x2+y2+z2)32

, ∂2w∂z2 = 2z2−x2−y2

(x2+y2+z2)52

.

∂2w∂x2 + ∂2w

∂y2 + ∂2w∂z2 = 2x2−y2−z2

(x2+y2+z2)52

+ 2y2−x2−z2

(x2+y2+z2)52

+ 2z2−x2−y2

(x2+y2+z2)52

= 0

(x2+y2+z2)52

= 0

(11) (a) ∂w∂x = A (Ct + D), ∂2w

∂x2 = 0, ∂w∂t = C (Ax + B), ∂2w

∂t2= 0

∂2w∂t2− c2 ∂2w

∂x2 = (0)− c2 (0) = 0

57

Page 60: AMA SolutionsManual

CHAPTER 4. PARTIAL DIFFERENTIATION

(b)

∂w

∂x=(

Akekx −Bke−kx)(

Ceckt + De−ckt)

,

∂2w

∂x2=(

Ak2ekx + Bk2e−kx)(

Ceckt + De−ckt)

= k2(

Aekx + Be−kx)(

Ceckt + De−ckt)

,

∂w

∂t=(

Aekx + Be−kx)(

Cckeckt −Dcke−ckt)

,

∂2w

∂t2=(

Aekx + Be−kx)(

Cc2k2eckt + Dc2k2e−ckt)

= c2k2(

Aekx + Be−kx)(

Ceckt + De−ckt)

,

∂2w

∂t2− c2 ∂2w

∂x2= c2k2

(

Aekx + Be−kx)(

Ceckt + De−ckt)

−(

c2)

k2(

Aekx + Be−kx)(

Ceckt + De−ckt)

= 0

(c) ∂w∂x = g′ (x + ct), ∂2w

∂x2 = g′′ (x + ct), ∂w∂t = cg′ (x + ct), ∂2w

∂t2= c2g′′ (x + ct)

∂2w∂t2− c2 ∂2w

∂x2 =[

c2g′′ (x + ct)]

− c2 [g′′ (x + ct)] = 0

(12) ∂w∂x = 2x

x2+y2 , ∂w∂y = 2y

x2+y2 , ∂2w∂x2 =

2(y2−x2)(x2+y2)2

, ∂2w∂y2 =

2(x2−y2)(x2+y2)2

∂2w∂x2 + ∂2w

∂y2 =2(y2−x2)(x2+y2)2

+2(x2−y2)(x2+y2)2

= 0

(13) (a) fx (x, y) = (x+3y)(2)−(2x−y)(1)

(x+3y)2= 7y

(x+3y)2, fy (x, y) = −7x

(x+3y)2

L.H.S. = xfx (x, y) + yfy (x, y) = (x)[

7y

(x+3y)2

]

+ (y)[

−7x(x+3y)2

]

= 0 = R.H.S.

(b) fxx (x, y) = −14y

(x+3y)3, fyy (x, y) = 42x

(x+3y)3, fxy (x, y) = 7x−21y

(x+3y)3

L.H.S. = x2 −14y

(x+3y)3+ 2xy 7x−21y

(x+3y)3+ y2 42x

(x+3y)3= 0 = R.H.S.

or

∂∂x [xfx (x, y) + yfy (x, y)] = 0 =⇒ fx (x, y) + xfxx (x, y) + yfyx (x, y) = 0 ... (1)

∂∂y [xfx (x, y) + yfy (x, y)] = 0 =⇒ xfxy (x, y) + fy (x, y) + yfyy (x, y) = 0 ... (2)

x× (1) + y × (2) :

xfx (x, y) + x2fxx (x, y) + xyfyx (x, y) + xyfxy (x, y) + yfy (x, y) + y2fyy (x, y) = 0

[xfx (x, y) + yfy (x, y)] + x2fxx (x, y) + xyfyx (x, y) + xyfxy (x, y) + y2fyy (x, y) = 0

x2fxx (x, y) + xyfyx (x, y) + xyfxy (x, y) + y2fyy (x, y) = 0

58

Page 61: AMA SolutionsManual

CHAPTER 4. PARTIAL DIFFERENTIATION

(14) ∂F∂u = ∂f

∂x∂x∂u + ∂f

∂y∂y∂u = ∂f

∂x (2u) + ∂f∂y (2v) = 2u∂f

∂x + 2v ∂f∂y

∂F∂v = ∂f

∂x∂x∂v + ∂f

∂y∂y∂v = ∂f

∂x (−2v) + ∂f∂y (2u) = −2v ∂f

∂x + 2u∂f∂y

(

∂F

∂u

)2

+

(

∂F

∂v

)2

=

(

2u∂f

∂x+ 2v

∂f

∂y

)2

+

(

−2v∂f

∂x+ 2u

∂f

∂y

)2

= 4u2

(

∂f

∂x

)2

+ 4v2

(

∂f

∂y

)2

+ 4v2

(

∂f

∂x

)2

+ 4u2

(

∂f

∂y

)2

= 4(

u2 + v2)

[

(

∂f

∂x

)2

+

(

∂f

∂y

)2]

If u∂F∂u − v ∂F

∂v = 0, then u(

2u∂f∂x + 2v ∂f

∂y

)

− v(

−2v ∂f∂x + 2u∂f

∂y

)

= 0

=⇒ 2(

u2 + v2) ∂f

∂x = 0 =⇒ ∂f∂x = 0 or u = 0 or v = 0.

f (x, y) = g (y), which is dependent on y only. Therefore, f (x, y) is independent of x.

(15) Since ∂w∂x = y(y2−x2)

(x2+y2)2and ∂w

∂y = x(x2−y2)(x2+y2)2

, therefore x∂w∂x + y ∂w

∂y = 0.

Since

∂2w

∂x2=

2xy (x2 − 3y2)

(x2 + y2)3,

∂2w

∂y2=

2xy (y2 − 3x2)

(x2 + y2)3and

∂2w

∂x∂y=−x4 − y4 + 6x2y2

(x2 + y2)3,

therefore, x2 ∂2w∂x2 + 2xy ∂2w

∂x∂y + y2 ∂2w∂y2 = 0.

(16) Let w = g (u, v) with u = xy and v = z

y .

∂w∂x = ∂w

∂u∂u∂x + ∂w

∂v∂v∂x = gu (u, v)

(

1y

)

+ gv (u, v) (0) = 1ygu (u, v)

∂w∂y = ∂w

∂u∂u∂y + ∂w

∂v∂v∂y = gu (u, v)

(

− xy2

)

+ gv (u, v)(

− zy2

)

= − 1y2 [xgu (u, v) + zgv (u, v)]

∂w∂z = ∂w

∂u∂u∂z + ∂w

∂v∂v∂z = gu (u, v) (0) + gv (u, v)

(

1y

)

= 1ygv (u, v)

x∂w

∂x+ y

∂w

∂y+ z

∂w

∂z=

x

ygu (u, v)− 1

y[xgu (u, v) + zgv (u, v)] +

z

ygv (u, v) = 0

(17) ∂w∂x = ∂w

∂u∂u∂x + ∂w

∂v∂v∂x = ∂w

∂u (1) + ∂w∂v (1) = ∂w

∂u + ∂w∂v

∂w∂t = ∂w

∂u∂u∂t + ∂w

∂v∂v∂t = ∂w

∂u (c) + ∂w∂v (−c) = c∂w

∂u − c∂w∂v

∂2w

∂x2=

∂x

∂w

∂u+

∂x

∂w

∂v=

(

∂2w

∂u2

∂u

∂x+

∂w

∂v∂u

∂v

∂x

)

+

(

∂2w

∂u∂v

∂u

∂x+

∂2w

∂v2

∂v

∂x

)

=

(

∂2w

∂u2+

∂2w

∂v∂u

)

+

(

∂2w

∂v2+

∂2w

∂u∂v

)

=∂2w

∂u2+ 2

∂2w

∂u∂v+

∂2w

∂v2

59

Page 62: AMA SolutionsManual

CHAPTER 4. PARTIAL DIFFERENTIATION

∂2w

∂t2= c

∂t

∂w

∂u− c

∂t

∂w

∂v= c

(

∂2w

∂u2

∂u

∂t+

∂w

∂v∂u

∂v

∂t

)

− c

(

∂2w

∂u∂v

∂u

∂t+

∂2w

∂v2

∂v

∂t

)

= c

(

c∂2w

∂u2− c

∂2w

∂v∂u

)

− c

(

c∂2w

∂u∂v− c

∂2w

∂v2

)

= c2 ∂2w

∂u2− 2c2 ∂2w

∂u∂v+ c2 ∂2w

∂v2

If c2 ∂2w∂x2 = ∂2w

∂t2, then c2

(

∂2w∂u2 + 2 ∂2w

∂u∂v + ∂2w∂v2

)

= c2 ∂2w∂u2 − 2c2 ∂2w

∂u∂v + c2 ∂2w∂v2

=⇒ 4c2 ∂2w∂u∂v = 0 =⇒ ∂2w

∂u∂v = 0.

(18) ∂w∂x = dw

du∂u∂x = f ′ (u) (2x)

∂w∂y = dw

du∂u∂y = f ′ (u) (2y)

y ∂w∂x = 2xyf ′ (u)

x∂w∂y = 2xyf ′ (u) = y ∂w

∂x

(19) ∂w∂x = ∂w

∂u∂u∂x + ∂w

∂v∂v∂x = ∂w

∂u (1) + ∂w∂v (1) = ∂w

∂u + ∂w∂v

∂w∂y = ∂w

∂u∂u∂y + ∂w

∂v∂v∂y = ∂w

∂u (−1) + ∂w∂v (1) = −∂w

∂u + ∂w∂v

Thus,(

∂w∂x

)

(

∂w∂y

)

=(

∂w∂u + ∂w

∂v

) (

−∂w∂u + ∂w

∂v

)

= −(

∂w∂u

)2+(

∂w∂v

)2.

(20) ∂f∂x = ∂f

∂u × ∂u∂x + ∂f

∂v × ∂v∂x + ∂f

∂w × ∂w∂x = ∂f

∂u −∂f∂w

∂f∂y = ∂f

∂u × ∂u∂y + ∂f

∂v × ∂v∂y + ∂f

∂w × ∂w∂y = −∂f

∂u + ∂f∂v

∂f∂z = ∂f

∂u × ∂u∂z + ∂f

∂v × ∂v∂z + ∂f

∂w × ∂w∂z = −∂f

∂v + ∂f∂w

Thus, ∂f∂x + ∂f

∂y + ∂f∂z =

(

∂f∂u −

∂f∂w

)

+(

−∂f∂u + ∂f

∂v

)

+(

−∂f∂v + ∂f

∂w

)

= 0.

(21) ∂w∂x = g

(

xy

)

∂∂x

√xy +

√xy ∂

∂xg(

xy

)

= 12

yxg(

xy

)

+√

xy g′(

xy

)

∂w∂y = g

(

xy

)

∂∂y

√xy +

√xy ∂

∂yg(

xy

)

= 12

xy g(

xy

)

−√

x3

y3 g′(

xy

)

Thus,

x∂w

∂x+ y

∂w

∂y− w

= x

[

1

2

y

xg

(

x

y

)

+

x

yg′(

x

y

)]

+ y

[

1

2

x

yg

(

x

y

)

−√

x3

y3g′(

x

y

)

]

−[√

xyg

(

x

y

)]

= 0.

(22) With w = g(u) and u = x− ct, one has

∂w

∂x=

dw

du

∂u

∂x= g′(u),

60

Page 63: AMA SolutionsManual

CHAPTER 4. PARTIAL DIFFERENTIATION

∂w

∂t=

dw

du

∂u

∂t= −c g′(u),

∂2w

∂x2=

∂x

(

∂w

∂x

)

=∂

∂x

(

g′(u))

=d

du

(

g′(u)) ∂u

∂x= g′′(u)

and∂2w

∂t2=

∂t

(

∂w

∂t

)

=∂

∂t

(

−cg′(u))

= (−c)d

du

(

g′(u)) ∂u

∂t= (−c)2g′′(u).

We therefore conclude that∂2w

∂t2= c2 ∂2w

∂x2.

(23) With w = g(u) and u = x2 − t, one has

∂w

∂x=

dw

du

∂u

∂x= g′(u)2x

∂w

∂t=

dw

du

∂u

∂t= g′(u)(−1)

∂2w

∂x2

∂x

(

∂w

∂x

)

= 2∂

∂x

(

xg′(u))

= 2g′(u) + 2x∂

∂x

(

g′(u))

= 2g′(u) + 4x2g′′(u)

and∂2w

∂t2=

∂t

(

∂w

∂t

)

=∂

∂t

(

−g′(u))

= (−1)2g′′(u).

Hence the result.

(24) ∂w∂s = ∂w

∂x∂x∂s + ∂w

∂y∂y∂s = 4xy3s + 6x2y2s,

∂w∂t = ∂w

∂x∂x∂t + ∂w

∂y∂y∂t = 4xy3t− 6x2y2t.

(25) (a) f(x, y) =√

9x2 + y2

f ≃ ∂f∂x (x0,y0)

x + ∂f∂y (x0,y0)

y

∂f∂x =

(

12

) (

9x2 + y2)− 1

2 (9) (2x) = 9x√9x2+y2

∂f∂y =

(

12

) (

9x2 + y2)− 1

2 (2y) = y√9x2+y2

f(1.95, 8.1) ≃ f(2, 8) +f = f(2, 8) +∂f

∂x (x0,y0)x +

∂f

∂y (x0,y0)

y

= 10 + (1.8)(−0.05) + (0.8)(0.1) = 9.99

Note:√

9 (1.95)2 + (8.1)2 = 9.9916.

(b) Let z = f (x, y) = 13√

x2+y2. zx = −2x

3(x2+y2)4/3 and zy = −2y

3(x2+y2)4/3 .

dz = −2(x∆x+y∆y)

3(x2+y2)4/3 = −2[(2)(−0.1)+(2)(0.04)]

3[(2)2+(2)2]4/3 = 0.005

61

Page 64: AMA SolutionsManual

CHAPTER 4. PARTIAL DIFFERENTIATION

f (1.9, 2.04) ≃ f (2, 2) + dz = 12 + 0.005 = 0.505

Note: 13√

1.92+2.042= 0.50485.

(c) Let z = f (x, y) = 1

(x2+y2)3/2 , then ∂z∂x = −3x

(x2+y2)5/2 and ∂z∂y = −3y

(x2+y2)5/2 .

1

(4.102 + 2.952)3/2= f (4.10, 2.95) = f (4 + 0.1, 3− 0.05)

≃ f (4, 3) +∂z

∂x (4,3)∆x +

∂z

∂y (4,3)

∆y

=1

125+

(−12

3125

)

(0.1) +

( −9

3125

)

(−0.05) = 0.00776

Note: 1

(4.102+2.952)3/2 = 0.0077602.

(d) f (x, y, z) = 3√

x2 + y2 + z2

fx = 2x

3(x2+y2+z2)2/3 , fy = 2y

3(x2+y2+z2)2/3 , fz = 2z

3(x2+y2+z2)2/3

3√

4.982 + 1.052 + 0.962 = f (4.98, 1.05, 0.96) = f (5− 0.02, 1 + 0.05, 1− 0.04)

≃ f (5, 1, 1) +∂f

∂x (5,1,1)∆x +

∂f

∂y (5,1,1)

∆y +∂f

∂z (5,1,1)∆z

= 3 +2 [(5) (−0.02) + (1) (0.05) + (1) (−0.04)]

27= 2.9933

Note: 3√

4.982 + 1.052 + 0.962 = 2.9935.

(26) fx (x, y) = 2x + 2y − 4

fy (x, y) = 2x

f(0.99, 2.04) ≃ f(1, 2) +f = f(1, 2) + fx (x0, y0)x + fy (x0, y0)y

= (1) + (2) (−0.01) + (2) (0.04) = 1.06

Note: f(0.99, 2.04) = 1. 0593.

(27) (a)

∂w

∂ρ=

∂w

∂x

∂x

∂ρ+

∂w

∂y

∂y

∂ρ+

∂w

∂z

∂z

∂ρ

= (2x) (sinφ cos θ) + (2y) (sin φ sin θ) + (−2z) (cos φ)

= 2 (ρ sinφ cos θ) (sinφ cos θ) + 2 (ρ sinφ sin θ) (sinφ sin θ)− 2 (ρ cos φ) (cos φ)

= 2ρ sin2 φ cos2 θ + 2ρ sin2 φ sin2 θ − 2ρ cos2 φ = −2ρ(

cos2 φ− sin2 φ)

= −2ρ cos 2φ

62

Page 65: AMA SolutionsManual

CHAPTER 4. PARTIAL DIFFERENTIATION

(b)

∂w

∂θ=

∂w

∂x

∂x

∂θ+

∂w

∂y

∂y

∂θ+

∂w

∂z

∂z

∂θ

= (2x) (−ρ sinφ sin θ) + (2y) (ρ sinφ cos θ) + (−2z) (0)

= 2 (ρ sinφ cos θ) (−ρ sinφ sin θ) + 2 (ρ sinφ sin θ) (ρ sinφ cos θ)

= −2ρ2 sin2 φ sin θ cos θ + 2ρ2 sin2 φ sin θ cos θ = 0

(28) Let f (x, y) = 11+x−y , then fx = −1

(1+x−y)2and fx = 1

(1+x−y)2.

f (0, 0) = 1, fx (0, 0) = −1 and fy (0, 0) = 1

Using the Taylor’s formula,

f (x, y) ≃ f (x0, y0) + [(x− x0) fx (x0, y0) + (y − y0) fy (x0, y0)]

= 1 + [(x− 0) (−1) + (y − 0) (1)] = 1− x + y.

(29) ∆w ≃ ∂w∂x ∆x + ∂w

∂y ∆y + ∂w∂z ∆z. With w = x2ye3z, we have ∂w

∂x = −2, ∂w∂y = 1 and

∂w∂z = −3 at the point (1,−1, 0). When ∆x = 0.01, ∆y = −0.03 and ∆z = 0.02, the linear

approximation formula gives ∆w ≃ −0.11.

(30) Let f (x, y) =√

x2 + y2, then fx (x, y) = x√x2+y2

and fy (x, y) = y√x2+y2

.

f ≈ fx (x, y)x + fy (x, y)y = x(x)+y(y)√x2+y2

ff

∣≈∣

x(x)+y(y)√x2+y2

1√x2+y2

=∣

x(x)+y(y)x2+y2

∣= x2|x/x|+y2|y/y|

x2+y2 = (0.005)x2+(0.005)y2

x2+y2 =

0.005 = 0.5%

(31) T ≃ dT =∣

∂T∂l (l0,g0)

∣|l|+

∂T∂g (l0,g0)

∣|g|

∂T∂l = 2π

1g

(

12√

l

)

= π√gl

, ∂T∂g = 2π

√l(

− 12g3/2

)

= −π√

lg3

T ≃ dT = π√(3.5)(9.79)

(0.05) +∣

∣−π√

(3.5)

(9.79)3

∣(0.05) = 0.036428

(32) 1R = 1

R1+ 1

R2⇒ R = R1R2

R1+R2and

∆R ≃ ∂R

∂R1∆R1 +

∂R

∂R2∆R2 =

R22

(R1 + R2)2 ∆R1 +

R21

(R1 + R2)2 ∆R2.

When R1 = 3, ∆R1 = 0.2, R2 = 8 and ∆R2 = −0.5, we obtain

∆R ≃ 64

121× 0.2 +

9

121× (−0.5) = 0.0686.

63

Page 66: AMA SolutionsManual

CHAPTER 4. PARTIAL DIFFERENTIATION

(33) ∆A ∼= ∂A∂a ∆a + ∂A

∂b ∆b + ∂A∂θ ∆θ = b sin θ

2 ∆a + a sin θ2 ∆b + ab cos θ

2 ∆θ. Therefore, one has

|∆A| ≤ 200

2sin

π

3|∆a|+ 150

2sin

π

3|∆b|+ 150× 200

2cos

π

3|∆θ|

≤ 43.75√

3 + 7500× 1.5× π

180≃ 272.1

(34) Let S = f (x, y) = xx−y , then ∂S

∂x = −y

(x−y)2and ∂S

∂y = x(x−y)2

.

|∆S| ≃ |dS| =∣

∣fx(9,5)

∣ |∆x|+∣

∣fy(9,5)

∣ |∆y|

=

− 5

(9− 5)2

|0.05|+∣

9

(9− 5)2

|0.1| = 0.071875 ≤ 0.072

(35) Let Q = f (P, R, V, L) = πPR4

5V L , then ∂Q∂P = πR4

5V L = QP , ∂Q

∂R = 4πPR3

5V L = 4QR , ∂Q

∂V =

− 1V

πPR4

5V L = −QV , and ∂Q

∂L = − 1L

πPR4

5V L = −QL

∆Q

Q

≃∣

dQ

Q

=1

Q

[(

Q

P

)

∆P +

(

4Q

R

)

∆R +

(

−Q

V

)

∆V +

(

−Q

L

)

∆L

]

=

∆P

P

+ 4

∆R

R

+

−∆V

V

+

−∆L

L

= (0.5%) + 4 (0.25%) + (0.15%) + (0.3%) = 1.95%

(36) (a) fx (x, y) = −2x cos(

y2 − x2)

fx (1, 1) = −2

fy (x, y) = 2y cos(

y2 − x2)

fy (1, 1) = 2

(b)

f (x, y) ≈ f (1, 1) + [(x− 1) fx (1, 1) + (y − 1) fy (1, 1)]

= (0) + (x− 1) (−2) + (y − 1) (2) = −2x + 2y

f(

0, 12

)

≈ −2 (0) + 2(

12

)

= 1

(c) fxx (x, y) = −2 cos(

y2 − x2)

− 4x2 sin(

y2 − x2)

fxx (1, 1) = −2

fyy (x, y) = 2 cos(

y2 − x2)

− 4y2 sin(

y2 − x2)

fyy (1, 1) = 2

fxy (x, y) = 4xy sin(

y2 − x2)

fyx (x, y) = 4xy sin(

y2 − x2)

= fxy (x, y)

64

Page 67: AMA SolutionsManual

CHAPTER 4. PARTIAL DIFFERENTIATION

fxy (1, 1) = 0

f (x, y) ≈ f (1, 1) + [(x− 1) fx (1, 1) + (y − 1) fy (1, 1)]

+1

2!

[

(x− 1)2 fxx (1, 1) + 2 (x− 1) (y − 1) fxy (1, 1) + (y − 1)2 fyy (1, 1)]

= (0) + (x− 1) (−2) + (y − 1) (2) +1

2

[

(x− 1)2 (−2) + 0 + (y − 1)2 (2)]

= −2x + 2y − (x− 1)2 + (y − 1)2

f(

0, 12

)

≈ −2 (0) + 2(

12

)

− (0− 1)2 +(

12 − 1

)2= 1

4

Note: f(

0, 12

)

= sin(

14

)

= 0.2474.

(37) We calculate the partial derivatives of f(x, y) at (0, 1) as follows:

fx(0, 1) = 14 , fy(0, 1) = −1

4 , fxx(0, 1) = 14 , fyy(0, 1) = 1

4 , fxy(0, 1) = −14 .

Since f(0, 1) = 12 , Taylor’s Formula then yields

f(x, y) ∼= 1

2+

1

4x− 1

4(y − 1) +

1

2

[

1

4x2 − 1

2x(y − 1) +

1

4(y − 1)2

]

.

(38) (a) Solving the equations fx = 4x3 − 4y = 0 and fy = 4y3 − 4x = 0, one obtains

(0, 0), (1, 1) and (−1,−1) as the critical points of the function. Using the second

order derivative test with fxx = 12x2, fxy = −4 and fyy = 12y2, we conclude that

H = fxxfyy − f2xy = 144x2y2 − 16. At (0, 0), H = −16 < 0. We therefore conclude

that (0, 0) is a saddle point. At (1, 1) and (−1,−1), H = 128 > 0 and A = 12 > 0.

As such, both are relative minimum points.

(b) Solving the equations fx = 12x + 6y − 6x2 = 0 and fy = 6x + 6y = 0, one obtains

(0, 0) and (1,−1) as the critical points. Since fxx = 12− 12x, fxy = 6 and fyy = 6,

we conclude that H = fxxfyy − f2xy = 72(1 − x) − 36. At (0, 0), H = 36 > 0 and

fxx = 12 > 0. Thus (0, 0) is a relative minimum. At (1,−1), H = −36 < 0 and

therefore (1,−1) is a saddle point.

(c) f (x, y) = 9x3 − 4xy + 13y3

fx = 27x2 − 4y fy = −4x + y2 fxx = 54x fyy = 2y fxy = fyx = −4

fx = 0 =⇒ y = 274 x2

fy = 0 =⇒ x = 14y2

=⇒ (x, y) = (0, 0) or(

49 , 4

3

)

At (0, 0), H = (0) (0)− (−4)2 = −16 < 0 A = 0.

At(

49 , 4

3

)

, H = (24)(

83

)

− (−4)2 = 48 > 0 A = 24 > 0.

Thus, (0, 0) is a saddle point and(

49 , 4

3

)

is a local minimum.

65

Page 68: AMA SolutionsManual

CHAPTER 4. PARTIAL DIFFERENTIATION

(d) Solve

∂w∂x = 3x2 − 96y = 0

∂w∂y = 192y2 − 96x = 0

to obtain two critical points (0, 0) and (8, 2) for

the given function. As ∂2w∂x2 = 6x, ∂2w

∂y2 = 384y and ∂2w∂x∂y = −96, one obtains

A B C H Conclusion

(0, 0) 0 −96 0 −9216 saddle point

(8, 2) 48 −96 768 27648 relative minimum

(e) Solve

∂w∂x = 2x + 4 = 0

∂w∂y = −6y + 6y2 − 12 = 0

to obtain critical points (−2, 2) and (−2,−1)

for the given function. As ∂2w∂x2 = 2, ∂2w

∂y2 = 12y − 6 and ∂2w∂x∂y = 0, one obtains

A B C H Conclusion

(−2, 2) 2 0 18 36 relative minimum

(−2,−1) 2 0 −18 −36 saddle point

(f) fx = 3x2 − 12, fy = 3y2 − 3, fxx = 6x, fyy = 6y and fxy = fyx = 0.[

fx = 0 =⇒ x = ±2

fy = 0 =⇒ y = ±1

]

=⇒ (x, y) = (2, 1) or (2,−1) or (−2, 1) or (−2,−1)

At (2, 1), H = (12) (6)− (0)2 = 96 > 0 A = 12 > 0.

At (2,−1), H = (12) (−6)− (0)2 = −96 < 0 A = 12 > 0.

At (−2, 1), H = (−12) (6)− (0)2 = −96 < 0 A = −12 < 0.

At (−2,−1), H = (−12) (−6)− (0)2 = 96 > 0 A = −12 < 0.

Thus, (2,−1) and (−2, 1) are saddle points, (2, 1) is a local minimum, and (−2,−1)

is a local maximum.

(g) Solve

fx = 6xy − 6x = 0

fy = 3x2 + 3y2 − 6y = 0

to obtain four critical points (0, 0), (0, 2) and

(±1, 1) for the given function. As fxx = 6y − 6, fyy = 6y − 6 and fxy = fyx = 6x,

thenf H A Nature

(0, 0) 2 36 −6 relative maximum

(0, 2) −2 36 6 relative minimum

(±1, 1) 0 −36 saddle point

66

Page 69: AMA SolutionsManual

CHAPTER 4. PARTIAL DIFFERENTIATION

(h) fx = y − 64x2 , fy = x− 27

y2 , fxx = 128x3 , fyy = 54

y3 and fxy = fyx = 1

fx = 0 =⇒ y = 64x2

fy = 0 =⇒ x = 27y2

=⇒ (x, y) =(

163 , 9

4

)

At(

163 , 9

4

)

, H =(

2732

) (

12827

)

− (1)2 = 3 > 0 A = 2732 > 0.

Thus,(

163 , 9

4

)

is a local minimum.

(i) Since

fx = cos x− cos(x + y) = 0

fy = cos y − cos(x + y) = 0,

we conclude that cosx−cos y = 0, or sin(x−y

2

)

sin(x+y

2

)

= 0. Therefore, x−y = 2nπ

or x+y = 2mπ for some integers m, n. As x, y are required to satisfy 0 < x+y < 2π,

the second possibility is ruled out, and we have x = y + 2nπ for some integer n.

Substituting this into the equation cosx− cos(x+ y) = 0, one has cos y− cos 2y = 0,

or 2 cos2 y − cos y − 1 = 0. Solving this quadratic equation, one obtains cos y = −12

or cos y = 1.

If cos y = 1, then y = 2kπ and x = (2k+2n)π where n, k are integers. This however

implies that x + y = (4k + 2n)π, which is impossible because 0 < x + y < 2π.

If cos y = −12 , then y = 2π

3 + 2kπ or y = 4π3 + 2lπ for some integers k and l. Let us

consider the following two cases:

i. When y = 2π3 + 2kπ, one has x = 2π

3 + (2k + 2n)π and therefore x + y =4π3 +(4k +2n)π. The condition 0 < x+ y < 2π forces 4k +2n = 0, or n = −2k.

As such, the critical points are given by (2π3 − 2kπ, 2π

3 + 2kπ) for any integer k.

ii. When y = 4π3 + 2kπ, one has x = 4π

3 + (2k + 2n)π and therefore x + y =8π3 + (4k + 2n)π. Once again, the condition 0 < x + y < 2π ⇒ −2k − 4

3 < n <

−2k − 13 , or n = −2k − 1. We therefore conclude that the critical points are

(4π3 − 2kπ − 2π, 4π

3 + 2kπ) for any integer k.

A simple calculation shows that fxx = − sinx+sin(x+ y), fyy = − sin y +sin(x+ y)

and fxy = sin(x + y). At (2π3 − 2kπ, 2π

3 + 2kπ), fxx = fyy = −√

3 and fxy =

−√

32 . Therefore, those critical points are relative maximum. On the other hand, at

(4π3 −2kπ−2π, 4π

3 +2kπ), one has fxx = fyy =√

3 and fxy =√

32 . We thus conclude

that those critical points are relative minimum.

(39) The cost C is given by C = 9xy + 16z(x + y), with xyz = 36. With z = 36xy , we conclude

that the cost is given by C = 9xy + 576y + 576

x . By solving equations ∂C∂x = 0 and ∂C

∂y = 0,

one obtains x = 4 and y = 4 as the critical point of C (as a function of two variables x,

y), and second order derivative test confirms that (4, 4) is a relative minimum. As such,

the optimal dimensions of the box are 4m×4m×9m.

67

Page 70: AMA SolutionsManual

CHAPTER 4. PARTIAL DIFFERENTIATION

(40) Solving the equations fx = y + 2− 2x = 0 and fy = x− 1

y = 0, we obtain (12 , 2) is the only

critical point of the function f(x, y) = xy + 2x − lnx2y. Since fxx = 2x2 , fyy = 1

y2 and

fxy = 1, we have fxx(12 , 2) fyy(

12 , 2)−

[

fxy(12 , 2)

]2= 8× 1

4 − 1 = 3 > 0. The point is thus

a relative minimum.

(41) (a) R (2) =[

8, 2, 2√

6]

R′ (t) =[

3t2, 1,√

6t]

R′ (2) =[

12, 1, 2√

6]

Equation of the line tangent: x−812 = y−2

1 = z−2√

62√

6.

(b)

Arc length =

∫ 3

0

∣R′ (t)∣

∣ dt =

∫ 3

0

(3t2)2 + (1)2 +(√

6t)2

dt

=

∫ 3

0

9t4 + 6t2 + 1 dt

=

∫ 3

0

(

3t2 + 1)

dt =[

t3 + t]3

0= 30

(42) (a) 2 (x− 1) + 3 (y − 2)− 1 (z − 1) = 0 =⇒ 2x + 3y − z = 7

(b) 3 (x + 3)− 2 (y + 1) + 2 (z − 1) = 0 =⇒ 3x− 2y + 2z = −5

(c) 3 (x− 2)− (y + 2)− 2 (z − 3) = 0 =⇒ 3x− y − 2z = 2

(43) (a) Since 22 + 2× 12 = 6, P0 = (2, 1, 6) is a point on the surface. Putting

f(x, y, z) = x2 + 2y2 − z,

one has ∇f = 2xi + 4yj − k. Thus ∇f(2, 1, 6) = 4i + 4j − k, and this is a vector

normal to the surface at P0. Finally, equation of the tangent plane is given by

4(x− 2) + 4(y − 1)− (z − 6) = 0,

or 4x + 4y − z = 6.

(b) Since z(0, 0, 1) = (0) cos 0 − (0) e0 + 1 = 1, P0 = (0, 0, 1) is a point on the surface.

Putting

f(x, y, z) = x cos y − yex + 1− z,

one has ∇f = (cos y − yex) i + (−x sin y − ex) j − k. Thus ∇f(0, 0, 1) = i− j − k,

and this is a vector normal to the surface at P0.

68

Page 71: AMA SolutionsManual

CHAPTER 4. PARTIAL DIFFERENTIATION

Finally, equation of the tangent plane is given by

(x− 0)− (y − 0)− (z − 1) = 0,

or x− y − z = 1.

(44) (a) x = 1 + 3t, y = 2− 2t.

(b) x = 1 + t, y = 2 + 3t, z = 3− 2t.

(c) x = −3 + t, y = 2− 2t, z = 2 + t.

(45) (a) Let f (x, y, z) = xy + yz + zx− 11, then ∇f (x, y, z) = 〈y + z, x + z, x + y〉 and

∇f (1, 2, 3) = 〈5, 4, 3〉 .

Equation of the tangent plane 5 (x− 1)+4 (y − 2)+3 (z − 3) = 0 i.e., 5x+4y+3z = 22

(b) Let f (x, y, z) = x3 + y3 + z3 − 18, then ∇f (x, y, z) =⟨

3x2, 3y2, 3z2⟩

and

∇f (−2, 3,−1) = 〈12, 27, 3〉 .

Equation of the tangent plane 12 (x + 2)+27 (y − 3)+3 (z + 1) = 0 i.e., 4x+9y+z =

18

(c) Let f (x, y, z) = zey sinx− 1, then ∇f (x, y, z) = 〈zey cos x, zey sin x, ey sinx〉 and

∇f(π

2, 0, 1

)

= 〈0, 1, 1〉 .

Equation of the tangent plane 0(

x− π2

)

+ (y − 0) + (z − 1) = 0 i.e., y + z = 1

(d) Let f (x, y, z) = x2 + y2 − xyz − 7, then ∇f (x, y, z) = 〈2x− yz, 2y − xz,−xy〉 and

∇f (−2, 3,−1) = 〈−1, 4, 6〉 .

Equation of the tangent plane (−1) (x + 2)+4 (y − 3)+6 (z + 1) = 0 i.e., −x+4y +

6z = 8

(46) Equation of the cone is x2 + y2 − z2 = 0. At P = (x0, y0, z0), the normal vector to the

cone is given by n = ∇(

x2 + y2 − z2)

at P= 2x0 i + 2y0 j− 2z0 k.

(a) Equation of the tangent plane at P is n • (r− r0) = 0, or x0(x− x0) + y0(y − y0) +

z0(z− z0) = 0. Since x20 + y2

0 − z20 = 0, the equation is satisfied when x = y = z = 0.

Therefore, the tangent plane passes through the origin.

69

Page 72: AMA SolutionsManual

CHAPTER 4. PARTIAL DIFFERENTIATION

(b) Equation of the normal line is r− r0 = tn, where t is any real parameter. This

vector equation also takes the form x = x0 + x0t, y = y0 + y0t, z = z0 + z0t. When

x = y = 0, we have t = −1. This implies z = 2z0. Hence the normal line intersects

the z−axis at (0, 0, 2z0).

(47) The normal vector to the ellipsoid at (1, 1, 3) is given by

n = ∇(

x2 + 2y2 + 3z2)

at (1,1,3)= 2i + 4 j+18k.

As such, equation of the tangent plane is 2(x−1)+4(y−1)+18(z−3) = 0, or x+2y+9z =

30.

(48) Tangent vector =(

drdt

)

at t= 14

=(

i + 12√

tj)

at t= 14

= i + j. When t = 14 , x = 1

4 and y = 12 .

Vector equation of the normal line is given by T • (r− r0) = 0, where T = i + j is the

tangent vector, r =xi + yj and r0=14 i + 1

2 j.

(49) (a) By (45) b., ∇f (−2, 3,−1) = 〈12, 27, 3〉, then the normal line to the surface at

(−2, 3,−1) is x+212 = y−3

27 = z+13 or x = −2 + 12t, y = 3 + 27t and z = −1 + 3t.

(b) By (45) c., ∇f(

π2 , 0, 1

)

= 〈0, 1, 1〉, then the normal line to the surface at(

π2 , 0, 1

)

isx−π/2

0 = y1 = z−1

1 or x = π2 , y = t and z = 1 + t.

(c) By (45) d., ∇f (−2, 3,−1) = 〈−1, 4, 6〉, then the normal line to the surface at

(−2, 3,−1) is x+2−1 = y−3

4 = z+16 or x = −2− t, y = 3 + 4t and z = −1 + 6t.

(50) (a) r =⟨

t2, 3t, 0⟩

, ∇r (x, y, z) = 〈2t, 3, 0〉 and ∇r (1, 3, 0) = 〈2, 3, 0〉.x = 1 + 2s, y = 3 + 3s and z = 0.

(b) r =⟨

2t2, 3t, t3⟩

, ∇r (x, y, z) =⟨

4t, 3, 3t2⟩

and ∇r (2, 3, 3) = 〈4, 3, 3〉.x = 4 + 4s, y = 3 + 3s and z = 3 + 3s.

(c) r = 〈cos t, sin t, t〉, ∇r (x, y, z) = 〈− sin t, cos t, 1〉 and ∇r(

12 ,

√3

2 , π3

)

=⟨

−√

32 , 1

2 , 1⟩

.

x = 12 −

√3

2 s, y =√

32 + 1

2s and z = π3 + s.

(51) (a) r =⟨

t,−1, e2t⟩

, ∇r (x, y, z) =⟨

1, 0, 2e2t⟩

and ‖v‖ = ‖∇r (x, y, z)‖ =√

1 + 4e4t.

(b) r =⟨√

92 t2, 3t, t3

, ∇r (x, y, z) =⟨√

18t, 3, 3t2⟩

and ‖v‖ =√

18t2 + 9 + 9t4 =

3(

t2 + 1)

.

(52) Arc length of a curve is given by the integral

∫ β

α

(

dx

dt

)2

+

(

dy

dt

)2

+

(

dz

dt

)2

dt.

70

Page 73: AMA SolutionsManual

CHAPTER 4. PARTIAL DIFFERENTIATION

(a) arc length =∫ π0

(− sin t)2 + (1 + cos t)2dt =∫ π0

√2 + 2 cos tdt = 4.

(b) arc length =∫ 40

(t)2 +(√

2t + 1)2

dt =∫ 40

√t2 + 2t + 1dt =

∫ 40 (t + 1)dt = 12.

(c) arc length =∫ 2π0

(−a sin t)2 + (a cos t)2 + k2dt = 2π√

a2 + k2.

(53) (a) ∇f (x, y) =⟨

2x, 6y2⟩

, ∇f (2,−1) = 〈4, 6〉 and b‖b‖ = 1√

5〈1,−2〉 =

1√5,− 2√

5

.

∇f (2,−1) • b‖b‖ = 〈4, 6〉 •

1√5,− 2√

5

= −8√5

(b) ∇f (x, y) = 〈ey cos x, ey sinx〉, ∇f(

π3 , 0)

=⟨

12 ,

√3

2

and b‖b‖ =

45 ,−3

5

.

∇f(

π3 , 0)

• b‖b‖ =

12 ,

√3

2

•⟨

45 ,−3

5

= 4−3√

310

(c) ∇f (x, y, z) = 〈yzexy, xzexy, exy〉, ∇f (2, 1,−1) =⟨

−e2,−2e2, e2⟩

and

b‖b‖ =

1√3, 1√

3, 1√

3

.

∇f (2, 1,−1) • b‖b‖ =

−e2,−2e2, e2⟩

•⟨

1√3, 1√

3, 1√

3

= − 2√3e2

(54) (a) ∇T (x, y) = 〈ex cos y − ey sinx,−ex sin y + ey cos x〉 and ∇T (0, 0) = i + j, then

‖∇T (x, y)‖ =√

(1)2 + (1)2 =√

2.

(b) −∇T (0, 0) = −i− j

(55) ∂T∂x = 2x and ∂T

∂y = −2y.

dxdt = ∂T

∂x = 2xdydt = ∂T

∂y = −2y=⇒

x (t) = C1e2t

y (t) = C2e−2t

With initial condition (−2, 1), x (t) = −2e2t and y (t) = e−2t.

(56) ∇f(1,−1, 0) = 2i + 2j− k. Therefore

Duf(1,−1, 0) = ∇f(1,−1, 0) • u

‖u‖ = (2i + 2j− k) • 2i− 3j + 6k√

22 + (−3)2 + 62= −8

7.

The direction along which f increases most rapidly at the point (1,−1, 0) is 2i + 2j− k,

and the direction along which f decreases most rapidly is −2i − 2j + k. The rate of

increase/decrease is given by

∇f(1,−1, 0) • ± (2i + 2j− k)

‖± (2i + 2j− k)‖ = ±3.

(57) To minimize f(x, y, z) = (x− 1)2 + y2 + (z + 1)2 subject to g = 2x− y + 5z − 3 = 0, we

71

Page 74: AMA SolutionsManual

CHAPTER 4. PARTIAL DIFFERENTIATION

use Lagrange Multiplier Method:

fx = λgx

fy = λgy

fz = λgz

g = 0

2(x− 1) = 2λ

2y = −λ

2(z + 1) = 5λ

2x− y + 5z = 3

From the first 3 equations, we conclude that x = λ + 1, y = −λ2 and

z = −1 + 5λ2 . Substitution into the last equation yields λ = 2

5 , which implies that the

minimum point is (75 ,−1

5 , 0). Hence the minimum distance is equal to

(7

5− 1)2 +

(

−1

5

)2

+ (0 + 1)2 =

6

5.

(58) Label the pentagon as ABCDE where ABE is an isoceles triangle with AB = AE = x,

BE = 2y and BC = ED = z. We wish to minimize P = 2x + 2y + 2z, subject to the

constraint g = y√

x2 − y2 + 2yz − 1000 = 0. Lagrange Multiplier Method yields the

equations

2 = λxy√x2−y2

2 = λ

(

x2 − y2 − y2√x2−y2

+ 2z

)

2 = λ2y

y√

x2 − y2 + 2yz = 1000

.

The third equation implies that λ = 1y . Substitution into the first and the second

equation yields y =√

3x2 , z =

(√

3+1)x

2 and√

x2 − y2 = x2 . These, together with the

fourth equation, give

x =

4000

6 + 3√

3, y =

1000

2 +√

3, z =

(

1 +√

3)

1000

6 + 3√

3.

(59) We minimize the function f(x, y) = (x − a)2 + y2, subject to the constraint g(x, y) =

y2 − 4bx = 0, where a > 0, b > 0.

Lagrange Multiplier method leads to

2(x− a) = λ (−4b)

2y = λ2y

y2 − 4bx = 0

.

There are two cases:

y = 0: The third equation yields x = 0 as well. Therefore, (0, 0) is the point on the

72

Page 75: AMA SolutionsManual

CHAPTER 4. PARTIAL DIFFERENTIATION

parabola which minimizes f(x, y). As such, the shortest distance from (a, 0) to the

parabola is given by√

(0− a)2 + 02 = a.

y 6= 0: The second equation implies λ = 1. Substituting λ = 1 into the first equation,

we obtain x = a − 2b. Therefore y = ±√

4b(a− 2b). To summarize, there are

two points on the parabola which minimize f(x, y), i.e., (a − 2b,√

4b(a− 2b)) and

(a − 2b, −√

4b(a− 2b)). The shortest distance from (a, 0) to the parabola is given

by 4b(a− b).

Note: You may prove that case (b) occurs if and only if a > 2b by considering the

intersection of the circle (x− a)2 + y2 = a2 and the parabola y2 = 4bx.

(60) We apply Lagrange Multiplier Method to the function T = 2x2 + 5y2 + 11z2 + 20xy −4xz + 16yz, subject to g = x2 + y2 + z2 − 1 = 0, to obtain the equations

4x + 20y − 4z = λ2x

20x + 10y + 16z = λ2y

−4x + 16y + 22z = λ2z

x2 + y2 + z2 = 1

The first three equations are equivalent to an eigenvalue problem

4 20 −4

20 10 16

−4 16 22

x

y

z

= λ

x

y

z

.

Solving the characteristic equation

4− λ 20 −4

20 10− λ 16

−4 16 22− λ

= 0, one obtains eigenvalues

λ = 9, −9 and 18. Corresponding eigenvectors are obtained as follows: when λ = −9,

x

y

z

=

2t

−2t

t

; when λ = 9,

x

y

z

=

2s

s

−2s

; when λ = 18,

x

y

z

=

r

2r

2r

. We

then substitute these solutions into the constraint and obtain t = ±13 , s = ±1

3 , r = ±13 .

Therefore, the critical points of the function T on the sphere are (23 , −2

3 , 13), (−2

3 , 23 , −1

3),

(23 , 1

3 , −23), (−2

3 , −13 , 2

3), (13 , 2

3 , 23) and (−1

3 , −23 , −2

3). Substituting these points into T ,

we conclude that (13 , 2

3 , 23) and (−1

3 , −23 , −2

3) are the hottest points on the sphere, with

maximum temperature = 18.

(61) We minimize H = R1i21 + R2i

22 + R3i

23 + R4i

24, subject to the constraint g = i1 + i2 + i3 +

i4 − I = 0. Lagrange Multiplier Method gives 2Rkik = λ for k = 1, 2, 3, 4, from which it

73

Page 76: AMA SolutionsManual

CHAPTER 4. PARTIAL DIFFERENTIATION

follows that ik = λ2Rk

for each k. Substitution into the constraint gives

λ

2=

I1

R1+ 1

R2+ 1

R3+ 1

R4

,

from which it follows that ik = RRk× I for k = 1, 2, 3, 4, where 1

R = 1R1

+ 1R2

+ 1R3

+ 1R4

.

(62) We maximize A = 4xy, subject to the constraint g = 9x2 + 4y2 − 36 = 0. Lagrange

Multiplier Method yields

4y = λ18x

4x = λ8y

9x2 + 4y2 = 36

. We may eliminate λ from the first

two equations to obtain 4y2 = 9x2. Substitution into the third equation ⇒ x =√

2

and y = 3√2. As a result, the largest rectangle that can be inscribed in the ellipse

has vertices at (√

2, 3√2), (−

√2, 3√

2), (−

√2,− 3√

2) and (

√2,− 3√

2). Its area is given by

A = 4×√

2× 3√2

= 12.

(63) We wish to minimize S = 2πrh + 2πr2, subject to g = πr2h − 1000 = 0. Lagrange

Multiplier Method gives

2πh + 4πr = λ2πrh

2πr = λπr2

πr2h = 1000

. The first two equations together yield

h = 2r. Substituting this into the last equation, we conclude that r = 3

500π ≃ 5.42 and

h = 2× 3

500π ≃ 10.84.

(64) Tx = 2x− 1, Ty = 4y, Txx = 2, Tyy = 4 and Txy = Tyx = 0.

Tx = 0 =⇒ x = 12 Ty = 0 =⇒ y = 0

H = fxx

(

12 , 0)

fyy

(

12 , 0)

−[

fxy

(

12 , 0)]2

= 8 > 0 A = fxx = 2 > 0

Thus, f (x, y) has a local minimum (coldest point) at(

12 , 0)

and f(

12 , 0)

= −14 .

It is evident that the maximum occurs at the boundary. Then we have to use the

Lagrange Multipliers method to find the maximum point.

Let f (x, y) = x2 − x + 2y2 and g (x, y) = x2 + y2 − 1.

2x− 1 = 2λx

4y = 2λy

x2 + y2 = 1

=⇒ (x, y, λ) =(

±1, 0, 12

)

or(

−12 ,±

√3

2 , 2)

For (±1, 0), f (1, 0) = 0 and f (−1, 0) = 2.

For(

−12 ,±

√3

2

)

, f(

−12 ,±

√3

2

)

= 2.25.

Thus, the maximum (hottest point) is at(

−12 ,±

√3

2

)

and f(

−12 ,±

√3

2

)

= 2.25.

74

Page 77: AMA SolutionsManual

CHAPTER 4. PARTIAL DIFFERENTIATION

(65) Let f (A, B, C) =∑4

i=1 (Axi + Byi + C − zi)2, then fA = 2

∑4i=1 (Axi + Byi + C − zi) xi,

fB = 2∑4

i=1 (Axi + Byi + C − zi) yi and fC = 2∑4

i=1 (Axi + Byi + C − zi).

fA = 0 =⇒ 2∑4

i=1 (Axi + Byi + C − zi)xi = 0 =⇒ 2A + B + 2C = 0

fB = 0 =⇒ 2∑4

i=1 (Axi + Byi + C − zi) yi = 0 =⇒ A + 2B + 2C + 2 = 0

fC = 0 =⇒ 2∑4

i=1 (Axi + Byi + C − zi) = 0 =⇒ 2A + 2B + 4C + 1 = 0

Solving the system of linear equations, A = 12 , B = −3

2 and C = 14 .

Thus, the ”fitted” plane is z = 12x− 3

2y − 14 .

(66) fx = −2x + 2, fy = −2y + 2, fxx = −2, fyy = −2 and fxy = fyx = 0.

fx = 0

fx = 0=⇒

x = 1

y = 1

H = fxx (1, 1) fyy (1, 1)− [fxy (1, 1)]2 = 4 > 0 A = fxx = −2 < 0

Thus, f (x, y) has a maxima at (1, 1) and f (1, 1) = 4.

(67) Let f (x, y, z) = (x− 1)2 + (y − 1)2 + (z + 2)2 and g (x, y, z) = x − 2y + 5z − 4, then

fx = 2 (x− 1), fy = 2 (y − 1), fz = 2 (z + 2), gx = 1, gy = −2 and gz = 5.

fx = λgx

fy = λgy

fz = λgz

x− 2y + 5z = 4

=⇒

2 (x− 1) = λ

2 (y − 1) = −2λ

2 (z + 2) = 5λ

x− 2y + 5z = 4

=⇒

x = 12λ + 1

y = −λ + 1

z = 52λ− 2

x− 2y + 5z = 4

=⇒(

12λ + 1

)

− 2 (−λ + 1) + 5(

52λ− 2

)

= 15λ− 11 = 4 =⇒ λ = 1

Thus, x = 32 , y = 0, z = 1

2 and f (x, y, z) = 152 . The minimum distance is

√302 .

(68) Let f (x, y, z) = x2 + y2 + z2 and g (x, y, z) = x2 + y2 − z2 − 1, then fx = 2x, fy = 2y,

fz = 2z, gx = 2x, gy = 2y and gz = −2z.

fx = λgx

fy = λgy

fz = λgz

x2 + y2 − z2 = 1

=⇒

2x = 2λx

2y = 2λy

2z = −2λz

x2 + y2 − z2 = 1

=⇒

x (λ− 1) = 0

y (λ− 1) = 0

z (λ + 1) = 0

x2 + y2 − z2 = 1

While λ 6= 0, z = 0 and x2 + y2 = 1. If x = 0, then y = ±1. If y = 0, then x = ±1.

Thus, f (0,±1, 0) = 1 and f (±1, 0, 0) = 1. The minimum distance is 1.

(69) Let f (x, y, z) = xyz and g (x, y, z) = x + y + z2 − 16, then fx = yz, fy = xz, fz = xy,

gx = 1, gy = 1 and gz = 2z.

75

Page 78: AMA SolutionsManual

CHAPTER 4. PARTIAL DIFFERENTIATION

fx = λgx

fy = λgy

fz = λgz

x + y + z2 = 16

=⇒

yz = λ ... (1)

xz = λ ... (2)

xy = 2λz ... (3)

x + y + z2 = 16 ... (4)

From (1) and (2), yz = xz =⇒ z (y − x) = 0 =⇒ y = x as z 6= 0.

Again, from (1) and (2), x = y = λz ... (5).

From (3) and (5),(

λz

) (

λz

)

= 2λz =⇒ λ = 2z3 ... (6) as λ 6= 0.

For (4) ,(

λz

)

+(

λz

)

+ z2 = 16 =⇒ 2λz + z2 = 16 =⇒ 4z2 + z2 = 16 =⇒ z = 4√

5.

So, λ = 1285√

5and x = y = 128

5√

5/ 4√

5= 32

5 .

Thus, f (x, y, z) =(

325

) (

325

)

(

4√5

)

= 4096125

√5.

(70) Let f (x, y, z) = x − 2y + 5z and g (x, y, z) = x2 + y2 + z2 − 36, then fx = 1, fy = −2,

fz = 5, gx = 2x, gy = 2y and gz = 2z.

1 = 2λx

−2 = 2yλ

5 = 2λz

x2 + y2 + z2 = 36

=⇒ (x, y, z, λ) =

(√

65 ,−2

65 ,√

30,√

524

)

(

−√

65 , 2√

65 ,−√

30,√

524

)

f(√

65 ,−2

65 ,√

30)

=(√

65

)

− 2(

−2√

65

)

+ 5(√

30)

= 6√

30 = 32.863

f(

−√

65 , 2√

65 ,−√

30)

= −6√

30 = −32.863

Thus, the maximum value of f is 32.863 and the minimum value of f is −32.863.

(71) Let x, y and z be respectively the length, the width and the height of the box. We wish

to minimize f(x, y, z) = 2xy + 8[xy + 2xz + 2yz], subject to the constraint g(x, y, z) =

xyz − 36 = 0. Using the Lagrange Multiplier method, we solve the equations

10y + 16z = λyz

10x + 16z = λxz

16x + 16y = λxy

xyz = 36

to obtain x = y = 3

2885 ≃ 3.862 and z = 10

16x = 58

3

2885 ≃ 2.414.

Physical consideration indicates that this must be a minimum. Thus the cost for building

such a box is approximately $447.48.

76

Page 79: AMA SolutionsManual

CHAPTER 4. PARTIAL DIFFERENTIATION

(72) We wish to minimize f(x, y) = x2 +(y +2)2, subject to the constraint g(x, y) = x2−2x+

y2 = 0. Using the Lagrange Multiplier method, we simplify the equations

2x = λ(2x− 2)

2(y + 2) = λ2y

x2 − 2x + y2 = 0

to conclude that y = 2x− 2 and 5x2 − 10x + 4 = 0.

Thus x = 1 − 1√5, y = − 2√

5or x = 1 + 1√

5, y = 2√

5. For mininum, we have x = 1 − 1√

5,

y = − 2√5.

The shortest distance is given by

(

1− 1√5

)2+(

− 2√5

+ 2)2

=√

5− 1 ≃ 1.236.

(73) Using the Lagrange Multiplier method, we solve the equations

1 = 2λx

3 = 2λy

5 = 2λz

x2 + y2 + z2 = 1

to obtain two critical points (x, y, z, λ) = ±(

1√35

, 3√35

, 5√35

, 2√35

)

. Thus the maximum

value is

f

(

1√35

,3√35

,5√35

)

=√

35

and the minimum value is

f

(

− 1√35

,− 3√35

,− 5√35

)

= −√

35.

(74) We minimize f(x, y, z) = (x + 1)2 + (y − 2)2 + (z − 3)2, subject to

g(x, y, z) = x + 2y − 3z − 4 = 0

h(x, y, z) = 2x− y + 2z − 5 = 0.

77

Page 80: AMA SolutionsManual

CHAPTER 4. PARTIAL DIFFERENTIATION

By the Lagrange Multiplier method, we solve the equations

2(x + 1) = λ + 2µ

2(y − 2) = 2λ− µ

2(z − 3) = −3λ + 2µ

x + 2y − 3z = 4

2x− y + 2z = 5

to obtain x = 3715 , y = 49

15 , z = 2515 . Physical consideration indicates that this must be a

minimum. Therefore, the shortest distance is given by

(

37

15+ 1

)2

+

(

49

15− 2

)2

+

(

25

15− 3

)2

≃ 3.9243.

(75) Let f (x, y, z) = x2 + y2 + z2 and g (x, y, z) = xyz − 1, then fx = 2x, fy = 2y, fz = 2z,

gx = yz, gy = xz and gz = xy.

2x = yz

2y = xz

2z = xy

xyz = 1

=⇒ (x, y, z) =

(1, 1, 1)

(−1,−1, 1)

(−1, 1,−1)

(1,−1,−1)

f (1, 1, 1) = f (−1,−1, 1) = f (−1, 1,−1) = f (1,−1,−1) = 3

Therefore, the minimum distance is√

3 and the maximum distance is ∞.

(76) We differentiate the equation x2 − 2yw + w3 + 1 = 0 with respect to x to obtain

2x− 2y∂w

∂x+ 3w2 ∂w

∂x= 0, (4.0.1)

which implies ∂w∂x = −2x

3w2−2y. If we now differentiate (4.0.1) with respect to x, we obtain

2− 2y∂2w

∂x2+ 6w

(

∂w

∂x

)2

+ 3w2 ∂2w

∂x2= 0.

From this equation, one concludes that

∂2w

∂x2= −2 + 6w

(

∂w∂x

)2

3w2 − 2y.

(77) Let F (x, y, z) = x2 + y2 − z2 − 2x (y + z) = 0. Then Fx = 2x − 2y − 2z, Fy = 2y − 2x

and Fz = −2z − 2x.

78

Page 81: AMA SolutionsManual

CHAPTER 4. PARTIAL DIFFERENTIATION

Thus, ∂z∂x = −Fx

Fz= x−y−z

x+z and ∂z∂y = −Fy

Fz= y−x

x+z .

∂2z

∂y2=

(x + z) (1)− (y − x)∂z∂y

(x + z)2=

(x + z)− (y − x)y−xx+z

(x + z)2

=(x + z)2 − (y − x)2

(x + z)3=

(y + z) (2x− y + z)

(x + z)3.

∂2z

∂y∂x=

∂y

(

∂z

∂x

)

=(x + z)

(

−1− ∂z∂y

)

− (x− y − z) ∂z∂y

(x + z)2

=(x + z)

(

−1− y−xx+z

)

− (x− y − z) y−xx+z

(x + z)2

=x2 − 3xy − 2xz + y2 − z2

(x + z)3.

∂2z

∂x∂y=

∂x

(

∂z

∂y

)

=(x + z) (−1)− (y − x)

(

1 + ∂z∂x

)

(x + z)2

=(x + z) (−1)− (y − x)

(

1 + x−y−zx+z

)

(x + z)2

=(x + z)2 (−1)− (y − x) (2x− y)

(x + z)3

=x2 − 3xy − 2xz + y2 − z2

(x + z)3.

(78) Let F (x, y, w) = x + y + xyw −w3, then Fx = 1 + yw, Fy = 1 + xw and Fw = xy − 3w2.

∂w∂x = − Fx

Fw= − 1+yw

xy−3w2 and ∂w∂y = − Fy

Fw= − 1+xw

xy−3w2

∂2w

∂x2=

∂x

(

1 + yw

3w2 − xy

)

=

(

3w2 − xy)

∂∂x (1 + yw)− (1 + yw) ∂

∂x

(

3w2 − xy)

(3w2 − xy)2

=

(

3w2 − xy) (

y ∂w∂x

)

− (1 + yw)(

6w ∂w∂x − y

)

(3w2 − xy)2

=

(

3w2 − xy)

(

y 1+yw3w2−xy

)

− (1 + yw)(

6w 1+yw3w2−xy

− y)

(3w2 − xy)2

=−2(

xy2 + 3w)

(wy + 1)

(3w2 − xy)3

79

Page 82: AMA SolutionsManual

CHAPTER 4. PARTIAL DIFFERENTIATION

∂2w

∂y2=

∂y

(

1 + xw

3w2 − xy

)

=

(

3w2 − xy)

∂∂y (1 + xw)− (1 + xw) ∂

∂y

(

3w2 − xy)

(3w2 − xy)2

=

(

3w2 − xy)

(

x∂w∂y

)

− (1 + xw)(

6w ∂w∂y − x

)

(3w2 − xy)2

=

(

3w2 − xy)

(

x 1+xw3w2−xy

)

− (1 + xw)(

6w 1+xw3w2−xy

− x)

(3w2 − xy)2

=−2(

x2y + 3w)

(wx + 1)

(3w2 − xy)3

∂2w

∂x∂y=

∂x

(

1 + xw

3w2 − xy

)

=

(

3w2 − xy)

∂∂x (1 + xw)− (1 + xw) ∂

∂x

(

3w2 − xy)

(3w2 − xy)2

=

(

3w2 − xy) (

w + x∂w∂x

)

− (1 + xw)(

6w ∂w∂x − y

)

(3w2 − xy)2

=

(

3w2 − xy)

(

w + x 1+yw3w2−xy

)

− (1 + xw)(

6w 1+yw3w2−xy

− y)

(3w2 − xy)2

=w(

3w2 − xy)2

+ x(

3w2 − xy)

(1 + yw)− 6w (1 + yw)2 − y(

3w2 − xy)

(1 + xw)

(3w2 − xy)3

∂2w

∂y∂x=

∂y

(

1 + yw

3w2 − xy

)

=

(

3w2 − xy)

∂∂y (1 + yw)− (1 + yw) ∂

∂y

(

3w2 − xy)

(3w2 − xy)2

=

(

3w2 − xy)

(

w + y ∂w∂y

)

− (1 + yw)(

6w ∂w∂y − x

)

(3w2 − xy)2

=

(

3w2 − xy)

(

w + y 1+xw3w2−xy

)

− (1 + yw)(

6w 1+xw3w2−xy

− x)

(3w2 − xy)2

=w(

3w2 − xy)2

+ x(

3w2 − xy)

(1 + yw)− 6w (1 + yw)2 − y(

3w2 − xy)

(1 + xw)

(3w2 − xy)3

(79) Differentiating the equation with respect to x and y, we obtain

3w2 ∂w

∂x+ 4

∂w

∂x− 2x + y2 = 0 and 3w2 ∂w

∂y+ 4

∂w

∂y+ 2xy + 8 = 0.

Therefore, ∂w∂x = 2x−y2

3w2+4and ∂w

∂y = −2xy−83w2+4

. Solving

2x− y2 = 0

−2xy − 8 = 0, one obtains

80

Page 83: AMA SolutionsManual

CHAPTER 4. PARTIAL DIFFERENTIATION

(2,−2) as the only critical point of the function. At (2,−2), further differentiations yield

∂2w

∂x2=

2(

3w2 + 4)

−(

2x− y2)

(6w ∂w∂x )

(3w2 + 4)2,

∂2w

∂y2=

(−2x)(

3w2 + 4)

+ (2xy + 8) (6w ∂w∂x )

(3w2 + 4)2,

∂2w

∂x∂y=−2y

(

3w2 + 4)

−(

2x− y2)

(6w ∂w∂y )

(3w2 + 4)2.

Since ∂w∂x = 0 at (2,−2), conclude that ∂2w

∂x2 > 0 and ∂2w∂y2 < 0 at that point. Therefore,

∂2w∂x2

∂2w∂y2 −

(

∂2w∂x∂y

)2< 0 at (2,−2) and therefore the point (2,−2) is a saddle point.

(80) Differentiating the equation with respect to x and y, we obtain

1 + yw + xy∂w

∂x− ew ∂w

∂x= 0 and 1 + xw + xy

∂w

∂y− ew ∂w

∂y= 0.

This imply ∂w∂x = 1+yw

ew−xy and ∂w∂y = 1+xw

ew−xy . Further differentiation gives

∂2w

∂x2=

(ew − xy) y ∂w∂x − (1 + yw)

(

ew ∂w∂x − y

)

(ew − xy)2,

∂2w

∂y2=

x (ew − xy) ∂w∂y − (1 + xw)

(

ew ∂w∂y − x

)

(ew − xy)2,

∂2w

∂x∂y=

(ew − xy)(

w + y ∂w∂y

)

− (1 + yw)(

ew ∂w∂y − x

)

(ew − xy)2.

(81) Differentiating the equations with respective to x and y, we obtain

2u∂u∂x −4v ∂v

∂x = −2x− 5y

(v cos uv + 1− 6v)∂u∂x +(u cos uv − 3− 6u) ∂v

∂x = −ex cos y

2u∂u∂y −4v ∂v

∂y = 6y − 5x

(v cos uv + 1− 6v)∂u∂y +(u cos uv − 3− 6u)∂v

∂y = ex sin y.

We may now use Cramer’s rule to obtain ∂u∂x , ∂v

∂x , ∂u∂y and ∂v

∂y . For instance, we have

∂u

∂x=

1

D

−2x− 5y −4v

−ex cos y u cos uv − 3− 6u

81

Page 84: AMA SolutionsManual

CHAPTER 4. PARTIAL DIFFERENTIATION

and∂v

∂x=

1

D

2u −2x− 5y

v cos uv + 1− 6v −ex cos y

,

where

D =

2u −4v

v cos uv + 1− 6v u cos uv − 3− 6u

.

(82) (a) If T and Φ are defined as functions of P and V by the equations, we differentiate

the equations to obtain

∂T

∂P=

V

R,

∂Φ

∂P=

CV

P,

∂T

∂V=

P

Rand

∂Φ

∂V=

CP

V.

Therefore,∂T

∂P

∂Φ

∂V− ∂T

∂V

∂Φ

∂P=

CP − CV

R= 1.

(b) If P and V are defined as functions of T and Φ by the equations, we differentiate

the equations with respect to T and Φ to obtain

V ∂P∂T +P ∂V

∂T = RCVP

∂P∂T +CP

V∂V∂T = 0

and

V ∂P∂Φ +P ∂V

∂Φ = 0CVP

∂P∂Φ +CP

V∂V∂Φ = 1

.

Cramer’s rule then yields

∂P

∂T=

CP

V,

∂V

∂T= −CV

P,

∂P

∂Φ= −P

Rand

∂V

∂Φ=

V

R.

Therefore,∂P

∂T

∂V

∂Φ− ∂V

∂T

∂P

∂Φ= 1.

82

Page 85: AMA SolutionsManual

Chapter 5

Multiple Integrals

(1) (a)∫ 3

0

∫ 2

04xydxdy =

∫ 3

04y

[

x2

2

]2

0

dy =

∫ 3

08ydy =

[

4y2]3

0= 36

(b)∫ 4

2

∫ 3

14xydxdy =

∫ 4

24y

[

x2

2

]3

1

dy =

∫ 4

216ydy =

[

8y2]4

2= 96

(c)∫ 1

0

∫ 1

y4xydxdy =

∫ 1

04y

[

x2

2

]1

y

dy =

∫ 1

0

(

2y − 2y3)

dy =

[

y2 − 1

2y4

]1

0

=1

2

(d)

∫ 2

0

∫ x+1

x/24xydydx =

∫ 2

04x

[

y2

2

]x+1

x/2

dx =

∫ 2

04x

[

(x + 1)2

2− x2

8

]

dx

=1

2

∫ 2

0

(

3x3 + 8x2 + 4x)

dx =1

2

[

3

4x4 +

8

3x3 + 2x2

]2

0

=62

3

(e)

∫ 1

0

∫ 3√

y

y2

4xydxdy =

∫ 1

04y

[

x2

2

] 3√

y

y2

dy = 2

∫ 1

0

(

y5/3 − y5)

dy

= 2

[

3

8y8/3 − 1

6y6

]1

0

=5

12

(f)∫ 1

0

√y

04xydxdy =

∫ 1

04y

[

x2

2

]

√y

0

dy =

∫ 1

02y2dy =

[

2

3y3

]1

0

=2

3

83

Page 86: AMA SolutionsManual

CHAPTER 5. MULTIPLE INTEGRALS

(2)

y = x2

y = x3=⇒ (x, y) = (0, 0) or (1, 1)

∫ 1

0

∫ x2

x3

(

x4 + y2)

dydx =

∫ 1

0

[

x4y +1

3y3

]x2

x3

dx =

∫ 1

0

(

4

3x6 − x7 − 1

3x9

)

dx

=

[

4

21x7 − 1

8x8 − 1

30x10

]1

0

=9

280= 0.032143

(3)

x = y2

y = x=⇒ (x, y) = (0, 0) or (1, 1)

∫ 1

0

∫ y

y2

√xydxdy =

∫ 1

0

√y

[

2

3x3/2

]y

y2

dy =

∫ 1

0

2

3

(

y2 − y7/2)

dx

=2

3

[

1

3y3 − 2

9y9/2

]1

0

=2

27

(4) The parabola and the line intersect each other at (−1,−3) and (4, 12).

Hence,

Rx dA =

∫ 4

−1

[∫ 3x

x2−4x dy

]

dx =

∫ 4

−1x[

3x− x2 + 4]

dx

=

∫ 4

−1

[

−x3 + 3x2 + 4x]

dx =125

4.

(5) (a)∫∫

R2xydxdy = X + Y

X =

∫ 1

0

∫ x+2

02xydydx =

∫ 1

0x[

y2]x+2

0dx =

∫ 1

0

(

x3 + 4x2 + 4x)

dx =43

12

Y =

∫ 3

1

∫ 4−x

02xydydx =

∫ 3

1x[

y2]4−x

0dx =

∫ 3

1

(

x3 − 8x2 + 16x)

dx =44

3

Thus,∫ ∫

R2xydxdy =

43

12+

44

3=

73

4

(b)∫∫

R2xydxdy = X + Y

84

Page 87: AMA SolutionsManual

CHAPTER 5. MULTIPLE INTEGRALS

X =

∫ 2

1

∫ 6−2y

y2xydxdy =

∫ 2

1y[

x2]6−2y

ydy =

∫ 2

1

(

3y3 − 24y2 + 36y)

dy =37

4

Y =

∫ 1

0

∫ 4y

y2xydxdy =

∫ 1

0y[

x2]4y

ydy =

∫ 1

015y3dy =

15

4

Thus,∫ ∫

R2xydxdy =

37

4+

15

4= 13

(c)∫∫

R2xydxdy = X + Y

X =

∫ 3

1

∫ (9−y)/2

3(y−1)/22xydxdy =

∫ 3

1y[

x2](9−y)/2

3(y−1)/2dy =

1

4

∫ 3

1

(

72y − 8y3)

dy = 32

Y =

∫ 1

0

∫ 3y+1

1−y2xydxdy =

∫ 1

0y[

x2]3y+1

1−ydy =

∫ 1

0

(

8y3 + 8y2)

dy =14

3

Thus,∫ ∫

R2xydxdy = 32 +

14

3=

110

3

(d)∫∫

R2xydxdy = X + Y

X =

∫ 1

0

∫ x+2

2−2x2xydydx =

∫ 1

0x[

y2]x+2

2−2xdx =

∫ 1

0

(

12x2 − 3x3)

dx =13

4

Y =

∫ 3

1

∫ (7−x)/2

(x−1)/22xydydx =

∫ 3

1x[

y2](7−x)/2

(x−1)/2dx =

1

4

∫ 3

1

(

48x− 12x2)

dx = 22

Thus,∫∫

R2xydxdy =

13

4+ 22 =

101

4

(6)

y

x1

3

85

Page 88: AMA SolutionsManual

CHAPTER 5. MULTIPLE INTEGRALS

(a)∫ 3

0

∫ 1

0x2dxdy =

∫ 3

0

[

1

3x3

]1

0

dy =

[

1

3y

]1

0

= 1

(b)

∫ 3

0

∫ 1

0ex+ydxdy =

∫ 3

0

[

ex+y]1

0dy =

∫ 3

0

(

e1+y − ey)

dy

=[

e1+y − ey]3

0= e4 − e3 − e + 1

(c)∫ 3

0

∫ 1

0xy2dxdy =

∫ 3

0

[

1

2y2x2

]1

0

dy =

∫ 3

0

1

2y2dy =

[

1

6y3

]3

0

=9

2

(7)

y = 1

(a)

∫ 1

0

∫ 1

yx3ydxdy =

∫ 1

0

[

1

4x4y

]1

y

dy =

∫ 1

0

(

1

4y − 1

4y5

)

dy

=

[

1

8y2 − 1

24y6

]1

0

=1

12

(b)

∫ 1

0

∫ 1

yxy3dxdy =

∫ 1

0

[

1

2x2y3

]1

y

dy =

∫ 1

0

(

1

2y3 − 1

2y5

)

dy

=

[

1

8y4 − 1

12y6

]1

0

=1

24

(c)

∫ 1

0

∫ 1

yx2y2dxdy =

∫ 1

0

[

1

3x3y2

]1

y

dy =

∫ 1

0

(

1

3y2 − 1

3y5

)

dy

=

[

1

9y3 − 1

18y6

]1

0

=1

18

86

Page 89: AMA SolutionsManual

CHAPTER 5. MULTIPLE INTEGRALS

(8)

y

x

2

2

(a)

∫ π/2

0

∫ π/2

0sin (x + y) dxdy

=

∫ π/2

0[− cos (x + y)]

π/20 dy =

∫ π/2

0

(

cos y − cos(π

2+ y))

dy

=[

sin y − sin(π

2+ y)]π/2

0= 2

(b)

∫ π/2

0

∫ π/2

0cos (x + y) dxdy =

∫ π/2

0[sin (x + y)]

π/20 dy

=

∫ π/2

0

[

sin(π

2+ y)

− sin (y)]

dy =[

cos(π

2+ y)

− cos (y)]π/2

0= 0

(c)

∫ π/2

0

∫ π/2

0(1 + xy) dxdy =

∫ π/2

0

[

x +1

2x2y

]π/2

0

dy

=

∫ π/2

0

(

1

2π +

1

8π2y

)

dy =

[

1

2πy +

1

16π2y2

]π/2

0

=1

4π2 +

1

64π4

(9) R =

(x, y) | 3√

y ≤ x ≤ 2, 0 ≤ y ≤ 8

=

(x, y) |0 ≤ x ≤ 2, 0 ≤ y ≤ x3

∫ 8

0

(

∫ 2

3√

yex4

dx

)

dy =

∫ 2

0

(

∫ x3

0ex4

dy

)

dx =

∫ 2

0ex4

x3dx =1

4

(

e16 − 1)

(10) R =

(x, y) |0 ≤ x ≤ 1, 1 ≤ y ≤ x2

=

(x, y) |0 ≤ x ≤ √y, 0 ≤ y ≤ 1

∫ 1

0

(∫ 1

x2

sin y√y

dy

)

dx =

∫ 1

0

(

∫ +√

y

0

sin y√y

dx

)

dy =

∫ 1

0

sin y√y

√ydy = 1− cos 1 = 0.4597

87

Page 90: AMA SolutionsManual

CHAPTER 5. MULTIPLE INTEGRALS

(11) (a)

∫ 1

−1

√1−y2

−√

1−y2

(

x2 + 3y3)

dxdy =

∫ 1

−1

[

1

3x3 + 3y3x

]

√1−y2

−√

1−y2

dy

=2

3

∫ 1

−1

(1− y2)dy − 2

3

∫ 1

−1

(1− y2)y2dy + 6

∫ 1

−1y3√

(1− y2)dy

=2

3

∫ π/2

−π/2cos2 θdθ − 2

3

∫ π/2

−π/2sin2 θ cos2 θdθ + 6

∫ π/2

−π/2sin3 θ cos2 θdθ

=2

3

∫ π/2

−π/2

1 + cos 2θ

2dθ − 2

3

∫ π/2

−π/2

sin2 2θ

4dθ − 6

∫ π/2

−π/2

(

cos2 θ − cos4 θ)

d (cos θ)

=2

3

(

1

)

− 2

3

∫ π/2

−π/2

1− cos 4θ

8dθ − 6

[(

cos3 θ

3− cos5 θ

5

)]π/2

−π/2

=1

3π − 2

3

(

1

)

+ 0 =1

(b)

∫ 1

0

∫ y

y2

√xydxdy =

∫ 1

0

[

2

3x3/2y1/2

]y

y2

dy =

∫ 1

0

2

3

(

y2 − y7/2)

dy

=

[

2

9y3 − 4

27y9/2

]1

0

=2

27

(c)

∫ 1

0

∫ y2

0yexdxdy =

∫ 1

0[yex]y

2

0 dy =

∫ 1

0

(

yey2 − y)

dy

=

[

1

2ey2 − 1

2y2

]1

0

=1

2e− 1

(d)

88

Page 91: AMA SolutionsManual

CHAPTER 5. MULTIPLE INTEGRALS

1 4

-3

-2

-1

1

2

3

32

y2 = 2x

y2 = 8 - 2x

y

x

∫ 2

−2

∫ (8−y2)/2

y2/2

(

4− y2)

dxdy =

∫ 2

−2

[(

4− y2)

x](8−y)2/2

y2/2dy

=

∫ 2

−2

(

16− 8y2 + y4)

dy =

[

16y − 8

3y3 +

1

5y5

]2

−2

=512

15

(e)

∫ 1

0

∫ 3√

y

√y

(

x4 + y2)

dxdy =

∫ 1

0

[

1

5x5 + y2x

] 3√

y

√y

dy

=

∫ 1

0

(

1

5y5/3 + y7/3 − 6

5y5/2

)

dy =

[

3

40y8/3 +

3

10y10/3 − 12

35y7/2

]1

0

=9

280= 0.03214

(f)

y = x

y = -x

y

x

1.0

1.0

-1.0

-1.0

0.5

0.5-0.5

-0.5

∫ 1

0

∫ x

−x

(

3xy2 − y)

dydx +

∫ 0

−1

∫ −x

x

(

3xy2 − y)

dydx

=

∫ 1

0

[

3

2x2y2 − xy

]x

−x

dx =

∫ 1

02x4dx−

∫ 0

−12x4dx = 0

(g)

89

Page 92: AMA SolutionsManual

CHAPTER 5. MULTIPLE INTEGRALS

y

x

2y = x

20

∫ 2

0

∫ x/2

0ex2

dydx =

∫ 2

0

[

ex2

y]x/2

0dx =

∫ 2

0

1

2xex2

dx =

[

1

4ex2

]2

0

=1

4e4 − 1

4

(h)

1

0.5

-0.5

-1

0

0.5 1-0.5-1

y

x

∫ 1

0

∫ x3

x4

(x + y) dydx +

∫ 0

−1

∫ x4

x3

(x + y) dydx

=

∫ 1

0

[

1

2x2 + yx

]x3

x4

dx +

∫ 0

−1

[

1

2x2 + yx

]x4

x3

dx

=

∫ 1

0

(

x4 +1

2x6 − x5 − 1

2x8

)

dx +

∫ 0

−1

(

x5 +1

2x8 − x4 − 1

2x6

)

dx

=

[

1

5x5 +

1

14x7 − 1

6x6 − 1

18x9

]1

0

+

[

1

6x6 +

1

18x9 − 1

5x5 − 1

14x7

]0

−1

=31

630− 241

630= −1

3

(i)

y

x

2y = x

1

0

90

Page 93: AMA SolutionsManual

CHAPTER 5. MULTIPLE INTEGRALS

∫ 1

0

∫ 2y

0e−y2/2dxdy =

∫ 1

0

[

xe−y2/2]2y

0dy =

∫ 1

02ye−y2/2dy

=[

−2e−12y2]1

0= 2− 2e−

12

(12)

Boundaries: u = 0, u = 1, v = 0, v = 1

Jacobian:

∂ (x, y)

∂ (u, v)=

∂x∂u

∂y∂u

∂x∂v

∂y∂v

=

∂(u−uv)∂u

∂(uv)∂u

∂(u−uv)∂v

∂(uv)∂v

=

1− v v

−u u

= u

∫ 1

0

∫ 1−x

0ey/(x+y)dydx =

∫ 1

0

∫ 1

0e

uvu

∂ (x, y)

∂ (u, v)dudv

=

∫ 1

0

∫ 1

0uevdvdu =

∫ 1

0[uev]10 du =

∫ 1

0(ue− u) du

=

[(

u2e

2− u2

2

)]1

0

=e− 1

2

(13)∫ 10 f (x) dx = A =⇒

∫ 10 f (y) dy = A

∫ 10

∫ 10 f (x) f (y) dxdy =

∫ 10 f (y)

[

∫ 10 f (x) dx

]

dy =∫ 10 Af (y) dy = A

∫ 10 f (y) dy = A2

∫ 10

∫ x0 f (x) f (y) dydx +

∫ 10

∫ 1x f (x) f (y) dydx =

∫ 10

∫ 10 f (x) f (y) dxdy = A2

Let A = (x, y) |0 ≤ x ≤ 1, 0 ≤ y ≤ x and B = (x, y) |0 ≤ x ≤ 1, x ≤ y ≤ 1.

Consider the following transformation that transforms the space in A to B,

v (x, y) = x

u (x, y) = y.

Then J =

∂u∂x

∂v∂x

∂u∂y

∂v∂y

=

0 1

1 0

= −1.

91

Page 94: AMA SolutionsManual

CHAPTER 5. MULTIPLE INTEGRALS

∫ 1

0

∫ x

0f (x) f (y) dydx =

∫ 1

0

∫ 1

uf (u) f (v) dvdu =

∫ 1

0

∫ 1

xf (x) f (y) dydx

Thus,∫ 10

∫ x0 f (x) f (y) dydx +

∫ 10

∫ 1x f (x) f (y) dydx = 2

∫ 10

∫ 1x f (x) f (y) dydx = A2.

Therefore,∫ 10

∫ 1x f (x) f (y) dydx = A2

2 .

(14) Let u = xy and v = xy3, where 4 ≤ u ≤ 8 and 5 ≤ v ≤ 15.

Jacobian:

∂ (u, v)

∂ (x, y)=

∂u∂x

∂v∂x

∂u∂y

∂v∂y

=

∂(xy)∂x

∂(xy3)∂x

∂(xy)∂y

∂(xy3)∂y

=

y y3

x 3xy2

= 2xy3 = 2v

∫∫

Ddxdy =

∫∫

D1

∂ (x, y)

∂ (u, v)dudv =

∫∫

D1

1∂(u,v)∂(x,y)

dudv =

∫ 8

4

∫ 15

5

1

2vdvdu

=

∫ 8

4

[

ln v

2

]15

5

du =

∫ 8

4

(

ln 15

2− ln 5

2

)

du =

∫ 8

4

(

1

2ln 3

)

du = 2 ln 3

(15) (a)

∫ ∫

Ω(x + y) dxdy

=

∫ π

0

∫ 2

0(r cos θ + r sin θ) rdrdθ =

∫ π

0(cos θ + sin θ) dθ

∫ 2

0r2dr

= [sin θ − cos θ]π0

[

1

3r3

]2

0

= (2)

(

8

3

)

=16

3

(b)

∫ ∫

Ω(x + y) dxdy

=

∫ π/2

−π/2

∫ 3

2(r cos θ + r sin θ) rdrdθ =

∫ π/2

−π/2(cos θ + sin θ) dθ

∫ 3

2r2dr

= [sin θ − cos θ]π/2−π/2

[

1

3r3

]3

2

= (2)

(

27− 8

3

)

=38

3

92

Page 95: AMA SolutionsManual

CHAPTER 5. MULTIPLE INTEGRALS

(c)

∫ ∫

Ω(x + y) dxdy

=

∫ π/2

−π/2

∫ 2 cos θ

0(r cos θ + r sin θ) rdrdθ

=

∫ π/2

−π/2(cos θ + sin θ)

∫ 2 cos θ

0r2drdθ =

∫ π/2

−π/2(cos θ + sin θ)

[

1

3r3

]2 cos θ

0

=8

3

∫ π/2

−π/2

(

cos4 θ + cos3 θ sin θ)

dθ =8

3

∫ π/2

−π/2cos4 θdθ +

8

3

∫ π/2

−π/2cos3 θ sin θdθ

=8

3(2)

∫ π/2

0cos4 θdθ +

8

3

∫ π/2

−π/2cos3 θ sin θdθ =

16

3

(

16

)

+8

3

[

−1

4cos4 θ

]π/2

−π/2

= π + 0 = π

(d)

∫ ∫

Ω(x + y) dxdy

=

∫ π/2

0

∫3√

θ

0(r cos θ + r sin θ) rdrdθ =

∫ π/2

0(cos θ + sin θ)

∫3√

θ

0r2drdθ

=

∫ π/2

0(cos θ + sin θ)

[

1

3r3

]3√

θ

0

dθ =1

3

∫ π/2

0θ (cos θ + sin θ) dθ3

=1

3[cos θ + θ sin θ + sin θ − θ cos θ]

π/20 =

π

6

(16) Distance from (0, 0) to(

1,√

3)

=

(1)2 +(√

3)2

= 2

sec θ = 2 =⇒ θ = π3

∫ ∫

R

x2 + y2dxdy

=

∫ π/3

0

∫ sec θ

0(r) rdrdθ =

∫ π/3

0

[

1

3r3

]sec θ

0

dθ =1

3

∫ π/3

0sec3 θdθ

=

√3

3+

ln(

2 +√

3)

6= 0.79684

(17) R =

(x, y) | − 2 ≤ x ≤ 4, x2−62 ≤ y ≤ x + 1

∫ ∫

RxydA =

∫ 4

−2

∫ x+1

x2−62

xydydx =1

8

∫ 4

−2(−x5 + 16x3 + 8x2 − 32x)dx = 36

93

Page 96: AMA SolutionsManual

CHAPTER 5. MULTIPLE INTEGRALS

(18) Let x = r cos θ and y = r sin θ. Jacobian J = ∂(x,y)∂(r,θ) = r.

R = (r, θ) |0 ≤ r ≤ 4 sin θ, 0 ≤ θ ≤ π∫ π

0

∫ 4 sin θ

0r2 sin θdrdθ =

64

3

∫ π

0sin4 θdθ = 8π

(19) Let x = r cos θ and y = r sin θ. Jacobian J = ∂(x,y)∂(r,θ) = r.

R =

(r, θ) |2 cos θ ≤ r ≤ 4 cos θ,−π2 ≤ θ ≤ π

2

∫ π2

−π2

∫ 4 cos θ

2 cos θr2 cos θdrdθ =

56

3

∫ π2

−π2

cos4 θdθ = 7π

(20) A sketch will convince you that

∫ 2

0

(

√4−y2

0

x2 + y2dx

)

dy =

R

x2 + y2dxdy,

where R is the region given by (x, y) : x ≥ 0, y ≥ 0 and x2 + y2 ≤ 4.

The above integral is therefore equal to

∫ π2

0

(∫ 2

0r × r dr

)

dθ =4π

3.

(21) (a)

∫∫

x2+y2≤1cos(

x2 + y2)

dxdy =

∫ 2π

0

∫ 1

0

(

cos r2)

rdrdθ

=

∫ 2π

0

[

1

2sin r2

]1

0

dθ =

∫ 2π

0

(

1

2sin 1

)

dθ = π sin 1

(b)

∫∫

1≤x2+y2≤4cos(

x2 + y2)

dxdy =

∫ 2π

0

∫ 2

1

(

cos r2)

rdrdθ

=

∫ 2π

0

[

1

2sin r2

]2

1

dθ =

∫ 2π

0

(

1

2sin 4− 1

2sin 1

)

dθ = π sin 4− π sin 1

94

Page 97: AMA SolutionsManual

CHAPTER 5. MULTIPLE INTEGRALS

(22) (a)

∫∫

x2+y2≤1sin(

x2 + y2)

dxdy =

∫ 2π

0

∫ 1

0(sin r) rdrdθ

=

∫ 2π

0[sin r − r cos r]10 dθ =

∫ 2π

0(sin 1− cos 1) dθ = 2π sin 1− 2π cos 1

(b)

∫∫

1≤x2+y2≤4sin(

x2 + y2)

dxdy =

∫ 2π

0

∫ 2

1(sin r) rdrdθ

=

∫ 2π

0[sin r − r cos r]21 dθ =

∫ 2π

0(sin 2− 2 cos 2− sin 1 + cos 1) dθ

= 2π sin 2− 4π cos 2− 2π sin 1 + 2π cos 1

(23) (a)

x

y

1

1

∫∫

0≤x2+y2≤1(x + y) dxdy =

∫ π/2

0

∫ 1

0(r cos θ + r sin θ) rdrdθ

=

∫ π/2

0

∫ 1

0r2 (cos θ + sin θ) drdθ =

∫ π/2

0

[

1

3r3 (cos θ + sin θ)

]1

0

=

∫ π/2

0

1

3(cos θ + sin θ) dθ =

1

3[sin θ − cos θ]

π/20 =

2

3

(b)

2

2

1

1

y

x

95

Page 98: AMA SolutionsManual

CHAPTER 5. MULTIPLE INTEGRALS

∫∫

1≤x2+y2≤4(x + y) dxdy =

∫ π/2

0

∫ 2

1(r cos θ + r sin θ) rdrdθ

=

∫ π/2

0

∫ 2

1r2 (cos θ + sin θ) drdθ =

∫ π/2

0

[

1

3r3 (cos θ + sin θ)

]2

1

=

∫ π/2

0

(

8

3− 1

3

)

(cos θ + sin θ) dθ =7

3[sin θ − cos θ]

π/20 =

14

3

(24)

y

x1

y x= 3

∫∫

Ω

x2 + y2dxdy =

∫ π/3

0

∫ sec θ

0

(r cos θ)2 + (r sin θ)2rdrdθ

=

∫ π/3

0

∫ sec θ

0r2drdθ =

∫ π/3

0

[

1

3r3

]sec θ

0

=

∫ π/3

0

1

3sec3 θdθ =

1

3

∫ π/3

0sec θ

(

1 + tan2 θ)

=1

3

∫ π/3

0sec θdθ +

1

3

∫ π/3

0sec θ tan2 θdθ

=1

3[ln (sec θ + tan θ)]

π/30 +

1

3

∫ π/3

0tan θd (sec θ)

=1

3ln(

2 +√

3)

+1

3[tan θ sec θ]

π/30 −

∫ π/3

0sec3 θdθ

=1

6ln(

2 +√

3)

+1

3

√3

(25) (a)

1

1

1−

x

96

Page 99: AMA SolutionsManual

CHAPTER 5. MULTIPLE INTEGRALS

∫ 1

−1

√1−y2

0

x2 + y2dxdy =

∫ π/2

−π/2

∫ 1

0r2drdθ =

∫ π/2

−π/2

[

1

3r3

]1

0

=

∫ π/2

−π/2

1

3dθ =

[

1

]π/2

−π/2

=1

(b)

x

y

2

2

∫ 2

0

√4−x2

0

x2 + y2dxdy =

∫ π/2

0

∫ 2

0r2drdθ =

∫ π/2

0

[

1

3r3

]2

0

=

∫ π/2

0

8

3dθ =

[

8

]π/2

0

=4

(c)

x

y

Ω

1

112

∫ 2

1/2

√1−x2

0

3√

x2 + y2dxdy =

∫ π/3

0

∫ 1

12

sec θr4drdθ =

∫ π/3

0

[

1

5r5

]1

12

sec θ

=

∫ π/3

0

(

1

5− sec5 θ

160

)

dθ =1

15π − 11

640

√3− 3

1280ln(

2 +√

3)

Note: For n ≥ 2,

secn θdθ =1

n− 1secn−2 θ tan θ +

n− 2

n− 1

secn−2 θdθ.

97

Page 100: AMA SolutionsManual

CHAPTER 5. MULTIPLE INTEGRALS

(d)

x

y

Ω

1

112

∫ 1/2

0

√1−x2

0xy√

x2 + y2dxdy

=

∫ π/2

π/3

∫ 1

0r4 cos θ sin θdrdθ +

∫ π/3

0

∫ 12

sec θ

0r4 cos θ sin θdrdθ

=

∫ π/2

π/3

[

1

5r5 cos θ sin θ

]1

0

dθ +

∫ π/3

0

[

1

5r5 cos θ sin θ

] 12

sec θ

0

=1

5

∫ π/2

π/3cos θ sin θdθ +

∫ π/3

0

1

160sec5 θ cos θ sin θdθ

=1

5

∫ π/2

π/3

sin 2θ

2dθ −

∫ π/3

0

1

160sec4 θd (cos θ)

=1

5

[

−1

4cos 2θ

]π/2

π/3

− 1

160

[

−1

3cos−3 θ

]π/3

0

=1

40+

7

480=

19

480

(26)

∫∫

x2+y2≤b2(y + b) dxdy =

∫ 2π

0

∫ b

0(r sin θ + b) rdrdθ

=

∫ 2π

0

[

1

3(sin θ) r3 +

1

2br2

]b

0

dθ =

∫ 2π

0

(

1

3(sin θ) b3 +

1

2b3

)

=

[

−1

3(cos θ) b3 +

1

2b3θ

]2π

0

= πb3

(27)

∫∫

x2+y2≤1

[

1−(

x2 + y2)]

dxdy =

∫ 2π

0

∫ 1

0

(

1− r2)

rdrdθ

=

∫ 2π

0

[

−1

4r4 +

1

2r2

]1

0

dθ =

∫ 2π

0

1

4dθ =

[

1

]2π

0

2

98

Page 101: AMA SolutionsManual

CHAPTER 5. MULTIPLE INTEGRALS

(28)

2

∫∫

x2+y2≤4

3

(

1− x2 + y2

4

)

dxdy = 2

∫ 2π

0

∫ 2

0

3

(

1− r2

4

)

rdrdθ

= 2

∫ 2π

0

[

− 1

18

(

12− 3r2)3/2

]2

0

dθ = 2

∫ 2π

0

(

2

3

√12

)

= 2

[(

2

3

√12

)

θ

]2π

0

=16

3π√

3

(29)

∫∫

x2+y2≤5

(

5− x2 − y2)1/6

dxdy =

∫ 2π

0

√5

0

(

5− r2)1/6

rdrdθ

=

∫ 2π

0

[

−3

7

(

5− r2)7/6

]

√5

0

dθ =

∫ 2π

0

(

15

76√

5

)

dθ =

[

15

76√

]2π

0

=30

6√

5 = 17.6

(30)

∫∫

x2+y2≤1

4− x2 − y2dA =

∫ 2π

0

∫ 1

0

4− r2rdrdθ

=

∫ 2π

0

[

−1

3

(

4− r2)3/2

]1

0

dθ =

∫ 2π

0

(

−√

3 +8

3

)

=

[

−√

3 +8

3

]2π

0

=16π

3− 2π

√3

(31)

x

Ω

99

Page 102: AMA SolutionsManual

CHAPTER 5. MULTIPLE INTEGRALS

∫∫

(

1− x2 − y2)

dA =

∫ π/2

−π/2

∫ cos θ

0

(

1− r2)

rdrdθ

=

∫ π/2

−π/2

[

−1

4r4 +

1

2r2

]cos θ

0

dθ =

∫ π/2

−π/2

(

−1

4cos4 θ +

1

2cos2 θ

)

=5

32π

Note:∫

cos2 θdθ =

1 + cos 2θ

2dθ =

1

2θ +

1

4sin 2θ

and∫

cosn θdθ =1

n− 1cosn−1 θ sin θ +

n− 1

n

cosn−2 θdθ

(32)

2

∫∫

2xdA =

∫ π/2

−π/2

∫ 2 cos θ

02r2 cos θdrdθ =

∫ π/2

−π/2

[

2

3r3 cos θ

]2 cos θ

0

=

∫ π/2

−π/2

(

16

3cos4 θ

)

dθ = 2π

(33)

∫∫

x2 + y2dA =

∫ π/2

−π/2

∫ 2a cos θ

0r2drdθ =

∫ π/2

−π/2

[

1

3r3

]2a cos θ

0

=

∫ π/2

−π/2

(

8

3a3 cos3 θ

)

dθ =

∫ π/2

−π/2

(

8

3a3(

1− sin2 θ)

)

d (sin θ)

=

[

8

3a3

(

sin θ − 1

3sin3 θ

)]π/2

−π/2

=32

9a3

(34)

100

Page 103: AMA SolutionsManual

CHAPTER 5. MULTIPLE INTEGRALS

∫∫

b2 − b2

a2(x2 + y2)dA = b

∫ π

0

∫ a sin θ

0

1− r2

a2rdrdθ

= b

∫ π

0

[

−1

3a2

(

1− r2

a2

)3/2]a sin θ

0

dθ =2a2b

3

∫ π/2

0

(

1− cos3 θ)

=2a2b

3

∫ π/2

0dθ − 2a2b

3

∫ π/2

0

(

1− sin2 θ)

d (sin θ)

=1

3a2bπ − 4

9a2b

(35)

x = - y

Ix =

∫∫

Ry2ρ (x, y) dA =

∫ 2

0

∫ y−y2

−yy2 (x + y) dxdy =

∫ 2

0

[(

1

2x2y2 + y3x

)]y−y2

−y

dy

=

∫ 2

0

(

−2y5 +1

2y6 + 2y4

)

dy =

[

−1

3y6 +

1

14y7 +

2

5y5

]2

0

=64

105

(36)

x = y

y = 2 - x

101

Page 104: AMA SolutionsManual

CHAPTER 5. MULTIPLE INTEGRALS

M =

∫∫

Rρ (x, y) dA =

∫ 1

0

∫ 2−y

y(6x + 3y + 3) dxdy

=

∫ 1

0

[

3x2 + 3xy + 3x]2−y

ydy

=

∫ 1

0

(

−6y2 − 12y + 18)

dx =[

−2y3 − 6y2 + 18y]1

0= 10

My =

∫∫

Rxρ (x, y) dA =

∫ 1

0

∫ 2−y

xy (6x + 3y + 3) dxdy

=

∫ 1

0

(

22− 24y + 6y2 − 4y3)

dx

=[

22y − 12y2 + 2y3 − y4]1

0= 11

Mx =

∫∫

Ryρ (x, y) dA =

∫ 1

0

∫ 2−y

yy (6x + 3y + 3) dxdy

=

∫ 1

0

(

18y − 12y2 − 6y3)

dx

=

[

9y2 − 4y3 − 3

2y4

]1

0

=7

2

Hence,

x =My

M=

11

10and y =

Mx

M=

7/2

10=

7

20

(37)

M =

∫∫

Rρ (x, y) dA =

∫ 1

0

∫ 6

0(x + y + 1) dxdy

=

∫ 1

0

[

1

2x2 + yx + x

]6

0

dy

=

∫ 1

0(24 + 6y) dy =

[

24y + 3y2]1

0= 27

Mx =

∫∫

Ryρ (x, y) dA =

∫ 1

0

∫ 6

0y (x + y + 1) dxdy

=

∫ 1

0

[

1

2x2y + y2x + xy

]6

0

dy

=

∫ 1

0

(

24y + 6y2)

dy =[

12y2 + 2y3]1

0= 14

102

Page 105: AMA SolutionsManual

CHAPTER 5. MULTIPLE INTEGRALS

My =

∫∫

Rxρ (x, y) dA =

∫ 1

0

∫ 6

0x (x + y + 1) dxdy

=

∫ 1

0

[

1

3x3 +

1

2(y + 1) x2

]6

0

dy

=

∫ 1

0(90 + 18y) dy =

[

90y + 9y2]1

0= 99

Hence,

x =My

M=

99

27=

11

3and y =

Mx

M=

14

27

Iy =

∫∫

Rx2ρ (x, y) dA =

∫ 1

0

∫ 6

0x2 (x + y + 1) dxdy

=

∫ 1

0

[

1

4x4 +

1

3(y + 1)x3

]6

0

dy =

∫ 1

0(396 + 72y) dy = 432

(38)

x = 4 - y

Let k be the constant density,

M =

∫∫

Rρ (x, y) dA =

∫ 2

0

∫ 4−y

y2/2kdxdy

= k

∫ 2

0

(

4− y − y2

2

)

dy = k

[

4y − 1

2y2 − 1

6y3

]2

0

=14k

3

My =

∫∫

Rxρ (x, y) dA =

∫ 2

0

∫ 4−y

y2/2kxdxdy

= k

∫ 2

0

[

x2

2

]4−y

y2/2

= −1

8k

∫ 2

0

(

−64 + 32y − 4y2 + y4)

dy

= −1

8k

[

−64y + 16y2 − 4

3y3 +

1

5y5

]2

0

=128k

15

103

Page 106: AMA SolutionsManual

CHAPTER 5. MULTIPLE INTEGRALS

Mx =

∫∫

Ryρ (x, y) dA =

∫ 2

0

∫ 4−y

y2/2ykdxdy = k

∫ 2

0

(

4y − y2 − y3

2

)

dy

= k

[

2y2 − 1

3y3 − 1

8y4

]2

0

=10k

3

Hence,

x =My

M=

128k/15

14k/3=

64

35and y =

Mx

M=

10k/3

14k/3=

5

7

(39)

a-a 0

a

Density = kr

M =

∫∫

Rρ (x, y) dA =

∫ π

0

∫ a

0kr · rdrdθ

=

∫ π

0dθ

∫ a

0kr2dr = π

[

kr3

3

]a

0

=1

3πka3

Mx =

∫∫

Ryρ (x, y) dA =

∫ π

0

∫ a

0kr (r sin θ) · rdrdθ

=

∫ π

0sin θdθ

∫ a

0kr3dr = [− cos θ]2π

0 ·[

kr4

4

]a

0

=1

2ka4

Hence,

y =Mx

M=

ka4

213πka3

=3a

and by symmetry, x = 0.

(40) (a)

104

Page 107: AMA SolutionsManual

CHAPTER 5. MULTIPLE INTEGRALS

M =

∫∫

Rρ (x, y) dA =

∫ 1

0

∫ 1−y

0dxdy =

∫ 1

0(1− y) dy =

[

y − 1

2y2

]1

0

=1

2

Mx =

∫∫

Ryρ (x, y) dA =

∫ 1

0

∫ 1−y

0ydxdy =

∫ 1

0[yx]1−y

0 dy

=

∫ 1

0

(

y − y2)

dy =

[

1

2y2 − 1

3y3

]1

0

=1

6

My =

∫∫

Rxρ (x, y) dA =

∫ 1

0

∫ 1−y

0xdxdy

=

∫ 1

0

[

x2

2

]1−y

0

dy =

∫ 1

0

1

2(−1 + y)2 dy

=

[

1

6(−1 + y)3

]1

0

=1

6

Hence,

x =My

M=

1612

=1

3and y =

Mx

M=

1612

=1

3

(b)

a

a

0 a

M =

∫∫

Rρ (x, y) dA =

∫ π4

−π4

∫ a

0rdrdθ =

πa2

4

My =

∫∫

Rxρ (x, y) dA = 2

∫ π4

0

∫ a

0(r cos θ) rdrdθ =

1

3

√2a3

Hence,

x =My

M=

13

√2a3

πa2

4

=4√

2a

105

Page 108: AMA SolutionsManual

CHAPTER 5. MULTIPLE INTEGRALS

and by symmetry, y = 0.

(41) (a)

∫∫∫

Df (x, y, z) dx dy dz

=

∫ 2

0

∫ 1

0

∫ 1

−1yz dx dy dz =

∫ 1

−1dx

∫ 1

0y dy

∫ 2

0z dz

= [x]1−1

[

1

2y2

]1

0

[

1

2z2

]2

0

= (2)

(

1

2

)

(2) = 2

(b)

∫∫∫

Df (x, y, z) dx dy dz

=

∫ 2

0

∫ 1

0

∫ 1

−1(x + y + z) dx dy dz =

∫ 2

0

∫ 1

0

[

1

2x2 + xy + xz

]1

−1

dy dz

=

∫ 2

0

∫ 1

0(2y + 2z) dy dz = 2

∫ 2

0

[(

1

2y2 + yz

)]1

0

dz = 2

∫ 2

0

(

1

2+ z

)

dz = 6

(42) (a)

∫∫∫

Df (x, y, z) dx dy dz

=

∫ 1

0

∫ 1−x

0

∫ 1−x−y

0dz dy dx =

∫ 1

0

∫ 1−x

0(1− x− y) dy dx

=

∫ 1

0

[

y − xy − 1

2y2

]1−x

0

dx =

∫ 1

0

(

1

2− x +

1

2x2

)

dx =1

6

(b)

∫∫∫

Df (x, y, z) dx dy dz

=

∫ 1

0

∫ 1−x

0

∫ 1−x−y

0xy dz dy dx =

∫ 1

0x

∫ 1−x

0

(

y − xy − y2)

dy dx

=

∫ 1

0x

[

1

2y2 − 1

2xy2 − 1

3y3

]1−x

0

dx =1

6

∫ 1

0

(

x− 3x2 + 3x3 − x4)

dx =1

120

106

Page 109: AMA SolutionsManual

CHAPTER 5. MULTIPLE INTEGRALS

(43) (a)

∫∫∫

Df (x, y, z) dx dy dz

=

∫ 1

−1

∫ 2

0

∫ 1−x2

0(x + y) dz dy dx =

∫ 1

−1

∫ 2

0(x + y)

(

1− x2)

dy dx

=

∫ 1

−1

∫ 2

0

(

x− x3 + y − x2y)

dy dx =

∫ 1

−1

(

2x− 2x3 + 2− 2x2)

dx =8

3

(b)

∫∫∫

Df (x, y, z) dx dy dz

=

∫ 1

−1

∫ 2

0

∫ 1−x2

0y2 dz dy dx =

∫ 2

0y2 dy

∫ 1

−1

∫ 1−x2

0dz dx

=

[

1

3y3

]2

0

∫ 1

−1

(

1− x2)

dx =

(

8

3

)[

x− 1

3x3

]1

−1

=32

9

(44) (a)

∫∫∫

Ryz2dxdydz =

∫ 1

0

∫ 1

0

∫ 1

0yz2dzdydx =

∫ 1

0

∫ 1

0

[

1

3yz3

]1

0

dydx

=

∫ 1

0

∫ 1

0

1

3ydydx =

∫ 1

0

[

1

6y2

]1

0

dx

=

∫ 1

0

1

6dx =

1

6

(b)

107

Page 110: AMA SolutionsManual

CHAPTER 5. MULTIPLE INTEGRALS

∫∫∫

Rydxdydz =

∫ 1

0

∫ 1−y

0

∫ 1−x−y

0ydzdxdy =

∫ 1

0

∫ 1−y

0[yz]1−x−y

0 dxdy

=

∫ 1

0

∫ 1−y

0(y (1− y − x)) dxdy =

∫ 1

0

[

y

(

x− yx− 1

2x2

)]1−y

0

dy

=

∫ 1

0

(

1

2y − y2 +

1

2y3

)

dy

=

[

1

4y2 − 1

3y3 +

1

8y4

]1

0

=1

24

(c)

10.75

0.50.25

0

1

0.75

0.5

0.25

0

1

0.75

0.5

0.25

0

x y

z

x y

z

∫∫∫

Rxydxdydz =

∫ 1

0

∫ 1

0

∫ 1−y

0xydzdydx =

∫ 1

0xdx ·

∫ 1

0

∫ 1−y

0ydzdy

=

[

x2

2

]1

0

·∫ 1

0[yz]1−y

0 dy =1

2

∫ 1

0

(

y − y2)

dy

=1

2

[

1

2y2 − 1

3y3

]1

0

=1

12

(45) (a)

∫∫∫

D

(

x2 + y2)

dx dy dz =

∫ 2π

0

∫ 1

0

∫ r cos θ+r sin θ+2

0r3 dz dr dθ

=

∫ 2π

0

∫ 1

0

(

r4 cos θ + r4 sin θ + 2r3)

dr dθ

=

∫ 2π

0

[

1

5r5 cos θ +

1

5r5 sin θ +

1

2r4

]1

0

=

∫ 2π

0

(

1

5cos θ +

1

5sin θ +

1

2

)

=

[

1

5sin θ − 1

5cos θ +

1

]2π

0

= π

108

Page 111: AMA SolutionsManual

CHAPTER 5. MULTIPLE INTEGRALS

(b)

∫∫∫

Df (x, y, z) dx dy dz

=

∫ π

−π

∫ 1+cos θ

0

∫ r cos θ+r sin θ+2

0r3 dz dr dθ

=

∫ π

−π

∫ 1+cos θ

0

(

r4 cos θ + r4 sin θ + 2r3)

dr dθ

=

∫ π

−π

[

1

5r5 cos θ +

1

5r5 sin θ +

1

2r4

]1+cos θ

0

=

∫ π

−π

[

1

5(1 + cos θ)5 cos θ +

1

5(1 + cos θ)5 sin θ +

1

2(1 + cos θ)4

]

=1

5

(

105π

8

)

+1

5(0) +

1

2

(

35π

4

)

= 7π

(46) D =

(x, y, z) | 0 ≤ x2 + y2 + z2 ≤ 1

R = (ρ, φ, θ) | 0 ≤ ρ ≤ 1, 0 ≤ φ ≤ π, 0 ≤ θ ≤ 2π∫∫∫

Dρ (x, y, z) dx dy dz

=

∫ ∫ ∫

Rρ (ρ sinφ cos θ, ρ sinφ sin θ, ρ cos φ) ρ2 sinφdρ dφ dθ

=

∫ 2π

0

∫ π

0

∫ 1

0ρ4 sinφdρ dφ dθ =

∫ 2π

0dθ

∫ π

0sinφdφ

∫ 1

0ρ4 dρ

= [θ]2π0 [− cos φ]π0

[

1

5ρ5

]1

0

= (2π) (2)

(

1

5

)

=4

(47) D =

(x, y, z) | 1 ≤ x2 + y2 + z2 ≤ 4

R = (ρ, φ, θ) | 1 ≤ ρ ≤ 2, 0 ≤ φ ≤ π, 0 ≤ θ ≤ 2π∫∫∫

Dρ (x, y, z) dx dy dz

=

∫ 2π

0

∫ π

0

∫ 2

1ρ3 sinφdρ dφ dθ =

∫ 2π

0dθ

∫ π

0sinφdφ

∫ 2

1ρ3 dρ

= [θ]2π0 [− cos φ]π0

[

1

4ρ4

]2

1

= (2π) (2)

(

15

4

)

= 15π

(48)

z =√

x2 + y2

x2 + y2 + z2 = 1=⇒ x2 + y2 = 1

2

(The intersection of the surface is a cycle centred at the origin and radius is√

22 .)

109

Page 112: AMA SolutionsManual

CHAPTER 5. MULTIPLE INTEGRALS

D =

(x, y, z) | 0 ≤ x2 + y2 ≤ 12 ,√

x2 + y2 ≤ z ≤√

1− x2 − y2

R =

(ρ, φ, θ) | 0 ≤ ρ ≤ 1, 0 ≤ φ ≤ π4 , 0 ≤ θ ≤ 2π

∫∫∫

Dρ (x, y, z) dx dy dz

=

∫ 2π

0

∫ π/4

0

∫ 1

0ρ2 sinφdρ dφ dθ

=

∫ 2π

0

∫ π/4

0sinφ

[

1

3ρ3

]1

0

dφ dθ =1

3

∫ 2π

0dθ

∫ π/4

0sinφdφ =

1

3[θ]2π

0 [− cos φ]π/40

=1

3(2π)

(

1−√

2

2

)

=

(

2−√

2)

π

3

(49) S = (r, θ, z) |0 ≤ r ≤ 4, 0 ≤ θ ≤ 2π, 0 ≤ z ≤ 4− r,

V =

∫ 2π

0

∫ 4

0

∫ 4−r

0r dz dr dθ =

∫ 2π

0

∫ 4

0

(

4r − r2)

dr dθ = (2π)

(

32− 64

3

)

=64

(50)

R

h

y

z

x

(a) Let σ = kz.

Mass = M =

∫∫∫

TσdV =

∫ 2π

0

∫ R

0

∫ h

0rkzdzdrdθ

=

∫ 2π

0

∫ R

0

[

1

2rkz2

]h

0

drdθ =

∫ 2π

0

∫ R

0

1

2rkh2drdθ

=

∫ 2π

0

[

1

4r2kh2

]R

0

dθ =

∫ 2π

0

1

4R2kh2dθ =

[

1

4R2kh2θ

]2π

0

=1

2kπR2h2

110

Page 113: AMA SolutionsManual

CHAPTER 5. MULTIPLE INTEGRALS

(b) By symmetry, x = y = 0

Mz =

∫∫∫

TzσdV =

∫ 2π

0

∫ R

0

∫ h

0rkz2dzdrdθ =

∫ 2π

0dθ

∫ R

0rdr

∫ h

0kz2dz

= (2π)×[

1

2r2

]R

0

×[

1

3kz3

]h

0

= (2π)× 1

2R2 × 1

3kh3 =

1

3πkR2h3

z =13πkR2h3

12kπR2h2

=2

3h

(c)

Iz =

∫∫∫

Tσ(

x2 + y2)

dV =

∫ 2π

0dθ

∫ R

0r3dr

∫ h

0kzdz = (2π)×

[

1

4r4

]R

0

×[

1

2kz2

]h

0

= (2π)× 1

4R4 × 1

2kh2 =

1

4πR4kh2 =

1

2MR2

(51) (a) Since σ = constant

M = σ

∫∫∫

TdV = σ

∫ 2π

0dθ

∫ R

0rdr

∫ h

0dz = σ × (2π)×

[

1

2r2

]R

0

× [z]h0 = πσR2h

and

Iz = σ

∫∫∫

T

(

x2 + y2)

dV = σ

∫ 2π

0dθ

∫ R

0r3dr

∫ h

0dz = σ × (2π)×

[

1

4r4

]R

0

× [z]h0

= σ × (2π)× 1

4R4 × h =

1

2πσR4h =

M

2R2

(b)

Ix = σ

∫∫∫

T

(

y2 + z2)

dV = σ

∫ 2π

0dθ

∫ R

0rdr

∫ h

0

(

r2 sin2 θ + z2)

dz

= σ

∫ 2π

0sin2 θdθ

∫ R

0r3dr

∫ h

0dz + σ

∫ 2π

0dθ

∫ R

0rdr

∫ h

0z2dz

= σ

∫ 2π

0sin2 θdθ

∫ R

0r3dr

∫ h

0dz + σ × (2π)×

[

1

2r2

]R

0

×[

1

3z3

]h

0

= σ ×[

1

2θ − sin 2θ

4

]2π

0

×[

1

4r4

]R

0

× [z]h0 + σ × (2π)× 1

4R4 × h

=1

4σπR4h +

1

3σπR2h3 = M

(

R2

4+

h2

3

)

111

Page 114: AMA SolutionsManual

CHAPTER 5. MULTIPLE INTEGRALS

(c) By part (b), the moment of inertia is

M

(

R2

4+

h2

3

)

−M

(

h

2

)2

= M

(

R2

4+

h2

12

)

(52)

h

y

x

z

Rh

rz

=

(a)

V =

∫∫∫

TdV =

∫ 2π

0dθ

∫ R

0rdr

∫ h

hrR

dz =

∫ 2π

0dθ

∫ R

0r

(

h− hr

R

)

dr

= (2π)×[

1

2hr2 − 1

3hr3

]R

0

= πhR2 − 2

3πhR2 =

1

3πhR2

(b)

M = σV =1

3σπhR2

zM = σ

∫∫∫

TzdV = σ

∫ 2π

0dθ

∫ R

0rdr

∫ h

hrR

zdz = σ

∫ 2π

0dθ

∫ R

0r

(

h2

2− h2r2

2R2

)

dr

= σ

∫ 2π

0

1

8h2R2dθ =

1

4σπh2R2

Hence,

z =3

4h

By symmetry, x = y = 0.

112

Page 115: AMA SolutionsManual

CHAPTER 5. MULTIPLE INTEGRALS

(c)

Iz = σ

∫∫∫

T

(

x2 + y2)

dV = σ

∫ 2π

0dθ

∫ R

0r3dr

∫ h

hrR

dz

= σ

∫ 2π

0dθ

∫ R

0r3

(

h− hr

R

)

dr

= σ

∫ 2π

0

1

20hR4dθ =

1

10σπhR4 =

3

10MR2

(d)

Ix = σ

∫∫∫

T

(

y2 + z2)

dV = σ

∫ 2π

0dθ

∫ R

0rdr

∫ h

hrR

(

r2 sin2 θ + z2)

dz

= σ

∫ 2π

0sin2 θdθ

∫ R

0r3dr

∫ h

hrR

dz + σ

∫ 2π

0dθ

∫ R

0rdr

∫ h

hrR

z2dz

= σ

∫ 2π

0sin2 θdθ

∫ R

0r3

(

h− hr

R

)

dr + σ

∫ 2π

0dθ

∫ R

0

1

3r

(

h3 − h3r3

R3

)

dr

= σ

∫ 2π

0

1

20hR4 sin2 θdθ + σ

∫ 2π

0

1

10h3R2dθ =

1

20σπhR4 +

1

5σπR2h3

(53)

(a)

V =

∫∫∫

TdV =

∫ 2π

0dθ

∫ 1

0rdr

∫ 1−r2

0dz =

∫ 2π

0dθ

∫ 1

0r(

1− r2)

dr

=

∫ 2π

0

1

4dθ =

π

2

(b) Note that σ = kz

M =

∫∫∫

TσdV = k

∫∫∫

TzdV = k

∫ 2π

0dθ

∫ 1

0rdr

∫ 1−r2

0zdz

= k

∫ 2π

0dθ

∫ 1

0

1

2r(

1− r2)2

dr = k

∫ 2π

0

1

12dθ =

πk

6

113

Page 116: AMA SolutionsManual

CHAPTER 5. MULTIPLE INTEGRALS

(c) Note that σ = k(

r2 + z2)

M =

∫∫∫

TσdV = k

∫∫∫

T

(

r2 + z2)

dV

= k

∫ 2π

0dθ

∫ 1

0rdr

∫ 1−r2

0

(

r2 + z2)

dz

= k

∫ 2π

0dθ

∫ 1

0r

(

r2(

1− r2)

+1

3

(

1− r2)3)

dr

= k

∫ 2π

0

1

8dθ =

πk

4

(54)

V =

∫∫∫

dV =

∫ π/2

−π/2dθ

∫ 2a cos θ

0rdr

∫ r2/a

0dz =

∫ π/2

−π/2dθ

∫ 2a cos θ

0

r3

adr

=

∫ π/2

−π/24a3 cos4 θdθ =

3

2πa3

(55)

V =

∫∫∫

dV =

∫ π/2

−π/2dθ

∫ 2a cos θ

0rdr

∫ r

0dz =

∫ π/2

−π/2dθ

∫ 2a cos θ

0r2dr

=

∫ π/2

−π/2

8

3a3 cos3 θdθ =

32

9a3

(56)

V =

∫∫∫

dV =

∫ π/2

−π/2dθ

∫ 2 cos θ

0rdr

∫ 12(4+r cos θ)

0dz

=

∫ π/2

−π/2dθ

∫ 2 cos θ

0

1

2r (4 + r cos θ) dr

=

∫ π/2

−π/2

(

4

3cos4 θ + 4 cos2 θ

)

dθ =5

(57)

V =

∫∫∫

dV =

∫ π/2

−π/2dθ

∫ a cos θ

0rdr

∫ a−r

0dz =

∫ π/2

−π/2dθ

∫ a cos θ

0r (a− r) dr

=

∫ π/2

−π/2

(

−1

3a3 cos3 θ +

1

2a3 cos2 θ

)

dθ =1

4πa3 − 4

9a3

114

Page 117: AMA SolutionsManual

CHAPTER 5. MULTIPLE INTEGRALS

(58)

z =

z = r + 1

52 - r2

Where the sphere x2 + y2 + z2 = 25 intersects the cone z =√

x2 + y2 + 1, we have

r2 + (r + 1)2 = 25, thus, r = 3.

V =

∫ 2π

0dθ

∫ 3

0rdr

√25−r2

r+1dz

=

∫ 2π

0dθ

∫ 3

0r(√

25− r2 − (r + 1))

dr

=

∫ 2π

0

[

−1

3

(

25− r2)3/2 − 1

3r3 − 1

2r2

]3

0

=

∫ 2π

0

41

6dθ =

41

(59)

x

x

y

1

z = x2 + y2

z = x

The surfaces intersect at x2 + y2 = x, thus r = cos θ.

V =

∫∫∫

dV =

∫ π/2

−π/2dθ

∫ cos θ

0rdr

∫ r cos θ

r2

dz

=

∫ π/2

−π/2dθ

∫ cos θ

0r(

r cos θ − r2)

dr

=

∫ π/2

−π/2

(

−1

4cos4 θ +

1

3cos4 θ

)

dθ =1

32π

115

Page 118: AMA SolutionsManual

CHAPTER 5. MULTIPLE INTEGRALS

(60)

cone

hyperbola

z

Where the hyperboloid intersects the cone, we have r2 + a2 = 2r2, thus r = a.

V =

∫ 2π

0dθ

∫ a

0rdr

√a2+r2

√2r

dz =

∫ 2π

0dθ

∫ a

0r(√

a2 + r2 −√

2r)

dr

=

∫ 2π

0

[

1

3

(

a2 + r2)3/2 −

√2

3r3

]a

0

dθ =

∫ 2π

0

(

1

3a3√

2− 1

3a3

)

=2

3πa3√

2− 2

3πa3 =

2

3πa3

(√2− 1

)

(61)

y

x

z

21

Sphere: x2 + y2 + z2 = 1

Cone: z = 3(x2 + y2)

21

The cone z =√

3 (x2 + y2) has equation z =√

3r. It intersects the sphere z2 + r2 = 1 at

3r2 + r2 = 1 or r = 12 .

V =

∫ 2π

0dθ

∫ 12

0rdr

√1−r2

√3r

dz =

∫ 2π

0dθ

∫ 12

0r(√

1− r2 −√

3r)

dr

=

∫ 2π

0

[

−1

3

(

1− r2)3/2 − 1

3r3√

3

] 12

0

dθ =

∫ 2π

0

(

−1

6

√3 +

1

3

)

dθ =2

3π − 1

3π√

3

116

Page 119: AMA SolutionsManual

CHAPTER 5. MULTIPLE INTEGRALS

(62)

V =

∫ 2π

0dθ

∫ 2

1rdr

9− r2

4

0dz

=

∫ 2π

0dθ

∫ 2

1

1

2r√

(36− r2)dr =

∫ 2π

0

[

−1

6

(

36− r2)3/2

]2

1

=

∫ 2π

0

(

−16

3

√32 +

35

6

√35

)

dθ = −128

3π√

2 +35

3π√

35 = 27.273

(63) (a) r = 1. Cylinder of radius 1, with the z-axis as its axis.

φ = π2 . The xy-plane.

φ = 3π4 . The cone shown in diagram.

=34

y

x

z

θ = π4 . A plane prependicular to the xy-plane with the angle between this plane and

the xz-plane is π4 .

z = 1. The plane parallel to the xy-plane through (0, 0, 1) .

ρ = cos φ. The sphere with centre at (x, y, z) =(

0, 0, 12

)

and radius 12 .

(64)

V =

∫ 2π

0

∫ π

0

∫ R

0ρ2 sin φdρdφdθ =

∫ 2π

0

∫ π

0

1

3R3 sinφdφdθ =

∫ 2π

0

2

3R3dθ =

4

3πR3

(65) r = ρ sinφ, θ = θ, z = ρ cos φ.

(66)

R =

(x, y, z) | − 2 ≤ x ≤ 2,−√

4− x2 ≤ y ≤√

4− x2, 0 ≤ z ≤√

4− (x2 + y2)

=

(ρ, φ, θ) |0 ≤ ρ ≤ 2, 0 ≤ φ ≤ π

2, 0 ≤ θ ≤ 2π

117

Page 120: AMA SolutionsManual

CHAPTER 5. MULTIPLE INTEGRALS

∫ 2

−2

√4−x2

−√

4−x2

√4−(x2+y2)

0z2√

x2 + y2 + z2 dzdydx

=

∫ 2π

0

∫ π/2

0

∫ 2

0

(

ρ3 cos2 φ) (

ρ2 sinφ)

dρdφdθ

=

∫ 2π

0

∫ π/2

0

[

1

6ρ6

]2

0

cos2 φ sinφdφdθ

=32

3

∫ 2π

0

∫ π/2

0cos2 φ sinφdφdθ

=32

3

∫ 2π

0

[

−1

3cos3 φ

]π/2

0

=32

9

∫ 2π

0dθ =

64π

9

(67) Substitute x = ρ sinφ cos θ, y = ρ sinφ sin θ, z = ρ cos φ into z =√

3x2 + 3y2, then

ρ cos φ =√

3 (ρ sinφ cos θ)2 + 3 (ρ sinφ sin θ)2 =√

3ρ sinφ =⇒ tan φ = 1√3

=⇒ φ = π6 .

S =

(ρ, φ, θ) |0 ≤ ρ ≤ 4, 0 ≤ φ ≤ π6 , 0 ≤ θ ≤ 2π

V =∫ 2π0

∫ π/60

∫ 40 ρ2 sinφdρ dφ dθ = (2π)

(

1−√

32

)

(

643

)

= 643 π(

2−√

3)

= 17.958

(68)

∫ 3

0

√9−z2

0

∫ x

0xy dydxdz =

∫ 3

0

√9−z2

0x

[

y2

2

]x

0

dxdz =1

2

∫ 3

0

√9−z2

0x3 dxdz

=1

2

∫ 3

0

[

x4

4

]

√9−z2

0

dz =1

8

∫ 3

0

(

9− z2)2

dz

=1

8

∫ 3

0

(

81− 18z2 + z4)

dz =81

5

(69)

V =

∫ α

0

∫ π

0

∫ R

0ρ2 sinφdρdφdθ =

∫ α

0

∫ π

0

1

3R3 sin φdφdθ =

∫ α

0

2

3R3dθ =

2

3αR3

118

Page 121: AMA SolutionsManual

CHAPTER 5. MULTIPLE INTEGRALS

(70) Now, σ = k (R− ρ) , the mass is

M =

∫ ∫ ∫

σdV =

∫ 2π

0

∫ π

0

∫ R

0k (R− ρ) ρ2 sinφdρdφdθ

=

∫ 2π

0

∫ π

0

[

k (sinφ)

(

−1

4ρ4 +

1

3Rρ3

)]R

0

dφdθ

=

∫ 2π

0

∫ π

0

1

12R4k sinφdφdθ

=

∫ 2π

0

1

6R4kdθ =

1

3πR4k

(71)

h

y

RBase

Now, σ = kρ, tan α = Rh . On the base, ρ cos φ = h. The mass is

M =

∫ ∫ ∫

kρdV =

∫ 2π

0

∫ α

0

∫ h sec φ

0kρ3 sin φdρdφdθ

=

∫ 2π

0

∫ α

0

[

1

4kρ4 sinφ

]h sec φ

0

dφdθ =

∫ 2π

0

∫ α

0

1

4

h4

cos4 φk sin φdφdθ

=

∫ 2π

0

1

12h4k

(

sec3 α− 1)

dθ =1

6πh4k

(

sec3 α− 1)

=1

6πh4k

(

(

R2 + h2

h2

)3/2

− 1

)

=1

6πhk

(

(

R2 + h2)3/2 − h3

)

(72)

V =

∫∫∫

conedV =

∫ 2π

0

∫ α

0

∫ h sec φ

0ρ2 sinφdρdφdθ =

∫ 2π

0

∫ α

0

[

1

3ρ3 sinφ

]h sec φ

0

dφdθ

=

∫ 2π

0

∫ α

0

1

3

h3

cos3 φsinφdφdθ =

∫ 2π

0

1

6h3(

sec3 α− 1)

=1

3πh3

(

sec2 α− 1)

=1

3πh3 tan2 α =

1

3πR2h

119

Page 122: AMA SolutionsManual

CHAPTER 5. MULTIPLE INTEGRALS

(73) (a) i.

Iz =

∫∫∫

ballσ(

x2 + y2)

dV = σ

∫ 2π

0

∫ π

0

∫ R

0ρ2 sin2 φ · ρ2 sinφdρdφdθ

= σ

∫ 2π

0

∫ π

0

∫ R

0ρ4 sin3 φdρdφdθ = σ

∫ 2π

0

∫ π

0

R5

5sin3 φdφdθ

= σ

∫ 2π

0

4

15R5dθ =

8

15σπR5 =

2

5MR2

By parallel axis theorem,

I =

(

2

5MR2 + MR2

)

=7

5MR2

(b)

Mxy =

∫∫∫

TzσdV = σ

∫ 2π

0

∫ π/2

0

∫ R

0ρ2 sinφ · ρ cos φdρdφdθ

= σ

∫ 2π

0

∫ π/2

0

∫ R

0ρ3 sinφ cos φdρdφdθ = σ

∫ 2π

0

∫ π/2

0

R4

4sinφ cos φdφdθ

= σ

∫ 2π

0

∫ π/2

0

R4

4

sin 2φ

2dφdθ = σ

∫ 2π

0

1

8R4dθ =

1

4σπR4

Thus

z =Mxy

M/2=

14σπR4

23σπR3

=3

8R

and by symmetry x = y = 0.

(74) (a)

Iz =

∫∫∫

Tσ(

x2 + y2)

dV = σ

∫ 2π

0

∫ π

0

∫ R2

R1

ρ2 sin2 φ · ρ2 sinφdρdφdθ

= σ

∫ 2π

0

∫ π

0

∫ R2

R1

ρ4 sin3 φdρdφdθ = σ

∫ 2π

0

∫ π

0

(

R52

5− R5

1

5

)

sin3 φdφdθ

= σ

∫ 2π

0

4

15

(

R52 −R5

1

)

dθ =8

15σπ(

R52 −R5

1

)

=2

5M

R52 −R5

1

R32 −R3

1

(b) Setting R2 = R and R1 → R in part (a), we get

Iz = limR1→R

2

5M

R5 −R51

R3 −R31

=2

5M lim

R1→R

R5 −R51

R3 −R31

=2

5M lim

R1→R

−5R41

−3R21

=2

3MR2

120

Page 123: AMA SolutionsManual

CHAPTER 5. MULTIPLE INTEGRALS

(c) By parallel axis theorem,

I =2

3MR2 + MR2 =

5

3R2M

(d)

Mxy =

∫∫∫

TzσdV = σ

∫ 2π

0

∫ π/2

0

∫ R2

R1

ρ2 sinφ · ρ cos φdρdφdθ

= σ

∫ 2π

0

∫ π/2

0

∫ R2

R1

ρ3 sinφ cos φdρdφdθ

= σ

∫ 2π

0

∫ π/2

0

(

R42

4− R4

1

4

)

sinφ cos φdφdθ

= σ

∫ 2π

0

∫ π/2

0

(

R42

4− R4

1

4

)

sin 2φ

2dφdθ

= σ

∫ 2π

0

(

R42

8− R4

1

8

)

dθ =1

4πσ(

R42 −R4

1

)

and

M =

∫∫∫

TσdV = σ

∫ 2π

0

∫ π/2

0

∫ R2

R1

ρ2 sinφdρdφdθ =2

3πσ(

R32 −R3

1

)

Thus

z =Mxy

M=

14πσ

(

R42 −R4

1

)

23πσ

(

R32 −R3

1

) =3

8

R42 −R4

1

R32 −R3

1

and by symmetry x = y = 0.

(e) Setting R2 = R, R1 → R in part (a), we get

z = limR1→R

3

8

R4 −R41

R3 −R31

=1

2R

(75) (a) R sec φ ≤ ρ ≤ 2R cos φ, 0 ≤ φ ≤ π4 , 0 ≤ θ ≤ 2π.

(b) Sphere: ρ2 sin2 φ + (ρ cos φ−R)2 = R2, therefore ρ = 2R cos φ

121

Page 124: AMA SolutionsManual

CHAPTER 5. MULTIPLE INTEGRALS

(76) (a) σ = kρ

M =

∫∫∫

σdV =

∫ 2π

0

∫ π/2

0

∫ 2R cos φ

0ρ2 sinφ · kρdρdφdθ

=

∫ 2π

0

∫ π/2

0

[

1

4kρ4 sinφ

]2R cos φ

0

dφdθ

= k

∫ 2π

0

∫ π/2

04R4 cos4 φ sin φdφdθ = k

∫ 2π

0

4

5R4dθ =

8

5kπR4

(b) σ = kρ sinφ

M =

∫∫∫

σdV =

∫ 2π

0

∫ π/2

0

∫ 2R cos φ

0ρ2 sin φ · kρ sinφdρdφdθ

=

∫ 2π

0

∫ π/2

0

[

1

4kρ4 sin2 φ

]2R cos φ

0

dφdθ

= k

∫ 2π

0

∫ π/2

04R4 cos4 φ sin2 φdφdθ

= k

∫ 2π

0

1

8πR4dθ =

1

4π2kR4

(c) σ = kρ cos2 θ sinφ

M =

∫∫∫

σdV =

∫ 2π

0

∫ π/2

0

∫ 2R cos φ

0ρ2 sinφ · kρ cos2 θ sinφdρdφdθ

= k

∫ 2π

0

∫ π/2

0

[

1

4kρ4 sin2 φ cos2 θ

]2R cos φ

0

dφdθ

= k

∫ 2π

0

∫ π/2

04R4 cos4 φ sin2 φ cos2 θdφdθ

= k

∫ 2π

0

1

8πR4 cos2 θdθ =

1

8π2kR4

(77)

x

z

4

122

Page 125: AMA SolutionsManual

CHAPTER 5. MULTIPLE INTEGRALS

Where the sphere ρ = 2√

2 cos φ intersects the sphere ρ = 2, we have 2 = 2√

2 cos φ giving

φ = π4 .

V =

∫ 2π

0

∫ π/4

0

∫ 2

0ρ2 sin φdρdφdθ +

∫ 2π

0

∫ π/2

π/4

∫ 2√

2 cos φ

0ρ2 sinφdρdφdθ

=

∫ 2π

0

∫ π/4

0

8

3sinφdφdθ +

∫ 2π

0

∫ π/2

π/4

16

3

√2 cos3 φ sin φdφdθ

=

∫ 2π

0

(

8

3− 4

3

√2

)

dθ +

∫ 2π

0

1

3

√2dθ

=16

3π − 8

3π√

2 +2

3π√

2 =16

3π − 2π

√2

(78)

x

z

V =

∫ 2π

0

∫ π

0

∫ 1−cos φ

0ρ2 sinφdρdφdθ =

∫ 2π

0

∫ π

0

1

3(1− cos φ)3 sinφdφdθ

=

∫ 2π

0

1

12

[

(1− cos φ)4]π

0dθ =

∫ 2π

0

4

3dθ =

8

123

Page 126: AMA SolutionsManual
Page 127: AMA SolutionsManual

Chapter 6

Vector Calculus

(1) Let θ be the angle the two vectors a and b. Since (a • b) = ‖a‖ ‖b‖ cos θ,

‖a× b‖2 = (‖a‖ ‖b‖ sin θ)2 = ‖a‖2 ‖b‖2 sin2 θ = ‖a‖2 ‖b‖2(

1− cos2 θ)

= ‖a‖2 ‖b‖2 − ‖a‖2 ‖b‖2 cos2 θ = ‖a‖2 ‖b‖2 − (a • b)2

A

B

C

(2)−−→AB = b1i + b2j− (a1i + a2j) = (b1 − a1) i + (b2 − a2) j

−→AC = c1i + c2j− (a1i + a2j) = (c1 − a1) i + (c2 − a2) j

The area of the triangle is

1

2

−−→AB∥

−→AC∥

∥ sin θ

=1

2

−−→AB ×−→AC

∥=

1

2

det

i j k

(b1 − a1) (b2 − a2) 0

(c1 − a1) (c2 − a2) 0

=1

2

det

(

(b1 − a1) (b2 − a2)

(c1 − a1) (c2 − a2)

)

k

=1

2

det

(

(b1 − a1) (b2 − a2)

(c1 − a1) (c2 − a2)

)∣

where θ is the angle between−−→AB and

−→AC.

(3) Let x be the position vector of the point (x, y, z) .

125

Page 128: AMA SolutionsManual

CHAPTER 6. VECTOR CALCULUS

B

0

x

(XB)2 = (OX)2 − (OB)2 = ‖x‖2 −∥

−−→OB

2= ‖x‖2 − (‖x‖ cos θ)2

= ‖x‖2 −(

‖x‖ x • a‖x‖ ‖a‖

)2

= ‖x‖2 − (x • a)2

‖a‖2.

(4) (a)

∇f |(2,1) =∂ (y − 3x)

∂xi +

∂ (y − 3x)

∂yj

(2,1)

= −3i + j

(b)

∇f |(2,1,−1) =2x2 + z

xi + 2yj + (−4z + lnx)k

(2,1,−1)

=7

2i + 2j + (4 + ln 2)k

(5) (a)

∇ • F =

(

∂xi +

∂yj +

∂zk

)

•((

x2 − y)

i + (4z) j +(

x2)

k)

=∂

∂x

(

x2 − y)

+∂

∂y(4z) +

∂z

(

x2)

= 2x

∇× F =

i j k∂∂x

∂∂y

∂∂z

(

x2 − y)

(4z)(

x2)

= −4i−2xj + k

(b)

∇ • F =

(

∂xi +

∂yj +

∂zk

)

• ((yz) i + (xz) j + (xy)k) = 0

∇× F =

i j k∂∂x

∂∂y

∂∂z

(yz) (xz) (xy)

= 0

126

Page 129: AMA SolutionsManual

CHAPTER 6. VECTOR CALCULUS

(c)

∇ • F =

(

∂xi +

∂zj

)

• ((−y) i + (x) j) =∂

∂x(−y) +

∂y(x) = 0

∇× F =

i j k∂∂x

∂∂y

∂∂z

(−y) (x) 0

= 2k

(d)

∇ • F =

(

∂xi +

∂yj +

∂zk

)

•(

(2xz) i + (−xy) j +(

−z2)

k)

= −x

∇× F =

i j k∂∂x

∂∂y

∂∂z

(2xz) (−xy) 0

= 2xj− yk

(6) (a) div F (x, y, z) = ∇ • F (x, y, z) = 2x + 6y − 4z

div F (x0, y0, z0) = 2x0 + 6y0 − 4z0

curlF (x, y, z) = ∇× F (x, y, z) =

i j k∂∂x

∂∂y

∂∂z

x2 3y2 −2z2

= 0

(b) div F (x, y, z) = 0

curlF (x, y, z) =

i j k∂∂x

∂∂y

∂∂z

eyz ezx exy

= 〈xexy − xezx, yeyz − yexy, zezx − zeyz〉

curlF (0, 1, 2) =⟨

0, e2 − 1, 2− 2e2⟩

(c) div F (x, y, z) = 0

curlF (x, y, z) =

i j k∂∂x

∂∂y

∂∂z

sin yz cos zx xy

= 〈x− x sin zx, y cos yz − y,−z sin zx− z cos yz〉

curlF(

0, 1,π

2

)

= 〈0,−1, 0〉

(d) div F (x, y, z) = 2xy− 6xy2z2 + 1

div F (2, 1, 1) = 4− 12 + 1 = −7

127

Page 130: AMA SolutionsManual

CHAPTER 6. VECTOR CALCULUS

curlF (x, y, z) =

i j k∂∂x

∂∂y

∂∂z

x2y − z2 −2xy3z2 xy + z

=⟨

x + 4xy3z,−2z − y,−2y3z2 − x2⟩

curlF (2, 1, 1) = 〈10,−3,−6〉

(7) (a)

∇2f = ∇ •∇f

= ∇ •(

∂(

ln(

x2 + y2))

∂xi +

∂(

ln(

x2 + y2))

∂yj

)

=

(

∂xi +

∂yj

)

•(

2x

x2 + y2i +

2y

x2 + y2j

)

=∂

∂x

(

2x

x2 + y2

)

+∂

∂y

(

2y

x2 + y2

)

= 2−x2 + y2

(x2 + y2)2− 2−x2 + y2

(x2 + y2)2= 0

(b)

∇2f = ∇ •∇f

=

(

∂xi +

∂yj

)

•((

3Ax2 + 2Bxy + Cy2)

i +(

Bx2 + 2Cxy + 3Dy2)

j)

=∂

∂x

(

3Ax2 + 2Bxy + Cy2)

+∂

∂y

(

Bx2 + 2Cxy + 3Dy2)

= 6Ax + 2By + 2Cx + 6Dy

(c)

∇2f = ∇ •∇f

= ∇ •(

∂r−1

∂xi +

∂r−1

∂yj +

∂r−1

∂zj

)

=

(

∂xi +

∂yj +

∂zk

)

•(

−r−2 x

ri− r−2 y

rj−−r−2 z

rk)

= − ∂

∂x

(

r−3x)

− ∂

∂y

(

r−3y)

− ∂

∂z

(

r−3z)

= −r−3 + 3r−4 x2

r− r−3 + 3r−4 y2

r− r−3 + 3r−4 z2

r

= −3r−3 + 3r−5(

x2 + y2 + z2)

= −3r−3 + 3r−3 = 0

128

Page 131: AMA SolutionsManual

CHAPTER 6. VECTOR CALCULUS

(8)

∇ • (f (r) r)

=

(

∂xi+

∂yj+

∂zk

)

• (f (r)xi+f (r) yj+f (r) zk)

=

(

f (r)∂x

∂x+ x

∂f (r)

∂x

)

+

(

f (r)∂y

∂y+ y

∂f (r)

∂y

)

+

(

f (r)∂z

∂z+ z

∂f (r)

∂z

)

=

(

f (r) + xf ′ (r)∂r

∂x

)

+

(

f (r) + yf ′ (r)∂r

∂y

)

+

(

f (r) + zf ′ (r)∂r

∂z

)

=

(

f (r) + f ′ (r)x2

r

)

+

(

f (r) + f ′ (r)y2

r

)

+

(

f (r) + f ′ (r)z2

r

)

= 3f (r) + f ′ (r)x2 + y2 + z2

r= 3f (r) + rf ′ (r)

(9) (a) To find a φ such that ∇φ = A,

φx = ex cos y + yz ⇒ φ =

(ex cos y + yz) dx = ex cos y + xyz + f (y, z)

φy = xz − ex sin y ⇒ φ =

(xz − ex sin y) dy = ex cos y + xyz + g (x, z)

φz = xy + z + 2⇒ φ =

(xy + z + 2) dz = xyz +1

2z2 + 2z + h (x, y)

Thus, φ = ex cos y + xyz + 12z2 + 2z + C.

(b) To find a φ such that ∇φ = A,

φx = y + z ⇒ φ =

(y + z) dx = xy + xz + f (y, z)

φy = x + z ⇒ φ =

(x + z) dy = xy + yz + g (x, z)

φz = x + y ⇒ φ =

(x + y) dz = xz + yz + h (x, y)

Thus, φ = xy + yz + xz + C.

(c) To find a φ such that ∇φ = A,

φx = y sin z ⇒ φ =

y sin zdx = xy sin z + f (y, z)

φy = x sin z ⇒ φ =

x sin zdy = xy sin z + g (x, z)

φz = xy cos z ⇒ φ =

xy cos zdz = xy sin z + h (x, z)

Thus, φ = xy sin z + C.

129

Page 132: AMA SolutionsManual

CHAPTER 6. VECTOR CALCULUS

(10)

∇× F =

i j k∂∂x

∂∂y

∂∂z

(

2x2 − 3)

(−4z) cos z

=

[

∂y(cos z)− ∂

∂z(−4z)

]

i−[

∂x(cos z)− ∂

∂z

(

2x2 − 3)

]

j

+

[

∂x(−4z)− ∂

∂y

(

2x2 − 3)

]

k

= 4i 6= 0

Hence, F is not a conservative vector field.

(11) Let r (t) =[

4− t2, t]

, −3 ≤ t ≤ 2, then r′ (t) = [−2t, 1].

Γy2 dx + x dy =

∫ 2

−3

[

(t)2 (−2t) +(

4− t2)

(1)]

dt =

∫ 2

−3

(

4− t2 − 2t3)

dt =245

6

(12) Let r (t) = [cos t, sin t, t], 0 ≤ t ≤ 2π, then r′ (t) = [− sin t, cos t, 1].

Γy sin z ds =

∫ 2π

0(sin t) sin (t)

(− sin t)2 + (cos t)2 + (1)2 dt =√

2

∫ 2π

0sin2 t dt

=

√2

2

∫ 2π

0(1− cos 2t) dt =

√2π

(13) Let x (t) = 1 + 2t, y (t) = 2, z (t) = 3− 2t and 0 ≤ t ≤ 1.

f (x (t) , y (t) , z (t)) = (1 + 2t) + 2 (3− 2t) = 7− 2t

x′ (t) = 2, y′ (t) = 0, z′ (t) = −2 and

(

dxdt

)2+(

dydt

)2+(

dzdt

)2=√

8

C (x + yz) ds =∫ 10 (7− 2t)

(√8)

dt = 12√

2

(14) F =[

x4, xy]

R = (x, y) |0 ≤ x ≤ 1, 0 ≤ y ≤ x∫

Γx4 dx + xy dy =

ΓF • dr =

∫ 1

0

∫ x

0

[

∂x(xy)− ∂

∂y

(

x4)

]

dydx =

∫ 1

0

∫ x

0y dydx

=

∫ 1

0

[

y2

2

]x

0

dx =1

2

∫ 1

0x2 dx =

1

6

(15) F =[

y2, 3xy]

130

Page 133: AMA SolutionsManual

CHAPTER 6. VECTOR CALCULUS

R = (r, θ) |1 ≤ r ≤ 2, 0 ≤ θ ≤ π∫

Γy2 dx + 3xy dy =

ΓF • dr =

∫ π

0

∫ 2

1

[

∂x(3xy)− ∂

∂y

(

y2)

]

rdrdθ =

∫ π

0

∫ 2

1y rdrdθ

=

∫ π

0

∫ 2

1r2 sin θ drdθ =

7

3

∫ π

0sin θ dθ =

14

3

(16) (a) Let x = t, y = t, z = 0, 0 ≤ t ≤ 1, then dxdt = 1, dy

dt = 1, dzdt = 0.

ABf (x, y, z) ds =

∫ 1

0(t) (t)2

12 + 12 + 02 dt =√

2

∫ 1

0t3 dt =

√2

4

(b) Let x = 2t, y = 3t, z = 6t, 0 ≤ t ≤ 1, then dxdt = 2, dy

dt = 3, dzdt = 6.

AB f (x, y, z) ds =∫ 10

[

(2t) (3t) + (6t)2]√

22 + 32 + 62 dt = 294∫ 10 t2 dt = 98

(17) (a) Let x = cos t, y = sin t, where 0 ≤ t ≤ π2 .

Cxyds =

∫ π2

0cos t sin t

(

d (cos t)

dt

)2

+

(

d (sin t)

dt

)2

dt

=

∫ π2

0cos t sin tdt =

1

2

∫ π2

0sin 2tdt

=

[

−1

4cos 2t

]π/2

0

=1

2

(b) Let x = t, y = 2t, z = −t where 0 ≤ t ≤ 1.

C(xy + y + z) ds

=

∫ 1

0(t× 2t + 2t− t)

(

d (t)

dt

)2

+

(

d (2t)

dt

)2

+

(

d (−t)

dt

)2

dt

=

∫ 1

0

(

2t2 + t)√

6dt

=√

6

∫ 1

0

(

2t2 + t)

dt =7

6

√6

(c) Let x = cos t, y = sin t, z = t where 0 ≤ t ≤ 2π.

Cyds =

∫ 2π

0sin t

(

d (cos t)

dt

)2

+

(

d (sin t)

dt

)2

+

(

d (t)

dt

)2

dt

=

∫ 2π

0

√2 sin tdt = 0

131

Page 134: AMA SolutionsManual

CHAPTER 6. VECTOR CALCULUS

(18) (a) Let x = cos θ, y = sin θ, 0 ≤ θ ≤ π, then dxdθ = − sin θ, dy

dθ = cos θ.

Cf (x, y) ds =

∫ π

0(cos θ + sin θ)

(− sin θ)2 + (cos θ)2 dθ

=

∫ π

0(cos θ + sin θ) dθ = [sin θ − cos θ]π0 = 2

(b)

Cf (x, y) ds =

∫ π

0(cos θ)2 sin θ

(− sin θ)2 + (cos θ)2 dθ =

∫ π

0cos2 θ sin θ dθ

=

[

−1

3cos3 θ

0

=2

3

(19) x = t cos t, y = t sin t, z = t, 0 ≤ t ≤ 1, then dxdt = cos t− t sin t, dy

dt = sin t + t cos t, dzdt = 1

and

(

dxdt

)2+(

dydt

)2+(

dzdt

)2=√

(cos t− t sin t)2 + (sin t + t cos t)2 + 12 =√

t2 + 2.

(a)∫

C f (x, y, z) ds =∫ 10 (t)

√t2 + 2 dt =

[

13

(

t2 + 2)3/2

]1

0=√

3− 23

√2

(b)

Cf (x, y, z) ds =

∫ 1

0

t2 + 2 dt =

[

t

2

t2 + 2 + ln∣

∣t +√

t2 + 2∣

]1

0

= ln(√

3 + 1)

− ln√

2 +1

2

√3

= 1.5245 =1

2

[

ln(√

3 + 2)

+√

3]

(20) (a) x = cos t + t sin t, y = sin t − t cos t, 0 ≤ t ≤ 2π, then dxdt = t cos t, dy

dt = t sin t and√

(

dxdt

)2+(

dydt

)2= t.

Arc length =∫ 2π0

(

dxdt

)2+(

dydt

)2dt =

∫ 2π0 t dt =

[

12 t2]2π

0= 2π2

(b) x = θ − sin θ, y = 1 − cos θ, 0 ≤ θ ≤ π, then dxdθ = 1 − cos θ, dy

dt = sin θ and√

(

dxdθ

)2+(

dydθ

)2=√

2− 2 cos θ.

Arc length =∫ π0

√2− 2 cos θ dθ =

∫ π0 2 sin θ

2 dθ =[

−4 cos θ2

0= 4

132

Page 135: AMA SolutionsManual

CHAPTER 6. VECTOR CALCULUS

(21) (a)

curlF

=

i j k∂∂x

∂∂y

∂∂z

3 + 2xy x2 − 3y2 0

=

[

0− ∂

∂z

(

x2 − 3y2)

,∂

∂z(3 + 2xy)− 0,

∂x

(

x2 − 3y2)

− ∂

∂y(3 + 2xy)

]

= [0, 0, 0] = 0

Since curlF = 0, the F is conservative.

(b) Since F is conservative, then the potential function f exists.

fx = 3 + 2xy =⇒ f (x, y) =∫

(3 + 2xy) dx = 3x + x2y + g (y)

fy = ∂∂y

[

3x + x2y + g (y)]

= x2 + g′ (y) = x2 − 3y2

So, g′ (y) = −3y2 =⇒ g (y) =∫ (

−3y2)

dy = −y3 + C.

Thus, f (x, y) = 3x + x2y − y3 + C.

r (π) = [eπ sinπ, eπ cos π] = [0,−eπ]

r (0) =[

e0 sin 0, e0 cos 0]

= [0, 1]

By the Foundamental Theorem of Line Integrals,

ΓF • dr =f (0,−eπ)− f (0, 1) =

[

− (−eπ)3]

−[

− (1)3]

= e3π + 1.

(22) Let f (x, y, z) = 3x− 6yz, g (x, y, z) = 2y + 3xz and h (x, y, z) = 1− 4xz2.

(a) Let x = t, y = t2, z = t3, 0 ≤ t ≤ 1, then dxdt = 1, dy

dt = 2t, dzdt = 3t2.

fdx

dt+ g

dy

dt+ h

dz

dt= (3x− 6yz) + 2t (2y + 3xz) + 3t2

(

1− 4xz2)

=(

3t− 6t5)

+ 2t(

2t2 + 3t4)

+ 3t2(

1− 4t7)

= −12t9 + 4t3 + 3t2 + 3t

y

ABF • dr =

∫ 10

(

−12t9 + 4t3 + 3t2 + 3t)

dt = 2310

(b) Let x = t, y = t, z = t, 0 ≤ t ≤ 1, then dxdt = dy

dt = dzdt = 1.

f dxdt + g dy

dt + hdzdt =

(

3t− 6t2)

+(

2t + 3t2)

+(

1− 4t3)

= −4t3 − 3t2 + 5t + 1∫

y

ABF • dr =

∫ 10

(

−4t3 − 3t2 + 5t + 1)

dt = 32

(c) From A to C, let x = 0, y = 0, z = t, 0 ≤ t ≤ 1, then dxdt = dy

dt = 0, dzdt = 1.

f dxdt + g dy

dt + hdzdt = 0 + 0 + (1) = 1

133

Page 136: AMA SolutionsManual

CHAPTER 6. VECTOR CALCULUS

y

ACF • dr =

∫ 10 dt = 1

From C to B, let x = s, y = s, z = 0, 0 ≤ s ≤ 1, then dxds = dy

ds = 1, dzds = 0.

f dxds + g dy

ds + hdzds = (3s− 6s) + (2s + 3s) + 0 = 2s

y

CBF • dr =

∫ 10 2s ds = 1

y

ABF • dr =

y

ACF • dr +

y

CBF • dr = 1 + 1 = 2

(23) Let x (t) = t and y (t) = t3, −2 ≤ t ≤ 3. Then r (t) =[

t, t3]

, F (r (t)) =[

(t)(

t3)

, t3 − t]

=[

t4, t3 − t]

and r′ (t) =[

1, 3t2]

.

Work done =∫

C F • dr =∫ 3−2

[

t4, t3 − t]

•[

1, 3t2]

dt =∫ 3−2

(

t4 + 3t5 − 3t3)

dt = 13554

(24) r (t) = [cos t, sin t], 0 ≤ t ≤ π, then r′ (t) = [− sin t, cos t].

F (r (t)) =[

− (sin t)2 , (cos t) (sin t)]

=[

− sin2 t, sin t cos t]

Work done =

∫ π

0

[

− sin2 t, sin t cos t]

• [− sin t, cos t] dt =

∫ π

0

(

sin3 t + sin t cos2 t)

dt

=

∫ π

0sin t dt = 2

(25) (a) Let x = 1 + t, y = 2− t and 0 ≤ t ≤ 1, then dxdt = 1 and dy

dt = −1.

F (r (t)) = 〈2 (1 + t)− (2− t) + 4, 3 (1 + t) + 5 (2− t)− 6〉 = 〈3t + 4, 7− 2t〉r′ (t) = 〈1,−1〉∫

Γ F • dr = F (r (t)) • r′ (t) dt =∫ 10 [(3t + 4)− (7− 2t)] dt =

∫ 10 (5t− 3) dt = −1

2

(b) F (r (t)) =⟨

(

t2) (

t3)

,(

t2)2

, t3⟩

=⟨

t5, t4, t3⟩

r′ (t) =⟨

1, 2t, 3t2⟩

Γ F • dr =∫ 10

[(

t5)

(1) +(

t4)

(2t) +(

t3) (

3t2)]

dt =∫ 10 6t5 dt = 1

(c) From (0, 0) to (1, 0), let x = t, y = 0 and 0 ≤ t ≤ 1, then dxdt = 1and dy

dt = 0.

Then F (r (t)) = 〈0, 0〉 and∫

Γ1F•dr = 0.

From (1, 0) to (1, 1), let x = 1, y = s and 0 ≤ s ≤ 1, then dxds = 0 and dy

ds = 1.

Then F (r (s)) = 〈6s, 2s〉, r′ (s) = 〈0, 1〉 and∫

Γ2F • dr =

∫ 10 2s ds = 1.

From (1, 1) to (0, 1), let x = 1− u, y = 1 and 0 ≤ u ≤ 1, then dxdu = −1, dy

du = 0.

F (r (u)) =⟨

6 (1− u)2 , 2⟩

, r′ (u) = 〈−1, 0〉 and∫

Γ3F • dr =

∫ 10 −6u2 du = −2.

From (0, 1) to (0, 0), let x = 0, y = 1− v and 0 ≤ v ≤ 1, then dxdv = 0 and dy

dv = −1.

F (r (v)) = 〈0, 2 (1− v)〉, r′ (v) = 〈0,−1〉 and∫

Γ4F • dr =

∫ 10 2 (v − 1) dv = −1.

Therefore,∫

Γ F • dr =∫

Γ1+∫

Γ2+∫

Γ3+∫

Γ4= 0 + 1− 2− 1 = −2.

134

Page 137: AMA SolutionsManual

CHAPTER 6. VECTOR CALCULUS

(26) (a) Let x = t, y = t and z = t, where 0 ≤ t ≤ 1.

CF•dr =

C

[(

x2 − y)

i+(

y2 − z)

j+(

z2 − x)

k]

• [dxi+dyj+dzk]

=

C

(

x2 − y)

dx+(

y2 − z)

dy+(

z2 − x)

dz

=

∫ 1

0

(

t2 − t)

dt+(

t2 − t)

dt+(

t2 − t)

dt

= 3

∫ 1

0

(

t2 − t)

dt = −1

2

(b) Let x = t, y = t2 and z = t3, where 0 ≤ t ≤ 1.

CF•dr =

C

[(

x2 − y)

i+(

y2 − z)

j+(

z2 − x)

k]

• [dxi+dyj+dzk]

=

C

(

x2 − y)

dx+(

y2 − z)

dy+(

z2 − x)

dz

=

∫ 1

0

(

t2 − t2)

dt+(

(

t2)2 − t3

)

dt2+(

(

t3)2 − t

)

dt3

=

∫ 1

0

(

t2 − t2)

dt+2t(

t4 − t3)

dt+3t2(

t6 − t)

dt

=

∫ 1

0

(

2t5 − 2t4 + 3t8 − 3t3)

dt = −29

60

(27) (a) Let x = cos t and y = sin t, where 0 ≤ t ≤ 2π,

C−ydx + xdy =

∫ 1

0− sin td (cos t) + cos td (sin t)

=

∫ 2π

0

(

sin2 t + cos2 t)

dt = 2π.

(b) Let x = 1− t and y = t, where 0 ≤ t ≤ 1

C(x + y) dx +

(

x2 + y2)

dy =

∫ 1

0(1) d (1− t) +

∫ 1

0

(

(1− t)2 + t2)

dt

= −1 +

∫ 1

0

(

2t2 + 1− 2t)

dt = −1

3.

135

Page 138: AMA SolutionsManual

CHAPTER 6. VECTOR CALCULUS

(28) Since

∇× F =

i j k∂∂x

∂∂y

∂∂z

y sin z x sin z xy cos z

=

(

∂y(xy cos z)− ∂

∂z(x sin z)

)

i+

(

∂x(xy cos z)− ∂

∂z(y sin z)

)

j

+

(

∂x(x sin z)− ∂

∂y(y sin z)

)

k

= 0

Therefore, F is conservative and there exists a function φ such that ∇φ = F. In this case,

φ = xy sin z+ constant and

∫ (1,2,3)

(0,0,0)[(xy sin z) i + (x sin z) j + (xy cos z)k] • dr

= xy sin z|(1,2,3)(0,0,0) = (1) (2) sin (3)− (0) (0) sin 0 = 2 sin 3

(29) Since cos2 t + sin2 t = 1, so, let x− 1 = cos t and y = sin t, then x = cos t− 1, y = sin t.

(30) (a) F = 〈3y, 5x〉, P = 3y, Q = 5x and R = (r, θ) |0 ≤ r ≤ 1, 0 ≤ θ ≤ 2π.

∂R F • dr =∫ ∫

R

(

∂Q∂x − ∂P

∂y

)

dA =∫ 2π0

∫ 10 (5− 3) r dr dθ = 2 [θ]2π

0

[

12r2]1

0= 2π

(b) F =⟨

y2, x2⟩

, P = y2, Q = x2 and R = (x, y) |0 ≤ x ≤ 1, 0 ≤ y ≤ 1.

∂RF • dr =

∫ 1

0

∫ 1

0(2x− 2y) dx dy =

∫ 1

0

[

x2 − 2xy]1

0dy

=

∫ 1

0(1− 2y) dy =

[

y − y2]1

0= 0

(c) F = 〈xy, x− y〉, P = xy, Q = x− y, and R = (x, y) |0 ≤ x ≤ 1, 1− x ≤ y ≤ 1.

∂R F • dr =∫ 10

∫ 11−x (1− x) dy dx =

∫ 10

(

x− x2)

dx =[

12x2 − 1

3x3]1

0= 1

6

136

Page 139: AMA SolutionsManual

CHAPTER 6. VECTOR CALCULUS

(31) (a) Let x = a cos t and y = a sin t, where 0 ≤ t ≤ 2π.

C

(

−x2y)

dx +(

xy2)

dy

=

∫ 2π

0

(

−a2 cos2 t× a sin t)

d (a cos t) +(

a2 cos t× a sin2 t)

d (a sin t)

=

∫ 2π

0

(

a4 cos2 t sin2 t + a4 cos2 t sin2 t)

dt

= 2a4

∫ 2π

0

(

cos2 t sin2 t)

dt = 2a4

∫ 2π

0

(

sin 2t

2

)2

dt

=1

2a4

∫ 2π

0sin2 2tdt

=1

2a4

∫ 2π

0

(

1− cos 4t

2

)

dt =1

2πa4

and

∫∫

x2+y2≤a2

(

∂(

xy2)

∂x− ∂

(

−x2y)

∂y

)

dxdy =

∫∫

x2+y2≤a2

(

y2 + x2)

dxdy

Use polar coordinates, x = r cos θ and y = r sin θ, we have

∫∫

x2+y2≤a2

(

∂(

xy2)

∂x− ∂

(

−x2y)

∂y

)

dxdy

=

∫ 2π

0

∫ a

0

(

r2 cos2 θ + r2 sin2 θ)

rdrdθ

=

∫ 2π

0

∫ a

0r3drdθ =

∫ 2π

0

1

4a4dθ =

1

2πa4

(b)

x

y(0,1)

(1,0)

(-1,0)

(0,-1)

137

Page 140: AMA SolutionsManual

CHAPTER 6. VECTOR CALCULUS

On C1– the line x + y = 1, x = 1− t and y = t, where 0 ≤ t ≤ 1.

C1

(−y) dx + (x) dy =

∫ 1

0− (t) d (1− t) + (1− t) dt

=

∫ 1

0tdt + (1− t) dt =

∫ 1

0dt = 1

On C2– the line −x + y = 1, x = −t and y = 1− t, where 0 ≤ t ≤ 1.

C2

(−y) dx + (x) dy =

∫ 1

0− (1− t) d (−t) + (−t) d (1− t)

=

∫ 1

0(1− t) dt + tdt =

∫ 1

0dt = 1

On C3– the line −x− y = 1, x = t− 1 and y = −t, where 0 ≤ t ≤ 1.

C3

(−y) dx + (x) dy =

∫ 1

0− (−t) d (t− 1) + (t− 1) d (−t)

=

∫ 1

0tdt− (t− 1) dt =

∫ 1

0dt = 1

On C4– the line x− y = 1, x = t and y = t− 1, where 0 ≤ t ≤ 1.

C4

(−y) dx + (x) dy =

∫ 1

0− (t− 1) dt + td (t− 1)

=

∫ 1

0− (t− 1) dt + tdt =

∫ 1

0dt = 1

Hence,

C(−y) dx + (x) dy =

C1

+

C2

+

C3

+

C4

(−y) dx + (x) dy = 1 + 1 + 1 + 1 = 4

and

∫∫

square

(

∂ (x)

∂x− ∂ (−y)

∂y

)

dxdy =

∫∫

square2dxdy = 2× area of square

= 2×√

2×√

2 = 4

Therefore,

C(−y) dx + (x) dy =

∫∫

square

(

∂ (x)

∂x− ∂ (−y)

∂y

)

dxdy.

138

Page 141: AMA SolutionsManual

CHAPTER 6. VECTOR CALCULUS

and the Green’s Theorem is true in this case.

(32) By Green’s Theorem,

xy2dx +(

x2y + 2x)

dy =

∫∫

Region

(

∂(

x2y + 2x)

∂x− ∂

(

xy2)

∂y

)

dxdy

=

∫∫

Region2dxdy

= 2× area of the region

(33) (a)

X = cos u sin v, Y = sinu sin v, Z = cos v

Xu = − sinu sin v, Yu = cos u sin v, Zu = 0

Xv = cos u cos v, Yv = sinu cos v, Zv = − sin v

and

E = (Xu)2 + (Yu)2 + (Zu)2 = sin2 v

G = (Xv)2 + (Yv)

2 + (Zv)2 = 1

F = XuXv + YuYv + ZuZv = 0

Sf (x, y, z) dxdydz =

∫ π/2

0

∫ π/2

0cos u sin v sinu sin v cos v

sin2 vdudv

=

∫ π/2

0cos u sinudu

∫ π/2

0cos v sin3 vdv

=1

8

(b)

O

x

y

z

139

Page 142: AMA SolutionsManual

CHAPTER 6. VECTOR CALCULUS

On S1, the plane x = 0

S1

f (x, y, z) dS =

∫ 1

0

∫ 1−z

0(y + z) dydz =

1

3

On S2, the plane y = 0

S2

f (x, y, z) dS =

∫ 1

0

∫ 2

0(z) dxdz = 1

On S3, the plane x = 2

S3

f (x, y, z) dS =

∫ 1

0

∫ 1−z

0(y + z) dydz =

1

3

On S4, the plane z = 0

S4

f (x, y, z) dS =

∫ 1

0

∫ 2

0ydxdy = 1

On S5, the plane y + z = 1

S5

f (x, y, z) dS =

∫ 1

0

∫ 2

0(1)

1 + (0)2 + (−1)2dxdy = 2√

2

Hence,∫

Sf (x, y, z) dS = 1 +

1

3+ 1 +

1

3+ 2√

2 =8

3+ 2√

2

(34)

z =√

2− x2 − y2, zx =−x

2− x2 − y2, zy =

−y√

2− x2 − y2.

140

Page 143: AMA SolutionsManual

CHAPTER 6. VECTOR CALCULUS

Let x = r cos θ, y = r sin θ, 0 ≤ r ≤ 1 and 0 ≤ θ ≤ 2π.

∫∫

x2+y2≤1

1 + z2x + z2

ydxdy

=

∫∫

x2+y2≤1

1 +x2

2− x2 − y2+

y2

2− x2 − y2dxdy

=

∫ 2π

0

∫ 1

0r

1 +r2

2− r2drdθ

=

∫ 2π

0

∫ 1

0r

2

2− r2drdθ

= −∫ 2π

0

[√2√

2− r2]1

0dθ

=

∫ 2π

0

(

2−√

2)

dθ = 2π(

2−√

2)

(35)

Let X = a cos u, Y = a sinu, Z = v, where 0 ≤ u ≤ 2π and 0 ≤ v ≤ 10− a cos u− 2a sinu.

Then Xu = −a sinu, Yu = a cos u, Zu = 0 and Xv = 0, Yv = 0, Zv = 1.

E = (−a sinu)2 + (a cos u)2 + (0)2 = a2

G = (0)2 + (0)2 + (1)2 = 1

F = (−a sinu) (0) + (a cos u) (0) + (0) (1) = 0

and√

EG− F 2 =√

a2 cos2 u + a2 sin2 u = a

A =

∫ 2π

0

∫ 10−a cos u−2a sin u

0advdu = a

∫ 2π

0(10− a cos u− 2a sinu) du = 20πa

141

Page 144: AMA SolutionsManual

CHAPTER 6. VECTOR CALCULUS

(36) z = 2x, zx = 2, zy = 0.

∫∫

x2+y2≤1

1 + z2x + z2

ydxdy =

∫∫

x2+y2≤1

1 + (2)2 + (0)2dxdy

=√

5

∫∫

x2+y2≤1dxdy =

√5π

(37)

z = 4−x2−y2, zx = −2x, zy = −2y. Let x = r cos θ, y = r sin θ, 1 ≤ r ≤ 2 and 0 ≤ θ ≤ 2π

∫∫

1≤x2+y2≤4

1 + z2x + z2

ydxdy =

∫∫

1≤x2+y2≤4

1 + 4x2 + 4y2dxdy

=

∫ 2π

0

∫ 2

1r√

1 + 4r2drdθ

=

∫ 2π

0

[

1

12

(

1 + 4r2)3/2

]2

1

=

∫ 2π

0

(

17

12

√17− 5

12

√5

)

dθ =1

6π(

17√

17− 5√

5)

(38) (a) Let f (x, y, z) = 1, z = φ (x, y) = 1− x− y, and ∂z∂x = ∂z

∂y = −1.

R is the region inside x2 + y2 = a2 or R = (r, θ) | 0 ≤ r ≤ a, 0 ≤ θ ≤ 2π.

S =

∫ ∫

Sf (x, y, z) dS =

∫ ∫

Rf (x, y, φ (x, y))

(

∂z

∂x

)2

+

(

∂z

∂y

)2

+ 1 dx dy

=

∫ 2π

0

∫ a

0

(√3)

r dr dθ =√

3πa2

(b) Let f (x, y, z) = 1, z = φ (x, y) = 23

(

x3/2 + y3/2)

, and ∂z∂x =

√x, ∂z

∂y =√

y.

142

Page 145: AMA SolutionsManual

CHAPTER 6. VECTOR CALCULUS

R = (x, y) | 0 ≤ x ≤ 1, 0 ≤ y ≤ 1

S =

∫ ∫

Sf (x, y, z) dS =

∫ 1

0

∫ 1

0

(√x)2

+ (√

y)2 + 1 dx dy

=

∫ 1

0

[

2

3(x + y + 1)3/2

]1

0

dy =2

3

∫ 1

0

[

(y + 2)3/2 − (y + 1)3/2]

dy

=2

3

[

2

5(y + 2)5/2 − 2

5(y + 1)5/2

]1

0

=4

15

[(

35/2 − 25/2)

−(

25/2 − 15/2)]

=4

15

(

35/2 − 27/2 + 1)

= 1.4066

(39) (a) σ (x, y, z) = z, let z = φ (x, y) =√

4− x2 − y2, and ∂z∂x = −x

z , ∂z∂y = −y

z .

R = (r, θ) | 0 ≤ r ≤ 2, 0 ≤ θ ≤ 2π

S =

∫ ∫

Sσ (x, y, z) dS =

∫ 2π

0

∫ 2

0(z)

(

−x

z

)2+(

−y

z

)2+ 1r dr dθ

=

∫ 2π

0

∫ 2

0

x2 + y2 + z2r dr dθ =

∫ 2π

0dθ

∫ 2

02r dr = [θ]2π

0

[

r2]2

0= 8π

(b) Find the equation for the area, a plane, within ∆ABC:−−→AB =

−−→OB −−→OA = 〈0, 1, 0〉 − 〈1, 0, 0〉 = 〈−1, 1, 0〉 and

−→AC = 〈−1, 0, 1〉.

normal vector = n =−−→AB ×−→AC =

i j k

−1 1 0

−1 0 1

= 〈1, 1, 1〉

Equation for the plane with normal vector n and passing through A (1, 0, 0):

(x− 1) + (y − 0) + (z − 0) = 0 =⇒ x + y + z = 1

σ (x, y, z) = xyz, let z = φ (x, y) = 1− x− y, and ∂z∂x = ∂z

∂y = −1.

R = (x, y) | 0 ≤ x ≤ 1, 0 ≤ y ≤ 1− x

S =

∫ ∫

Sσ (x, y, z) dS =

∫ 1

0

∫ 1−x

0(xyz)

(−1)2 + (−1)2 + 1 dy dx

=√

3

∫ 1

0

∫ 1−x

0xy (1− x− y) dy dx =

√3

∫ 1

0

[

1

2xy2 − 1

2x2y2 − 1

3xy3

]1−x

0

dx

=√

3

∫ 1

0

(

−1

6x4 +

1

2x3 − 1

2x2 +

1

6x

)

dx =

√3

120

(c) σ (x, y, z) = z2

On the surface S1, z = 0 and σ (x, y, z) = 0, thus∫ ∫

S1σ (x, y, z) dS1 = 0.

On the surface S2, z = 1 and σ (x, y, z) = 1 and S2 = (x, y) |0 ≤ x ≤ 1, 0 ≤ y ≤ 1,thus

∫ ∫

S2σ (x, y, z) dS2 =

∫ ∫

S2dS2 =

∫ 10

∫ 10 dx dy = 1.

143

Page 146: AMA SolutionsManual

CHAPTER 6. VECTOR CALCULUS

On the surface S3 and S4, x = 0 and 1, and S3 = S4 = (y, z) |0 ≤ y ≤ 1, 0 ≤ z ≤ 1,thus

∫ ∫

S3σ (x, y, z) dS3 =

∫ ∫

S4σ (x, y, z) dS4 =

∫ 10

∫ 10 z2 dy dz = 1

3 .

On the surface S5 and S6, y = 0 and 1, and S5 = S6 = (x, z) |0 ≤ x ≤ 1, 0 ≤ z ≤ 1,thus

∫ ∫

S5σ (x, y, z) dS5 =

∫ ∫

S6σ (x, y, z) dS6 =

∫ 10

∫ 10 z2 dx dz = 1

3 .

Therefore,∫ ∫

S σ (x, y, z) dS =∫ ∫

S1+∫ ∫

S2+... +

∫ ∫

S6= 7

3 .

(40) (a) The equation for the plane that including ∆ABC:

unit normal vector = 〈6,3,2〉√62+32+22

=⟨

67 , 3

7 , 27

6 (x− 1) + 3 (y − 0) + 2 (z − 0) = 0 =⇒ 6x + 3y + 2z = 6

Let z = φ (x, y) = 3− 3x− 32y, then F = 〈xy, y, zx〉 =

xy, y, 3x− 3x2 − 32xy

Using the formula

∫ ∫

SF • n dS =

∫ ∫

R

(

−P∂z

∂x−Q

∂z

∂y+ R

)

dx dy

F • n =⟨

xy, y, 3x− 3x2 − 32xy

•⟨

67 , 3

7 , 27

= 67x + 3

7y + 37xy − 6

7x2

R = (x, y) | 0 ≤ x ≤ 1, 0 ≤ y ≤ 2− 2x

dS = 1|n • k| dx dy = 7

2 dx dy

∫ ∫

SF • n dS =

∫ 1

0

∫ 2−2x

0

(

6

7x +

3

7y +

3

7xy − 6

7x2

)

7

2dy dx =

7

4

=

∫ 1

0

[

3xy − 3x2y − 3

4xy2

]2−2x

0

dx

or using the formula

∫∫

SF • n dS =

∫∫

DF (r (u, v)) • (ru × rv) dudv

Let r (x, y) =⟨

x, y, 3− 3x− 32y⟩

and D = (x, y) |0 ≤ x ≤ 1, 0 ≤ y ≤ 2− 2x, then

rx = 〈1, 0,−3〉 and ry =⟨

0, 1,−32

.

rx × ry =

i j k

1 0 −3

0 1 −32

=⟨

3, 32 , 1⟩

144

Page 147: AMA SolutionsManual

CHAPTER 6. VECTOR CALCULUS

F (r (x, y)) =⟨

xy, y, 3x− 3x2 − 32xy

∫∫

DF (r (x, y)) • (rx × ry) dx dy

=

∫ 1

0

∫ 2−2x

0

xy, y, 3x− 3x2 − 3

2xy

•⟨

3,3

2, 1

dy dx

=

∫ 1

0

∫ 2−2x

0

(

3

2xy +

3

2y + 3x− 3x2

)

dy dx

=

∫ 1

0

(

9x3 − 15x2 + 3x + 3)

dx =7

4

(b) Let f (x, y, z) = x2 + y2 + z2 − a2 and z = φ (x, y) =√

a2 − x2 − y2.

F = 〈x, y, z〉 =⟨

x, y,√

a2 − x2 − y2⟩

Using the formula

∫ ∫

SF • n dS =

∫ ∫

R

(

−P∂z

∂x−Q

∂z

∂y+ R

)

dx dy

n = ∇f‖∇f‖ = 〈2x,2y,2z〉

2√

x2+y2+z2=

2x,2y,2√

a2−x2−y2⟩

2a =⟨

xa , y

a , 1a

a2 − x2 − y2⟩

F • n = x2

a + y2

a + 1a

(

a2 − x2 − y2)

= a

R = (r, θ) | 0 ≤ r ≤ a, 0 ≤ θ ≤ 2π

dS = 1|n • k|r dr dθ =

a√

a2 − x2 − y2r dr dθ =

ar√a2 − r2

dr dθ

∫ ∫

S F • n dS =∫ 2π0

∫ a0 (a)

ar√a2 − r2

dr dθ = a2∫ 2π0

[

−√

a2 − r2]a

0dθ = a3

∫ 2π0 dθ =

2πa3

or using the formula

∫ ∫

SF • n dS =

∫ ∫

DF (r (u, v)) • (ru × rv) du dv

Let

r (φ, θ) = 〈a sinφ cos θ, a sinφ sin θ, a cos φ〉 and D =

(φ, θ) |0 ≤ φ ≤ π

2, 0 ≤ θ ≤ 2π

then rφ = 〈a cos φ cos θ, a cos φ sin θ,−a sinφ〉 and rθ = 〈−a sinφ sin θ, a sinφ cos θ, 0〉.

145

Page 148: AMA SolutionsManual

CHAPTER 6. VECTOR CALCULUS

rφ × rθ =

i j k

a cos φ cos θ a cos φ sin θ −a sinφ

−a sinφ sin θ a sinφ cos θ 0

=⟨

a2 sin2 φ cos θ, a2 sin2 φ sin θ, a2 sinφ cos φ⟩

F (r (φ, θ)) = 〈a sinφ cos θ, a sinφ sin θ, a cos φ〉F (r (φ, θ)) • (rφ × rθ) = a3 sinφ

Therefore,

∫∫

DF (r (φ, θ)) • (rφ × rθ) dφ dθ =

∫ 2π

0

∫ π/2

0a3 sinφdφ dθ = 2πa3

(c) On the surface S1, z = 0, F = 〈x, 2y, 0〉 and n = 〈0, 0,−1〉, thus

∫∫

S1

F • ndS =

∫∫

S1

(0) dS = 0.

On the surface S2, z = 1, F = 〈x, 2y, 3〉, n = 〈0, 0, 1〉, dS = 1|n • k| dx dy = dx dy,

and S2 = (x, y) |0 ≤ x ≤ 1, 0 ≤ y ≤ 1, thus

∫∫

S2

F • ndS =

∫∫

S2

(3) dS = 3

∫ 1

0

∫ 1

0dx dy = 3.

On the surface S3, x = 0, F = 〈0, 2y, 3z〉, n = 〈−1, 0, 0〉, thus

∫∫

S3

F • ndS =

∫∫

S3

(0) dS = 0.

On the surface S4, x = 1, F = 〈1, 2y, 3z〉, n = 〈1, 0, 0〉, dS4 = 1|n • i| dx dy = dy dz,

and S4 = (y, z) |0 ≤ y ≤ 1, 0 ≤ z ≤ 1, thus

∫∫

S4

F • ndS =

∫∫

S4

(1) dS =

∫ 1

0

∫ 1

0dy dz = 1.

On the surface S5, y = 0, F = 〈x, 0, 3z〉, n = 〈0,−1, 0〉, thus

∫∫

S5

F • ndS =

∫∫

S5

(0) dS = 0.

On the surface S6, y = 1, F = 〈x, 2, 3z〉, n = 〈0, 1, 0〉, dS6 = 1|n•j| dx dy = dx dy, and

146

Page 149: AMA SolutionsManual

CHAPTER 6. VECTOR CALCULUS

S6 = (x, z) |0 ≤ x ≤ 1, 0 ≤ z ≤ 1, thus

∫∫

S6

F • ndS =

∫∫

S6

(2) dS = 2

∫ 1

0

∫ 1

0dx dz = 2.

Therefore,∫∫

SF • n dS =

∫∫

S1

+

∫∫

S2

+... +

∫∫

S6

= 6.

(41) For the top end-face of the cylinder x2 + y2 = 4, 0 ≤ z ≤ 3, n = 〈0, 0, 1〉. Using the

formula∫∫

SF • n dS =

∫∫

R

(

−P∂z

∂x−Q

∂z

∂y+ R

)

dx dy

F =⟨

4x,−2y2, z2x2⟩

=⟨

4x,−2y2, 9x2⟩

F • n = 9x2

R = (r, θ) | 0 ≤ r ≤ 2, 0 ≤ θ ≤ 2π

dS = 1|n • k|r dr dθ = r dr dθ

∫∫

SF • n dS =

∫ 2π

0

∫ 2

09 (r cos θ)2 r dr dθ = 9

∫ 2π

0

∫ 2

0r3 cos2 θ dr dθ

= 9

∫ 2π

0

1 + cos 2θ

2dθ

∫ 2

0r3 dr

= 9

[

θ

2+

1

4sin 2θ

]2π

0

[

1

4r4

]2

0

= 36π

or using the formula

∫∫

SF • n dS =

∫ ∫

DF (r (u, v)) • (ru × rv) du dv

Let

r (r, θ) = 〈r cos θ, r sin θ, 3〉 and D = (r, θ) |0 ≤ r ≤ 2, 0 ≤ θ ≤ 2π

then rφ = 〈cos θ, sin θ, 0〉 and rθ = 〈−r sin θ, r cos θ, 0〉.

rr × rθ =

i j k

cos θ sin θ 0

−r sin θ r cos θ 0

= 〈0, 0, r〉

F (r (r, θ)) =⟨

4r cos θ,−2r2 sin2 θ, 9r2 cos2 θ⟩

F (r (r, θ)) • (rr × rθ) = 9r3 cos2 θ∫∫

D F (r (r, θ)) • (rr × rθ) dr dθ =∫ 2π0

∫ 20 9r3 cos2 θ dr dθ = 36π

147

Page 150: AMA SolutionsManual

CHAPTER 6. VECTOR CALCULUS

(42) (a)

O

x

y

z

Let f = x + 2y + z − 3 = 0, then n = ∇f =i + 2j + k

(1)2 + (2)2 + (1)2= 1√

6(i + 2j + k)

SF • ndS =

S(−i + 2j + 3k) •

(

1√6

(i + 2j + k)

)

dS =

S

√6dS

=√

6

∫ 3

0

∫ 3−x2

0

1 + (−1)2 + (−2)2dydx

=27

2

(b)

O

x

y

z

Let f = ex − y = 0, then n = ∇f =exi− j

(ex)2 + (1)2=

exi− j√e2x + 1

SF • ndS =

S(−2i + 2yj + zk) •

(

exi− j√e2x + 1

)

dS =

S

(−2ex − 2y)√e2x + 1

dS

Let Z = v, X = u, Y = eu, where 0 ≤ v ≤ 1 and 0 ≤ u ≤ ln 2.

Note that Xu = 1, Yu = eu, Zu = 0 and Xv = 0, Yv = 0, Zv = 1

E = (Xu)2 + (Yu)2 + (Zu)2 = (1)2 + (eu)2 + 02 = 1 + e2u

G = (Xv)2 + (Yv)

2 + (Zv)2 = (0)2 + (0)2 + (1)2 = 1

F = XuXv + YuYv + ZuZv = (1) (0) + (eu) (0) + (0) (1) = 0

SF • ndS =

∫ 1

0

∫ ln 2

0

(−2eu − 2eu)√e2u + 1

(1 + e2u) (1)− 02dudv

= −4

∫ 1

0

∫ ln 2

0eududv = −4

∫ 1

0dv = −4

148

Page 151: AMA SolutionsManual

CHAPTER 6. VECTOR CALCULUS

(43) (a)

D∇ • Fdxdydz =

D

(

∂xi+

∂yj+

∂zk

)

•(

x3i+y3j+z3k)

dxdydz

=

D

(

∂x3

∂x+

∂y3

∂y+

∂z3

∂z

)

dxdydz = 3

D

(

x2 + y2 + z2)

dxdydz

= 3

∫ π/2

0

∫ 2π

0

∫ 1

0r4 sinφdrdθdφ =

6

On S1– the surface of the hemi-sphere x2 + y2 + z2 = 1 and n = r.

S1

F • ndS =

S1

F • rdS =

S1

(

x3i+y3j+z3k)

• (xi+yj+zk) dS

=

S1

(

x4+y4+z4)

dS

Let X = sin v cos u, Y = sin v sinu, Z = cos v, then√

EG− F 2 =√

sin2 v = sin v.

S1

F • ndS

=

∫ 2π

0

∫ π/2

0

(

(sin v cos u)4 +(sin v sinu)4 + (cos v)4)

sin vdvdu

=

∫ 2π

0

∫ π/2

0

(

sin4 v cos4 u + sin4 v sin4 u + cos4 v)

sin vdvdu

=

∫ 2π

0

∫ π/2

0

(

sin4 v(

cos4 u + sin4 u)

+ cos4 v)

sin vdvdu

= −∫ 2π

0

[

(

cos4 u + sin4 u)

(

1

5cos5 v − 2

3cos3 v + cos v

)

+1

5cos5 v

]π/2

0

du

=

∫ 2π

0

(

8

15

(

cos4 u + sin4 u)

+1

5

)

du

=8

15

∫ 2π

0

(

(

1 + cos 2u

2

)2

+

(

1− cos 2u

2

)2)

du +2

=8

15

∫ 2π

0

(

1 + 2 cos 2u + cos2 2u

4+

1− 2 cos 2u + cos2 2u

4

)

du +2

=4

15

∫ 2π

0

(

1 + cos2 2u)

du +2

=4

15

∫ 2π

0

(

1 +

(

1 + cos 4u

2

))

du +2

=4

15

[

3u

2+

sin 4u

8

]2π

0

+2

5π =

5

149

Page 152: AMA SolutionsManual

CHAPTER 6. VECTOR CALCULUS

On S2– the bottom surface (z = 0) and n = −k

S2

F • ndS =

S2

(

x3i+y3j+z3k)

• (−k) dS = −∫

S2

z3dS = 0

(b)

D∇ • Fdxdydz =

D

(

∂xi+

∂yj+

∂zk

)

•(

2xzi+yj−z2k)

dxdydz

=

D

(

∂2xz

∂x+

∂y

∂y+

∂(

−z2)

∂z

)

dxdydz

=

D(2z + 1− 2z) dxdydz

=

∫∫

x2+4y2≤16

∫ y

0dzdxdy =

∫ ∫

x2+4y2≤16

ydxdy

Let x = 4r cos θ, y = 2r sin θ and |J | = 8r

D∇ • Fdxdydz =

∫ π/2

0

∫ 1

016r2 sin θdrdθ =

16

3

On the plane z = y : n =−j + k√

2

S1

F • ndS =

S1

(

2xzi+yj−z2k)

• −j + k√2

dS =1√2

S1

(

−y − z2)

dS

= − 1√2

∫∫

R1

(

y + y2)

1 + z2x + z2

ydxdy

= − 1√2

∫∫

R1

(

y + y2)√

1 + 1dxdy

= −∫∫

R1

(

y + y2)

dxdy

= −8

∫ π/2

0

∫ 1

0

(

2r2 sin θ + 4r3 sin2 θ)

drdθ

= −2π − 16

3

On the plane z = 0 : n = −k

S2

F • ndS =

S2

(

2xzi+yj−z2k)

• (−k) dS = −∫

S2

z2dS = 0

150

Page 153: AMA SolutionsManual

CHAPTER 6. VECTOR CALCULUS

On the plane x2 + 4y2 = 16 : n =xi + 4yj

x2 + 16y2

S3

F • ndS =

S3

(

2xzi+yj−z2k)

•(

xi + 4yj√

x2 + 16y2

)

dS

=

S4

2x2z + 4y2

x2 + 16y2dS

Let X = 4 cos u, Y = 2 sinu and Z = v Then Xu = −4 sinu, Yu = 2 cos u, Zu = 0,

and Xv = 0, Yv = 0, Zv = 1.

E = X2u + Y 2

u + Z2u = 16 sin2 u + 4 cos2 u, G = 1, F = 0

S3

F • ndS

=

∫ π/2

0

∫ 2 sin u

0

2 (4 cos u)2 (v) + 4 (2 sinu)2√

(4 cos u)2 + 16 (2 sinu)2

16 sin2 u + 4 cos2 udvdu

=

∫ π/2

0

∫ 2 sin u

0

(

16v cos2 u + 8 sin2 u)

dvdu

=

∫ π/2

0

(

32 sin2 u cos2 u + 16 sin3 u)

du

= 2π +32

3

On the plane x = 0 : n = −i

S4

F • ndS =

S2

(

2xzi+yj−z2k)

• (−i) dS = −∫

S4

2xzdS = 0

Hence,

SF • ndS =

S1

F • ndS +

S2

F • ndS +

S3

F • ndS +

S4

F • ndS =16

3

(44) (a) F = 〈x, y, z〉

∇• F =∂

∂x(x) +

∂y(y) +

∂z(z) = 3

151

Page 154: AMA SolutionsManual

CHAPTER 6. VECTOR CALCULUS

D = (ρ, φ, θ) |0 ≤ ρ ≤ a, 0 ≤ φ ≤ π, 0 ≤ θ ≤ 2π∫ ∫

DF • n dS =

∫ ∫ ∫

D(∇ • F) dV =

∫ 2π

0

∫ π

0

∫ a

0(3) ρ2 sinφdρ dφ dθ

= 3 (Volume of a sphere with radius a) = 4πa3

(b) F = 〈x, y, z〉

∇• F =∂

∂x(x) +

∂y(y) +

∂z(z) = 3

D = (x, y, z) |0 ≤ x ≤ 1, 0 ≤ y ≤ 1, 0 ≤ z ≤ 1∫ ∫

DF • n dS =

∫ ∫ ∫

D(∇ • F) dV =

∫ 1

0

∫ 1

0

∫ 1

0(3) dx dy dz

= 3 (Volume of a unit cube) = 3

(c) F =⟨

x2, y2, z2⟩

∇• F =∂

∂x

(

x2)

+∂

∂y

(

y2)

+∂

∂z

(

z2)

= 2x + 2y + 2z

D = (ρ, φ, θ) |0 ≤ ρ ≤ 1, 0 ≤ φ ≤ π, 0 ≤ θ ≤ 2π∫∫

DF • n dS

=

∫ ∫ ∫

D(∇ • F) dV =

∫ 2π

0

∫ π

0

∫ 1

0(2x + 2y + 2z) ρ2 sinφdρ dφ dθ

=

∫ 2π

0

∫ π

0

∫ 1

02 (ρ sinφ cos θ + ρ sinφ sin θ + ρ cos φ) ρ2 sinφdρ dφ dθ

=

∫ 1

0ρ3 dρ

∫ π

0

∫ 2π

02(

sin2 φ cos θ + sin2 φ sin θ + sin φ cos φ)

dθ dφ

=

[

1

4ρ4

]1

0

∫ π

02[

sin2 φ sin θ − sin2 φ cos θ + θ sinφ cos φ]2π

0dθ dφ

=1

4

∫ π

04π sinφ cos φdφ = π

[

1

2sin2 φ

0

= 0

152

Page 155: AMA SolutionsManual

CHAPTER 6. VECTOR CALCULUS

(45) The total force acting on D is

−∫

SρgzndS • k = −

Sρgzk • ndS = −

D∇ • ρgzkdxdydz

= −ρg

D

(

∂xi+

∂yj+

∂zk

)

• zkdxdydz

= −ρg

D

∂z

∂zdxdydz

= −ρg

Ddxdydz

= −ρg × volume of D

(46) (a) F = 〈2z,−y, x〉

∇ × F =

i j k∂∂x

∂∂y

∂∂z

2z −y x

= 〈0, 1, 0〉

The equation for the area within ∆ABC:

unit normal vector = n = 〈1,1,1〉√12+12+12

=⟨

1√3, 1√

3, 1√

3

(x− 2) + (y − 0) + (z − 0) = 0 =⇒ x + y + z = 2

find the integral by using surface integral

S = (x, y) |0 ≤ x ≤ 2, 0 ≤ y ≤ 2− xdS = 1

|n • k| dx dy =√

3 dx dy

∂SF • dr =

S(∇× F) • n dS =

∫ 2

0

∫ 2−x

0

(

1√3

)√3 dy dx =

∫ 2

0[y]2−x

0 dx

=

∫ 2

0(2− x) dx =

[

2x− 1

2x2

]2

0

= 2

Find the integral by using line integral

Γ1 (AB) : r (t) = 〈2− 2t, 2t, 0〉, 0 ≤ t ≤ 1

F (r (t)) = 〈0,−2t, 2− 2t〉r′ (t) = 〈−2, 2, 0〉F (r (t)) • r′ (t) = 〈0,−2t, 2− 2t〉 • 〈−2, 2, 0〉 = −4t∫

Γ1F (r (t)) • r′ (t) dt =

∫ 10 (−4t) dt = −2

Γ2 (BC) : r (s) = 〈0, 2− 2s, 2s〉, 0 ≤ s ≤ 1

F (r (s)) = 〈4s, 2s− 2, 0〉r′ (s) = 〈0,−2, 2〉

153

Page 156: AMA SolutionsManual

CHAPTER 6. VECTOR CALCULUS

F (r (s)) • r′ (s) = 〈4s, 2s− 2, 0〉 • 〈0,−2, 2〉 = 4− 4s∫

Γ2F (r (s)) • r′ (s) ds =

∫ 10 (4− 4s) ds = 2

Γ3 (CA) : r (u) = 〈2u, 0, 2− 2u〉, 0 ≤ u ≤ 1

F (r (u)) = 〈4− 4u, 0, 2u〉r′ (u) = 〈2, 0,−2〉F (r (u)) • r′ (u) = 〈4− 4u, 0, 2u〉 • 〈2, 0,−2〉 = 8− 12u∫

Γ3F (r (u)) • r′ (u) du =

∫ 10 (8− 12u) du = 2

Therefore,∫

∂S F • dr =∫

Γ1+∫

Γ2+∫

Γ3= −2 + 2 + 2 = 2.

(b) F = 〈xz,−y, xy〉

∇ × F =

i j k∂∂x

∂∂y

∂∂z

xz −y xy

= 〈x, x− y, 0〉

The equation for the plane that including ∆ABC:

unit normal vector = n = 〈1,1,1〉√12+12+12

=⟨

1√3, 1√

3, 1√

3

(x− 1) + (y − 0) + (z − 0) = 0 =⇒ x + y + z = 1

Find the integral by using surface integral.

S = (x, y) |0 ≤ x ≤ 1, 0 ≤ y ≤ 1− xdS = 1

|n • k| dx dy =√

3 dx dy

∂SF • dr =

S(∇× F) • n dS =

∫ 1

0

∫ 1−x

0

[

1√3

(2x− y)

]√3 dy dx

=

∫ 1

0

[

2xy − 1

2y2

]1−x

0

dx =

∫ 1

0

(

3x− 1

2− 5

2x2

)

dx =1

6

Find the integral by using line integral

Γ1 (AB) : r (t) = 〈1− t, t, 0〉, 0 ≤ t ≤ 1

F (r (t)) =⟨

0,−t, t− t2⟩

r′ (t) = 〈−1, 1, 0〉F (r (t)) • r′ (t) =

0,−t, t− t2⟩

• 〈−1, 1, 0〉 = −t∫

Γ1F (r (t)) • r′ (t) dt =

∫ 10 (−t) dt = −1

2

Γ2 (BC) : r (s) = 〈0, 1− s, s〉, 0 ≤ s ≤ 1

F (r (s)) = 〈0, s− 1, 0〉r′ (s) = 〈0,−1, 1〉F (r (s)) • r′ (s) = 〈0, s− 1, 0〉 • 〈0,−1, 1〉 = 1− s∫

Γ2F (r (s)) • r′ (s) ds =

∫ 10 (1− s) ds = 1

2

Γ3 (CA) : r (u) = 〈u, 0, 1− u〉, 0 ≤ u ≤ 1

F (r (u)) =⟨

u− u2, 0, 0⟩

154

Page 157: AMA SolutionsManual

CHAPTER 6. VECTOR CALCULUS

r′ (u) = 〈1, 0,−1〉F (r (u)) • r′ (u) =

u− u2, 0, 0⟩

• 〈1, 0,−1〉 = u− u2

Γ3F (r (u)) • r′ (u) du =

∫ 10

(

u− u2)

du = 16

Therefore,∫

∂S F • dr =∫

Γ1+∫

Γ2+∫

Γ3= −1

2 + 12 + 1

6 = 16 .

(47) (a)

∇× F =

i j k∂∂x

∂∂y

∂∂z

x2 2x z2

= 2k

By Stokes’ Theorem,

CF • dr =

∫∫

S∇× F • ndS =

∫∫

4x2+y2≤42k • kdS

= 2

∫∫

4x2+y2≤4dS = 2× 2π = 4π

(b)

∇× F =

i j k∂∂x

∂∂y

∂∂z

y zx x2

= −xi− 2xj+(z − 1)k

By Stokes’ Theorem,

CF • dr =

∫∫

S∇× F • ndS

=

∫∫

S(−xi− 2xj+ (z − 1)k) •

(

1√3i +

1√3j+

1√3k

)

dS

=1√3

∫∫

S(−3x + z − 1) dS

=1√3

∫ 1

0

∫ 1−x

0(−3x− x− y)

1 + (zx)2 + (zy)2dydx

=1√3

∫ 1

0

∫ 1−x

0(−4x− y)

1 + (−1)2 + (−1)2dydx

=

∫ 1

0

∫ 1−x

0(−4x− y) dydx =

[

−4xy − 1

2y2

]1−x

0

=

∫ 1

0

(

−3x +7

2x2 − 1

2

)

dx

= −5

6

155

Page 158: AMA SolutionsManual

CHAPTER 6. VECTOR CALCULUS

(48) By Stokes’ Theorem,

∫∫

S∇× (yi) • ndS =

C(yi) • dr =

C(yi) • (dxi + dyj + dzk) =

Cydx

=

∫ 2π

0sin td (cos t) = −

∫ 2π

0sin2 tdt = −π

156

Page 159: AMA SolutionsManual

Chapter 7

Differential Equations

(1) (a) dydx = 4y

x(y−3) =⇒ (y−3)dyy = 4dx

x =⇒(

1− 3y

)

dy = 4dxx

(

1− 3y

)

dy = 4∫

dxx =⇒ y − 3 ln |y| = 4 ln |x|+ C

(b) dydx = y3+4y

x(y2+2y+4)=⇒ (y2+2y+4)dy

y3+4y= dx

x =⇒(

1y + 2

y2+4

)

dy = dxx

(

1y + 2

y2+4

)

dy =∫

dxx =⇒ ln |y|+ tan−1 y

2 = ln |x|+ C

(c) dydx − 2

xy = − 1x2 p (x) = − 2

x µ (x) = e∫

p(x)dx = e−∫

2x

dx = e−2 ln|x| = 1x2

1x2

(

dydx − 2

xy)

= − 1x4 =⇒ d

dx

(

1x2 y)

= − 1x4 =⇒ 1

x2 y = −∫

1x4 dx = 1

3x3 + C

y = 13x + Cx2

(d) (x− 1) dydx = y + 2 (x− 2)3 =⇒ dy

dx = yx−1 + 2(x−2)3

x−1 =⇒ dydx −

yx−1 = 2(x−2)3

x−1

p (x) = − 1x−1

µ (x) = e−∫

1x−1

dx = 1x−1

1x−1

(

dydx −

yx−1

)

= 2(x−2)3

(x−1)2=⇒ d

dx

(

yx−1

)

= 2x− 8 + 6x−1 − 2

(x−1)2

yx−1 =

[

2x− 8 + 6x−1 − 2

(x−1)2

]

dx = x2 − 8x + 6 ln |x− 1|+ 2x−1 + C

y =(

x2 − 8x + 6 ln |x− 1|+ 2x−1 + C

)

(x− 1)

(e) x3 dydx +

(

2− 3x2)

y = x3 =⇒ dydx +

(2−3x2)x3 y = 1

p (x) =(2−3x2)

x3

µ (x) = e∫ (2−3x2)

x3 dx = e

(

−3 ln x− 1

x2

)

= 1x3 e−

1

x2

1x3 e−

1

x2

[

dydx +

(2−3x2)x3 y

]

= 1x3 e−

1

x2 =⇒ ddx

(

1x3 e−

1

x2 y)

= 1x3 e−

1

x2

1x3 e−

1

x2 y =∫

1x3 e−

1

x2 dx = 12e−

1

x2 + C =⇒ y = 12x3 + Cx3e

1

x2

(f) dydx + (2 cos x) y = sin2 x cos x

157

Page 160: AMA SolutionsManual

CHAPTER 7. DIFFERENTIAL EQUATIONS

p (x) = 2 cos x

µ (x) = e∫

2 cos xdx = e2 sin x

e2 sin x[

dydx + (2 cos x) y

]

= e2 sin x sin2 x cos x =⇒ ddx

(

e2 sin xy)

= e2 sin x sin2 x cos x

e2 sin xy =∫

e2 sin x sin2 x cos xdx =∫

e2 sin x sin2 xd (sinx)

Let u = sin2 x and dv = e2 sin xd (sinx), then du = 2 sinx cos xdx and v = 12e2 sin x.

e2 sin xy =1

2e2 sin x sin2 x−

∫ (

1

2e2 sin x

)

(2 sinx cos xdx)

=1

2e2 sin x sin2 x−

e2 sin x sinx cos xdx

=1

2e2 sin x sin2 x−

e2 sin x sinxd (sinx)

=1

2e2 sin x sin2 x− 1

2e2 sin x sinx +

1

2

e2 sin x cos xdx

=1

2e2 sin x sin2 x− 1

2e2 sin x sinx +

1

4e2 sin x + C

= e2 sin x

(

1

2sin2 x− 1

2sinx +

1

4

)

+ C

y = 12 sin2 x− 1

2 sin x + 14 + Ce−2 sin x or y = Ce−2 sin x − 1

4 cos 2x− 12 sinx + 1

2

(2) (a) The substitution y = xv(x) reduces the differential equation to

v + xdv

dx=

2 + v

1 + 3v, or x

dv

dx=

2− 3v2

1 + 3v.

Therefore, one has∫ 1 + 3v

2− 3v2dv =

∫ dx

x. Integration yields

√6− 6

12ln

(

y

x+

√6

3

)

−√

6 + 6

12ln

(

y

x−√

6

3

)

= lnCx,

where C is an arbitrary constant.

(b) Re-writing the equation as y′ +

(

3

x

)

y = 4 +2

x2and multiplying both sides of this

equation by µ = e∫

3x

dx = x3, we obtain

d

dx

(

x3y)

= 4x3 + 2x.

Integration yields

x3y = x4 + x2 + C, or y = x +1

x+

C

x3.

158

Page 161: AMA SolutionsManual

CHAPTER 7. DIFFERENTIAL EQUATIONS

(c) Let v = yx , then dv

dx = 1x

dydx −

yx2 =⇒ dy

dx = x(

dvdx + y

x2

)

.

x2(

dvdx + y

x2

)

− y + 3x3y − x4 = 0 =⇒ x2 dvdx + 3x3y − x4 = 0 =⇒ dv

dx + 3x2v = x2

p (x) = 3x2

µ (x) = e∫

3x2dx = ex3

ex3 ( dvdx + 3x2v

)

= x2ex3

=⇒ ddv

(

ex3

v)

= x2ex3

=⇒ ex3

v =∫

x2ex3

dx = 13ex3

+ C

v = 13 + Ce−x3

=⇒ yx = 1

3 + Ce−x3

=⇒ y = 13x + Cxe−x3

(d) Let v = yx , then dv

dx = 1x

dydx −

yx2 =⇒ dy

dx = x(

dvdx + y

x2

)

.

x(

dvdx + y

x2

)

= 2 yx +

(

xy

)3= 2v + 1

v3 =⇒ x dvdx + v = 2v + 1

v3 =⇒ x dvdx = v + 1

v3

=⇒ x dvdx = v4+1

v3 =⇒ v3dvv4+1

= dxx

v3dvv4+1

=∫

dxx =⇒ 1

4 ln∣

∣v4 + 1∣

∣ = ln |x|+ C =⇒ 14 ln

( yx

)4+ 1∣

∣ = ln |x|+ C

(e) Let v = 1y , then dv = − 1

y2 dy =⇒ dy = −y2dv = − 1v2 dv.

− dvv2dx

+ 1v = 1

v2 =⇒ dvv2dx

= 1v − 1

v2 =⇒ dvdx = v − 1 =⇒ dv

v−1 = dx∫

dvv−1 =

dx =⇒ ln |v − 1| = x + C =⇒ ln∣

1y − 1

∣= x + C

(f) Let v = 1y4 , then dv = − 4

y5 dy =⇒ dy = −14y5dv.

−14y5 dv

dx − 12xy = −y5 =⇒ dv

dx + 2xy4 = 4 =⇒ dv

dx + 2vx = 4

p (x) = 2x

µ (x) = e∫

2x

dx = e2 ln x = x2

x2(

dvdx + 2v

x

)

= 4x2 =⇒ ddx

(

x2v)

= 4x2 =⇒ x2v =∫

4x2dx = 43x3 + C

v = 43x + C 1

x2 =⇒ 1y4 = 4

3x + C 1x2

(g) Let v = y − 4x, then dvdx = dy

dx − 4 =⇒ dydx = dv

dx + 4.

dvdx + 4 = v2 =⇒ dv

dx = v2 − 4 =⇒ dvv2−4

= dx∫

dvv2−4

=∫

dx =⇒ −∫

dv4−v2 =

dx =⇒ −14 ln

v+2v−2

∣= x + C

Use the formula∫

dua2−u2 = 1

2a ln∣

u+au−a

∣+ C.

(h) Putting v =1

y, the equation is then transformed into

− 1

v2

dv

dx+

1

xv=

x

v2, or

dv

dx− 1

xv = −x,

which is a linear equation with unknown v. Multiplying the equation by µ =

e∫

− 1x

dx =1

x, one obtains

d

dx

(

1

xv

)

= −1, which implies1

xv = −x + C, or

1

xy=

−x + C.

(i) Substituting y = w− 13 into the differential equation, we obtain

x3

3w− 4

3dw

dx+ x2w− 1

3 = w− 43 cos x,

159

Page 162: AMA SolutionsManual

CHAPTER 7. DIFFERENTIAL EQUATIONS

ordw

dx+

(

3

x

)

w =3 cos x

x3, which is a linear equation with unknown w. Multiplying

the equation by µ = x3, we haved

dx(x3w) = 3 cos x. Integration then gives w =

3 sinx + C

x3, or y =

x3√

3 sinx + C.

(3) (a) The equation is separable and may be written as y dy = ex dx, which may be inte-

grated to give y2 = 2ex + C, where C is a constant of integration. Applying the

initial condition y(0) = 1, we conclude that C = −1 and that the solution to the

initial value problem satisfies y2 = 2ex − 1.

(b) The change of variable y = xv(x) reduces the equation to

dx

x=

dv

cot v= tan v dv,

or∫ dx

x=∫

tan v dv, which implies lnx = − ln cos v + lnC. Therefore, the solution

of the differential equation satisfies

x cosy

x= C. By the initial condition y(π) = 0, we have C = π. Therefore,

x cosy

x= π.

(c) The differential equation is separable, which may be integrated to give

(1− y5)dy =

x ex2

dx,

or y − y6

6= 1

2ex2

+ C. Since y(0) = 0, we conclude that C = −12 . As such, the

solution of the initial value problem satisfies

y6 − 6y + 3ex2

= 3.

(d) dydx = x2y−y

y+1 =y(x2−1)

y+1 =⇒ y+1y dy =

(

x2 − 1)

dx =⇒(

1 + 1y

)

dy =(

x2 − 1)

dx

(

1 + 1y

)

dy =∫ (

x2 − 1)

dx =⇒ y + ln |y| = x3

3 − x + C

x = 3, y = −1 =⇒ −1 + ln |−1| = 9− 3 + C =⇒ C = −7

y + ln |y| = x3

3 − x− 7

(e) dydx + 2xy = 0 =⇒ dy

dx = −2xy =⇒ dyy = −2xdx =⇒ ln |y| = −x2 + C

x = 0, y = 2 =⇒ ln |2| = C

ln |y| = −x2 + ln 2 =⇒ y = e−x2+ln 2 = 2e−x2

(f) Let v = yx , then dv

dx = 1x

dydx −

yx2 =⇒ dy

dx = x(

dvdx + y

x2

)

.

x(

dvdx + y

x2

)

= 1+(y/x)2

2(xy/x2)= 1+v2

2v =⇒ x dvdx + v = 1+v2

2v =⇒ x dvdx = 1−v2

2v =⇒ 2v1−v2 dv = dx

x

160

Page 163: AMA SolutionsManual

CHAPTER 7. DIFFERENTIAL EQUATIONS

2v1−v2 dv =

dxx =⇒ − ln

∣1− v2∣

∣ = ln |x|+ C =⇒ − ln∣

∣1−

( yx

)2∣

∣= ln |x|+ C

x = 1, y = −2 =⇒ − ln∣

∣1−

(−21

)2∣

∣= ln |1|+ C =⇒ C = − ln 3

− ln∣

∣1−

( yx

)2∣

∣= ln |x| − ln 3

(g) dydx + xy = xy2 =⇒ dy

dx = xy (y − 1) =⇒ dyy(y−1) = xdx =⇒

(

1y−1 − 1

y

)

dy = xdx

(

1y−1 − 1

y

)

dy =∫

xdx =⇒ ln |y − 1| − ln |y| = x2

2 + C =⇒ ln∣

y−1y

∣= x2

2 + C

x = 0, y = 2 =⇒ ln∣

2−12

∣ = C =⇒ C = − ln 2

ln∣

y−1y

∣= x2

2 − ln 2

(4) (a) With M = 2xy+cosx and N = x2+sin y, a simple calculation gives∂M

∂y=

∂N

∂x= 2x.

So the equation is exact. Integrating the equation∂f

∂x= M with respect to x results

in

f(x, y) =

(2xy + cos x) dx = x2y + sinx + g(y),

where g(y) is an arbitrary function of f . If we substitute this into the second equation∂f

∂y= N , we obtain

dg

dy= sin y, or g(y) = − cos y. We therefore conclude that the

solution of the equation satisfies f(x, y) = C, or x2y + sinx− cos y = C.

(b) Since∂

∂y

[

y + 2xy3]

=∂

∂x

[

1 + 3x2y2 + x]

= 1 + 6xy2, the equation is exact. We

consider the equations

∂f

∂x= y + 2xy3

∂f

∂y= 1 + 3x2y2 + x

Integrating the first equation with respect to x, we obtain

f(x, y) =

(y + 2xy3)dx = xy + x2y3 + g(y).

Substituting this into the second equation, we conclude that

x + 3x2y2 +dg

dy= 1 + 3x2y2 + x.

Thusdg

dy= 1 and g(y) = y. Therefore, f(x, y) = xy + x2y3 + y, which implies that

the solution of the equation satisfies

xy + x2y3 + y = C.

161

Page 164: AMA SolutionsManual

CHAPTER 7. DIFFERENTIAL EQUATIONS

(5) Let y (t) be the number of bacteria at time t (hour) and dydt be the growth rate of the

bacteria. So dydt = ay, where a is a constant.

dydt = ay =⇒ dy

y = adt =⇒ ln |y| = at + C =⇒ y (t) = eateC = A0eat, where A0 is a

constant.

Let y (0) = A0, then y (6) = A0e6a = 2A0.

=⇒ e6a = 2 =⇒ a = 16 ln 2

Thus, y (t) = A0e(t ln 2)/6.

y (24) = A0e(24 ln 2)/6 = 16A0

So the number of bacteria is 16 times as much as the number at the beginning.

(6) Let x (t) be the weight of the radioactive material at time t (year) and dxdt be the rate of

change of the weight. So dxdt = −bx, where b is a constant.

dxdt = −bx =⇒ x (t) = B0e

−bt, where B0 is a constant.

Let x (0) = 100 = B0, then x (2) = B0e−2b = 0.95× 100.

=⇒ e−2b = 0.95 =⇒ b = −12 ln 0.95

Thus, x (t) = 100e(t ln 0.95)/2.

x (t) = 100e(t ln 0.95)/2 = 0.9× 100 =⇒ e(t ln 0.95)/2 = 0.9 =⇒ t = 2 ln 0.9ln 0.95 = 4.1082

So it takes 4.11 years for 10% of the original mass to decay.

(7) Let z (t) be the amount of the deposit at time t (year) and dzdt be the rate of change of

the deposit. So dzdt = 0.07z and z (t) = C0e

0.07t, where C0 is a constant.

Let z (0) = C0 = 10000, then z (t) = 10000e0.07t.

z (2) = 10000e0.07(2) = 11503

So there will be $11503 in the account after 2 years.

(8) Let w (t) be the temperature of the bar at time t (hour) and dwdt be the rate of change of

the temperature. So dwdt = d (w (t)− S), where d is a constant and S is the surrounding

temperature.

dww(t)−S = (d) dt =⇒ ln |w (t)− S| = dt + c =⇒ w (t) = S + Doe

dt, where D0 is a constant.

Let w (0) = 100 and S = 20, then 20 + D0 = 100 =⇒ D0 = 80.

w(

13

)

= 20 + 80ed/3 = 50 =⇒ ed/3 = 38 =⇒ d = 3 ln 3

8

Thus, w (t) = 20 + 80e(3 ln 38)t.

162

Page 165: AMA SolutionsManual

CHAPTER 7. DIFFERENTIAL EQUATIONS

(9) Let T (t) be the temperature of the copper ball at time t. By Newton’s Law of Cooling,

we havedT

dt= k(T − 30),

where k is a physical constant, with initial condition T (0) = 100. The equation is

separable, and an integration gives T (t)−30 = C ekt for any t > 0. T (0) = 100⇒ C = 70,

and T (3) = 70⇒ e3k =40

70⇒

k = −13 ln

(

74

) ∼= −0.187. Thus T (t) = 30 + 70 e−13

ln( 74)t for any t > 0. If T (t) = 40, a

simple calculation shows that t = 3× ln 7

ln(7/4)∼= 10.4 minutes.

(10) Dividing the equation by L and then multiplying both sides by

µ = eRL

t, we obtaind

dt

[

eRL

t × i]

=E0

Le

RL

t cos ωt.

Integration yields eRL

t × i =E0

L

eRL

t cos ωt dt, which implies

i(t) =E0

R2 + ω2L2(R cos ωt + ωL sin ωt) + C e−

RL

t,

where C is an arbitrary constant. Using the initial condition i(0) = 0, we conclude that

C =−E0R

R2 + ω2L2. Therefore,

i(t) =E0

R2 + ω2L2

[

R cos ωt + ωL sin ωt−R e−RL

t]

.

(11) Let S(t) be the amount of pollutants at any instant t, S(0) = S0 be the initial amount

of pollutants, V be the volume of water in the tank and r be the rate of influx / outflux.

Since the influx has no pollutants at all, we have

dS

dt=

(

dS

dt

)

in

−(

dS

dt

)

out

= −rS

V

Integration gives S(t) = S0e−( r

V )t for any t > 0.

(a) If S(t50%) =1

2S0, then e−( r

V )t50% =1

2, or

( r

V

)

t50% = ln 2. Thus t50% =V

r× ln 2.

(b) If S(t10%) =1

10S0, then e−( r

V )t10% =1

10, or

( r

V

)

t10% = ln 10. Thus t10% =

V

r× ln 10.

As V = 5000 and r = 4000, t50% =5000

4000× ln 2 ∼= 0.866 hours (or 52 minutes) and

163

Page 166: AMA SolutionsManual

CHAPTER 7. DIFFERENTIAL EQUATIONS

t10% =5000

4000× ln 10 ∼= 2.88 hours (or 2 hours 53 minutes).

(12) The equation of motion is my = mg, or y = g, where g ∼= 9.8 m/sec2 is the acceleration

due to gravity. The general solution is y(t) = 12gt2+c1t+c2, where c1 and c2 are arbitrary

constants. Using the initial conditions y(0) = 0 and y(0) = −5, we obtain c2 = 0 and

c1 = −5 and therefore y(t) = 12gt2 − 5t. Since y(t) = 0 if and only if t = 5

g∼= 0.51 sec,

the minimum value of y(t) is equal to −252g∼= −1.276. As such, the maximum height

reached by the stone is 1.276 meters above the point from which the stone was released.

When y(t) = 650, we solve 12gt2 − 5t − 650 = 0 to obtain t =

5 +√

25 + 1300g

g∼= 12.04

sec, meaning that the stone will strike the ground approximately 12.04 seconds after

release. Since y = gt − 5, the velocity when the stone strikes the ground is given by√

25 + 1300g ∼= 112.98 m/sec.

(13) (a) Auxiliary equation: m2 + 4m + 4 = 0. ∴ m1 = m2 = −2.

The general solution is: y = e−2x(c1 + c2x)

(b) Auxiliary equation: m2 − 7m + 12 = 0. ∴ m1 = 3, m2 = 4.

The general solution is: y = c1e3x + c2e

4x.

(c) Auxiliary equation: m2 + 4m + 9 = 0. ∴ m = −2±√

5i.

The general solution is: y = e−2x(c1 cos√

5x + c2 sin√

5x)

(d) Auxiliary equation: m2 − 2m + 10 = 0. ∴ m = 1± 3i.

The general solution is: y = ex(A cos 3x + B sin 3x).

(e) Auxiliary equation: m2 − 2m + 1 = 0. ∴ m = 1, 1.

The general solution is: y = c1ex + c2xex.

(f) Auxiliary equation: m2 + 2m + 2 = 0. ∴ m = −1± i.

The general solution is: y = e−x (c1 cos x + c2 sinx).

(14) (a) Let yp (x) = Ke3x, then y′p (x) = 3Ke3x.

y′ − 5y = e3x =⇒ 3Ke3x − 5(

Ke3x)

= e3x =⇒ −2K = 1 =⇒ K = −12

Therefore, yp (x) = −12e3x

(b) Let yp (x) = Ke−5x, then y′p (x) = −5Ke−5x.

y′ + 6y = 4e−5x =⇒ −5Ke−5x + 6(

Ke−5x)

= 4e−5x =⇒ K = 4

Therefore, yp (x) = 4e−5x

(c) Let yp (x) = (Ax + B) e2x, then

y′p (x) = Ae2x + 2 (Ax + B) e2x = [2Ax + (A + 2B)] e2x.

164

Page 167: AMA SolutionsManual

CHAPTER 7. DIFFERENTIAL EQUATIONS

y′ − 5y = xe2x

[2Ax + (A + 2B)] e2x − 5[

(Ax + B) e2x]

= xe2x

[−3Ax + (A− 3B)] = x

Comparing the coefficients of the polynomial,

−3A = 1

A− 3B = 0=⇒

A = −13

B = −19

Therefore, yp (x) =(

−13x− 1

9

)

e2x.

(d) Let yp (x) = (Ax + B) e2x, then y′p (x) = [2Ax + (A + 2B)] e2x.

y′ + 6y = (2x− 1) e2x =⇒ [2Ax + (A + 2B)] e2x + 6 (Ax + B) e2x = (2x− 1) e2x

=⇒ [8Ax + (A + 8B)] = 2x− 1

Comparing the coefficients of the polynormial,

8A = 2

A + 8B = −1=⇒

A = 14

B = − 532

Therefore, yp (x) =(

14x− 5

32

)

e2x.

(e) Let yp (x) =(

Ax2 + Bx + C)

e2x, then

y′p (x) = (2Ax + B) e2x + 2e2x(

Ax2 + Bx + C)

=[

2Ax2 + (2A + 2B)x + (B + 2C)]

e2x.

y′ − 5y =(

−9x2 + 6x)

e2x

=⇒[

2Ax2 + (2A + 2B)x + (B + 2C)]

e2x−5(

Ax2 + Bx + C)

e2x =(

−9x2 + 6x)

e2x

=⇒ y′p (x) =[

−3Ax2 + (2A− 3B)x + (B − 3C)]

e2x =(

−9x2 + 6x)

e2x

Comparing the coefficients of the polynormial,

− 3A = −9

2A− 3B = 6

B − 3C = 0

=⇒

A = 3

B = 0

C = 0

Therefore, yp (x) = 3x2e2x.

(15) (a) Putting yp = Ax2 +Bx+C, where A, B and C are coefficients to be determined, we

obtain y′p = 2Ax + B and y′′p = 2A. Substituting into the equation and comparing

coefficients, we have

A =1

2, B =

3

2, C =

7

4

Therefore, yp = 14(2x2 + 6x + 7).

165

Page 168: AMA SolutionsManual

CHAPTER 7. DIFFERENTIAL EQUATIONS

(b) Putting yp = A cos 4x + B sin 4x into the equation, we have

y′p = −A sin 4x + 4B cos 4x and y′′p = −16A cos 4x − 16B sin 4x. Comparing coeffi-

cients, we obtain A = 225 , B = − 1

25 . Therefore, yp = 125(2 cos 4x− sin 4x).

(c) Solve for yh: m− 5 = 0 =⇒ m = 5 =⇒ yh = c1e5x.

Let yp = Ax2 + Bx + C, then y′p = 2Ax + B.

(2Ax + B)− 5(

Ax2 + Bx + C)

= −5Ax2 + (2A− 5B)x + (B − 5C) = 8x2 − 2

Comparing the coefficients of the polynormial,

−5A = 8

2A− 5B = 0

B − 5C = −2

=⇒

A = −85

B = −1625

C = 34125

Therefore, yp = −85x2 − 16

25x + 34125 and y = c1e

5x − 85x2 +−16

25x + 34125 .

(d) yh = c1e5x

Let yp = Ke3x, then y′p = 3Ke3x.

3Ke3x − 5Ke3x = e3x =⇒ K = −12

Therefore, yp = −12e3x and y = c1e

5x − 12e3x.

(e) yh = c1e5x

Let yp = A sinx + B cos x, then y′p = A cos x−B sinx.

(A cos x−B sinx)−5 (A sinx + B cos x) = (A− 5B) cos x+(−B − 5A) sinx = cos x

Comparing the coefficients of sine and cosine,

A− 5B = 1

−B − 5A = 0=⇒

A = 126

B = − 526

Therefore, yp = 126 sinx− 5

26 cos x and y = c1e5x + 1

26 sinx− 526 cos x.

(f) Let yp = Ae3x + Bx2 + Cx + D, where A, B, C and D are constants to be deter-

mined. Substituting yp into the equation y′′ +3y′ +2y = 10e3x +4x2 and comparing

coefficients, we obtain

A =1

2, B = 2, C = −6 and D = 7.

Hence yp = 12e3x + 2x2 − 6x + 7.

(g) Putting yp = A cos 2x + B sin 2x + Cex, we obtain y′p = 2B cos 2x− 2A sin 2x + Cex

and y′′p = −4A cos 2x− 4B sin 2x + Cex. By comparing coefficients, we have

−3A + 4B = 2, − 4A− 3B = 0, 4C = 3.

Solving the equations, we conclude that A = − 625 , B = 8

25 and C = 34 . Therefore,

yp = − 625 cos 2x + 8

25 sin 2x + 34ex.

166

Page 169: AMA SolutionsManual

CHAPTER 7. DIFFERENTIAL EQUATIONS

(h) yh = c1e5x

Let yp = A sinx + B cos x + Ce3x, then y′p = A cos x−B sinx + 3Cex.

(A cos x−B sinx + 3Cex)− 5(

A sin x + B cos x + Ce3x)

= e3x + cos x

=⇒ (A− 5B) cos x− (5A + B) sinx− 2Ce3x = e3x + cos x

Comparing the coefficients of each function term,

A− 5B = 1

5A + B = 0

−2C = 1

=⇒

A = 126

B = − 526

C = −12

Therefore, yp = 126 sinx− 5

26 cos x− 12e3x and y = c1e

5x+ 126 sinx− 5

26 cos x− 12e3x.

(i) Solve for yh: m2 + 4m + 8 = 0 =⇒ m = −2± 2i.

With α = −2 and β = 2, yh = e−2x (c1 sin 2x + c2 cos 2x).

Let yp = Ax2 + Bx + C, then y′p = 2Ax + B and y′′p = 2A.

2A + 4 (2Ax + B) + 8(

Ax2 + Bx + C)

= 8Ax2 + (8A + 8B) x + (2A + 4B + 8C) =

8x2 + 8x + 18

Comparing the coefficients of the polynormial,

8A = 8

8A + 8B = 8

2A + 4B + 8C = 18

=⇒

A = 1

B = 0

C = 2

Therefore, yp = x2 + 2 and y = e−2x (c1 sin 2x + c2 cos 2x) + x2 + 2.

(j) Solve for yh: m2 + 4m + 5 = 0 =⇒ m = −2± i.

With α = −2 and β = 1, yh = e−2x (c1 sinx + c2 cos x).

Let yp = A sinx + B cos x, then y′p = A cos x−B sinx and y′′p = −A sinx−B cos x.

4 (A−B) sinx + 4 (A + B) cos x

= (−A sinx−B cos x) + 4 (A cos x−B sinx) + 5 (A sinx + B cos x)

Comparing the coefficients of the sine and cosine,

4 (A−B) = −2

4 (A + B) = 2=⇒

A = 0

B = 12

Therefore, yp = 12 cos x and y = e−2x (c1 sinx + c2 cos x) + 1

2 cos x.

(k) Solve for yh: m2 − 2m = 0 =⇒ m = 0, 2. Then yh = c1 + c2e2x.

167

Page 170: AMA SolutionsManual

CHAPTER 7. DIFFERENTIAL EQUATIONS

Let yp = Aex sinx + Bex cos x, then

y′p = Aex cos x + Aex sinx + Bex cos x−Bex sinx

= [(A−B) sinx + (A + B) cos x] ex

y′′p = [(A−B) sinx + (A + B) cos x] ex + ex [(A−B) cos x− (A + B) sin x]

= ex (2A cos x− 2B sinx) .

and

ex (2A cos x− 2B sinx)− 2 [(A−B) sinx + (A + B) cos x] ex = ex sin x

ex (−2A sinx− 2B cos x) = ex sin x

Comparing the coefficients of the sine and cosine,

−2A = 1

−2B = 0=⇒

A = −12

B = 0

Therefore, yp = −12ex sin x and y = c1 + c2e

2x − 12ex sin x.

(l) Solve for yh: m2 −m + 2 = 0 =⇒ m = 12 ±

√7

2 i.

With α = 12 and β =

√7

2 , yh = ex2

(

c1 sin√

72 x + c2 cos

√7

2 x)

.

Let yp =(

Ax2 + Bx + C)

ex, then

y′p =[

Ax2 + (2A + B)x + (B + C)]

ex

y′′p = ex[

Ax2 + (4A + B) x + (2A + 2B + C)]

.

Substituting into the differential equation and simplifying yield

ex[

2Ax2 + (2A + 2B) x + (2A + B + C)]

=(

6x2 + 8x + 7)

ex

Comparing the coefficients of polynomial,

2A = 6

2A + 2B = 8

2A + B + 2C = 7

=⇒

A = 3

B = 1

C = 0

Therefore, yp =(

3x2 + x)

ex and y = ex2

(

c1 sin√

72 x + c2 cos

√7

2 x)

+(

3x2 + x)

ex.

(m) Solve for yh: m2 + 2m + 3 = 0 =⇒ m = −1±√

2i.

With α = −1 and β =√

2, yh = e−x(

c1 sin√

2x + c2 cos√

2x)

.

168

Page 171: AMA SolutionsManual

CHAPTER 7. DIFFERENTIAL EQUATIONS

Let yp = Ax2 + Bx + C + D sin x + E cos x, then

y′p = 2Ax + B + D cos x− E sinx

y′′p = 2A−D sinx− E cos x.

Substituting into the differential equation and simplifying yield

3Ax2+(4A + 3B) x+(2A + 2B + 3C)+(2D − 2E) sinx+(2D + 2E) cos x = x2+sinx

Comparing the coefficients of each function term,

3A = 1

4A + 3B = 0

2A + 2B + 3C = 0

2D − 2E = 1

2D + 2E = 0

=⇒

A = 13

B = −49

C = 227

D = 14

E = −14

Therefore, yp = 13x2− 4

9x+ 227 + 1

4 sin x− 14 cos x and y = e−x

(

c1 sin√

2x + c2 cos√

2x)

+13x2 − 4

9x + 227 + 1

4 sinx− 14 cos x.

(16) Substitution of yp = K(x) ekx, where K(x) is a function to be determined, into the

differential equation yields

K ′′(x) + (2k + p)K ′(x) + (k2 + pk + q)K(x) = A.

Since k2 + pk + q = 0, one has K ′′(x) + (2k + p)K ′(x) = A.

(i) If 2k + p 6= 0 (distinct roots), then K(x) =Ax

2k + p.

(ii) If 2k + p = 0 (repeated root), we have K(x) =Ax2

2.

(17) Putting yp = w(x) ekx,where w(x) is a function to be determined, we obtain

w′′ + (2k + p)w′ + (k2 + pk + q)w = f(x).

(a) Substitution of y = e−xw into the equation y′′ + 2y′ + y = 3x2e−x yields w′′ = 3x2.

Therefore, w =x4

4and thus yp =

x4

4e−x is a particular solution.

(b) Substitution of y = e4xw into the equation y′′− 8y′ = 2e4x sin 3x yields w′′− 16w =

2 sin 3x. Putting w = A cos 3x + B sin 3x and using the method of undetermined

coefficients, we conclude that w = − 225 sin 3x. Therefore yp = − 2

25 sin 3xe4x is a

particular solution of the equation.

169

Page 172: AMA SolutionsManual

CHAPTER 7. DIFFERENTIAL EQUATIONS

(18) (a)

W(

et sin t, et cos t)

= det

[

et sin t et cos t

et (sin t + cos t) et (cos t− sin t)

]

=(

et sin t) [

et (cos t− sin t)]

−(

et cos t) [

et (sin t + cos t)]

= −e2t

(b)

W (y1, y2, y3) = det

1 sin 2t cos 2t

(1)′ (sin 2t)′ (cos 2t)′

(1)′′ (sin 2t)′′ (cos 2t)′′

= det

1 sin 2t cos 2t

0 2 cos 2t −2 sin 2t

0 −4 sin 2t −4 cos 2t

= det

[

2 cos 2t −2 sin 2t

−4 sin 2t −4 cos 2t

]

= −8 cos2 2t− 8 sin2 2t = −8

(c)

W (y1, y2, y3, y4) = det

1 x x2 x3

0 1 2x 3x2

0 0 2 6x

0 0 0 6

= 12

(19) (a) The homogeneous equation has two independent solutions

y1 = e−2x cos 2x and y2 = e−2x sin 2x. Therefore,

W (x) =

e−2x cos 2x e−2x sin 2x

−2e−2x (cos 2x + 2 sin 2x) 2e−2x (− sin 2x + cos 2x)

= 2e−4x.

Using variation of parameters, one has a particular solution of the form yp =

v1e−2x cos 2x + v2e

−2x sin 2x, where

v1 =

∫ −y2R(x)

W (x)dx = −

e−2x sin 2xe−2x

2e−4xdx =

cos 2x

4,

v2 =

y1R(x)

W (x)dx =

e−2x cos 2xe−2x

2e−4xdx =

sin 2x

4.

Therefore,

yp =cos 2x

4e−2x cos 2x +

sin 2x

4e−2x sin 2x.

170

Page 173: AMA SolutionsManual

CHAPTER 7. DIFFERENTIAL EQUATIONS

(b) The homogeneous equations has independent solutions y1 = e−x and y2 = e2x.

Therefore, W (x) = 3ex. The equation has a particular solution of the form yp =

v1e−x + v2e

2x, where

v1 =

∫ −e2x sin 2x

3exdx = −1

3

ex sin 2x dx =1

15(−ex sin 2x + 2ex cos 2x)

and

v2 =

e−x sin 2x

3exdx =

1

3

e−2x sin 2x dx =e−2x

12(− cos 2x− sin 2x) .

Therefore,

yp =1

15(− sin 2x + 2 cos 2x) +

1

12(− sin 2x− cos 2x) = − 3

20sin 2x +

1

20cos 2x.

(c) We have y1 = ex, y2 = e3x and W (x) = 2e4x. The equation has a particular solution

of the form yp = v1ex + v2e

3x, where

v1 =

∫ −e3x

2e4x

1

1 + e−xdx =

1

2ln(1 + ex)− x

2

and

v2 =

ex

2e4x

1

1 + e−xdx = −1

4e−2x +

1

2e−x − 1

2ln(1 + e−x).

Then yp = ex(

12 ln(1 + ex)− x

2

)

+ e3x(

−14e−2x + 1

2e−x − 12 ln(1 + e−x)

)

.

(20) (a) For y′ + 1xy = 0, dy

dx = − yx =⇒ dy

y = −dxx =⇒ ln |y| = − ln |x|+ C

=⇒ yh = eCe− ln|x| = c11x .

Let yp = v (x) yh = v (x) 1x , then y′p = 1

xv′ (x)− 1x2 v (x) and v′ (x) = ln x

x1yh

= lnx.

Thus, v (x) =∫

lnxdx = x (lnx− 1) and yp = lnx−1. Therefore, y = c11x +lnx−1.

(b) yh = c1e5x

v′ (x) = e−5x(

8x2 − 2)

v (x) =∫

e−5x(

8x2 − 2)

dx = e−5x(

−85x2 − 16

25x + 34125

)

yp = −85x2 − 16

25x + 34125

y = c1e5x − 8

5x2 − 1625x + 34

125

(c) yh = c1e5x

v′ (x) = e−5x(

e3x)

= e−2x

v (x) =∫

e−2xdx = −12e−2x

yp = −12e3x

y = c1e5x − 1

2e3x

171

Page 174: AMA SolutionsManual

CHAPTER 7. DIFFERENTIAL EQUATIONS

(d) yh = c1e5x

v′ (x) = e−5x (cos x)

v (x) =∫

e−5x (cos x) dx = 126e−5x (sin x− 5 cos x)

yp = 126 (sin x− 5 cos x)

y = c1e5x + 1

26 sinx− 526 cos x

(e) Solve for yh: m2 −m− 2 = 0 =⇒ m = −1, 2.

Thus, yh = c1e−x + c2e

2x.

Let y1 = e−x and y2 = e2x, then W (y1, y2) = 3ex.

v1 =∫

− e2x(4x2)3ex dx = −4

3ex(

x2 − 2x + 2)

v2 =∫ e−x(4x2)

3ex dx = −13e−2x

(

2x2 + 2x + 1)

yp = −43ex

(

x2 − 2x + 2)

(e−x)− 13e−2x

(

2x2 + 2x + 1) (

e2x)

= −2x2 + 2x− 3

y = c1e−x + c2e

2x − 2x2 + 2x− 3

(f) yh = c1e−x + c2e

2x

Let y1 = e−x and y2 = e2x, then W (y1, y2) = 3ex.

v1 =∫

− e2x sin 2x3ex dx = 2

15ex cos 2x− 115ex sin 2x

v2 =∫

e−x sin 2x3ex dx = − 1

12e−2x (cos 2x + sin 2x)

yp = 115 (2 cos 2x− sin 2x)− 1

12 (cos 2x + sin 2x) = 120 cos 2x− 3

20 sin 2x

y = c1e−x + c2e

2x + 120 cos 2x− 3

20 sin 2x

(g) yh = c1e−x + c2e

2x

Let y1 = e−x and y2 = e2x, then W (y1, y2) = 3ex.

v1 =∫

− e2x(e2x)3ex dx = −1

9e3x

v2 =∫ e−x(e2x)

3ex dx = 13x

yp = −19e2x + 1

3xe2x

y = c1e−x + c2e

2x − 19e2x + 1

3xe2x

(h) Solve for yh: m2 − 2m + 1 = 0 =⇒ m = 1, 1. Then yh = c1ex + c2xex.

Let y1 = ex and y2 = xex. Then W (y1, y2) = e2x.

v1 =∫

−xex

(

ex

x

)

e2x dx = −x

v2 =∫ ex

(

ex

x

)

e2x dx = lnx

yp = −xex + xex lnx

y = c1ex + c2xex − xex + xex lnx

(i) Solve for yh: m2 + 4 = 0 =⇒ m = ±2i. Then with α = 0 and β = 2, yh =

c1 cos 2x + c2 sin 2x.

172

Page 175: AMA SolutionsManual

CHAPTER 7. DIFFERENTIAL EQUATIONS

Let y1 = cos 2x and y2 = sin 2x, then W (y1, y2) = 2.

v1 =∫

− sin 2x(4 sec2 2x)2 dx = − sec 2x

v2 =∫ cos 2x(4 sec2 2x)

2 dx = ln |sec 2x + tan 2x|yp = 2 sin 2x ln |sec 2x + tan 2x| − 2

y = c1 cos 2x + c2 sin 2x + sin 2x ln |sec 2x + tan 2x| − 1

(21) By Hooke’s Law, the spring constant k satisfies the equation −98 = k(0.7), so that

k = 140N/m Then with m = 10 kg, y0 = −0.05 m, and v0 = −0.1 m/s, the motion of

the mass is given by

y = −0.05 cos√

14x +−0.0267 sin√

14x

(22) Kirchoff’s loop law gives i′′(t)+20i′(t)+200i(t) = 0, and the solution is i = e−10t(c1 cos 10t+

c2 sin 10t). The initial conditions are i(0) = 0 and i′(0) = E(0)L − R

L i(0) − q(0)LC =

120.5 − 10

0.5(0)− 0.510−2 (0) = 24. This yields c1 = 0 and c2 = 12

5 ; thus i(t) = 125 e−10t sin 10t for

any t > 0.

(23) (a) Let A =

[

0 1

8 −2

]

, then λ1 = 2, v1 =

[

1

2

]

, λ2 = −4 and v2 =

[

−1

4

]

.

y = c1e2xv1 + c2e

−4xv2

Therefore,

y1 = c1e2x − c2e

−4x

y2 = 2c1e2x + 4c2e

−4x.

(b) Let A =

[

0 1

8 −2

]

, then λ1 = 2, v1 =

[

1

2

]

, λ2 = −4 and v2 =

[

−1

4

]

.

P =

[

12 −1

4

1 1

]

and P−1 =

[

43

13

−43

23

]

.

f (x) =

[

0

ex

]

and g (x) = P−1f (x) =

[

13ex

23ex

]

.

w1 = e2x(∫

e−2x 13exdx + c1

)

= e2x(

13

e−xdx + c1

)

= −13ex + c1e

2x

w2 = e−4x(∫

e4x 23exdx + c2

)

= e−4x(

23

e5xdx + c2

)

= 215ex + c2e

−4x

y = Pw =

[

12 −1

4

1 1

][

−13ex + c1e

2x

215ex + c2e

−4x

]

=

[

12c1e

2x − 15ex − 1

4c2e−4x

c1e2x − 1

5ex + c2e−4x

]

.

(c) With P =

1 1 2

1 3 4

2 4 5

and D =

−2 0 0

0 4 0

0 0 1

, we have

173

Page 176: AMA SolutionsManual

CHAPTER 7. DIFFERENTIAL EQUATIONS

A = PDP−1, where A =

−5 −15 −21

3 1 3

0 6 7

. Putting

y1

y2

y3

= P

u1

u2

u3

into the equation, one concludes that

d

dt

u1

u2

u3

= P−1AP

u1

u2

u3

= D

u1

u2

u3

.

This implies u1 = −2u1, u2 = 4u2 and u3 = u3. Integration yields u1 = c1e−2t,

u2 = c2e4t and u3 = c3e

t, where c1, c2 and c3 are arbitrary constants. We thus

conclude that

y = Pu = P

c1e−2t

c2e4t

c3et

=

c1e−2t + c2e

4t + c3et

c1e−2t + 3c2e

4t + 4c3et

2c1e−2t + 4c2e

4t + 5c3et

.

(d) Let A =

1 −3 3

3 −5 3

6 −6 4

, then λ1 = −2, v1 =

1

1

0

, λ2 = −2, v2 =

−1

0

1

, λ3 = 4 and

v3 =

1212

1

.

P =

1 −1 12

1 0 12

0 1 1

and P−1 =

−12

32 −1

2

−1 1 0

1 −1 1

.

f (x) =

0

0

x2 + 5

and g (x) = P−1f (x) =

−12

32 −1

2

−1 1 0

1 −1 1

0

0

x2 + 5

=

−12x2 − 5

2

0

x2 + 5

.

w1 = e−2x[∫

e2x(

−12x2 − 5

2

)

dx + c1

]

= −18

(

2x2 − 2x + 11)

+ c1e−2x

w2 = c2e−2x

w3 = e4x(∫

e−4x(

x2 + 5)

dx)

= − 132

(

8x2 + 4x + 41)

+ c3e4x

174

Page 177: AMA SolutionsManual

CHAPTER 7. DIFFERENTIAL EQUATIONS

y = Pw =

316x + c1e

−2x − c2e−2x + 1

2c3e4x − 3

8x2 − 12964

316x + c1e

−2x + 12c3e

4x − 38x2 − 129

64

c2e−2x − 1

8x + c3e4x − 1

4x2 − 4132

(e) Diagonalize A =

[

1 −2

−2 4

]

to obtain AP = PD, where

P =

[

2 −12

1 1

]

and D =

[

0 0

0 5

]

.

Substituting y = Pz into the system y′ = Ay +

[

1

t

]

, one obtains

z′ = Dz + P−1

[

1

t

]

, or

[

z′1z′2

]

=

[

0 0

0 5

][

z1

z2

]

+

[

15(t + 2)15(4t− 2)

]

We thus have z′1 = 15(t + 2) and z′2 = 5z2 + 1

5(4t− 2). Solving these equations, we

conclude that z1 = t2

10 + 2t5 + C1 and

e−5t z2 =

1

5e−5t(4t− 2)dt + C2,

or z2 = C2e5t − 4

25t +

6

125. Finally, using y = Pz, one has

y1 = −1

2C2e

5t +t2

5+

22

25t + 2C1 −

3

125,

y2 = C2e5t +

t2

10+

6

25t + C1 +

6

125.

175

Page 178: AMA SolutionsManual
Page 179: AMA SolutionsManual

Chapter 8

Laplace and Fourier Transformations

(1) (a)

L[

e3t cos2 2t]

= L

[

e3t

(

1

2+

1

2cos 4t

)]

= L

[

e3t 1

2+

1

2e3t cos 4t

]

=1

2L[

e3t]

+1

2L[

e3t cos 4t]

=1

2

1

s− 3+

1

2

s− 3

(s− 3)2 + 16.

(b)

L[

t2 sin 3t]

=d2

ds2

[

3

s2 + 9

]

=d

ds

[

− 6s

(s2 + 9)2

]

=18 (s2 − 3)

(s2 + 9)3.

(2) (a) L−1[

1s2−2s+9

]

= L−1[

1(s−1)2+8

]

= 1√8L−1[ √

8(s−1)2+8

]

= 1√8et sin

√8t

(b) L−1[

3s2+4s+6

]

= L−1[

3(s+2)2+2

]

= 3√2L−1[ √

2(s+2)2+2

]

= 3√2e−2t sin

√2t

(c)

L−1

[

s + 4

s2 + 4s + 8

]

= L−1

[

s + 4

(s + 2)2 + 4

]

= L−1

[

s

(s + 2)2 + 4+

4

(s + 2)2 + 4

]

= L−1

[

s

(s + 2)2 + 4

]

+ 2L−1

[

2

(s + 2)2 + 4

]

= e−2t (cos 2t− sin 2t) + 2e−2t sin 2t = e−2t (sin 2t + cos 2t)

(d) L−1[

3s+7s2−2s−3

]

= L−1[

3s+7(s−3)(s+1)

]

= 4L−1[

1s−3

]

− L−1[

1s+1

]

= 4e3t − e−t

177

Page 180: AMA SolutionsManual

CHAPTER 8. LAPLACE AND FOURIER TRANSFORMATIONS

(e)

L−1

[

3s + 1

(9s− 1) (s2 + 1)

]

= L−1

[

54

41 (9s− 1)

]

− L−1

[

6s− 13

41 (s2 + 1)

]

=6

41L−1

[

1

s− 1/9

]

− 6

41L−1

[

s

s2 + 1

]

+13

41L−1

[

1

s2 + 1

]

=6

41et/9 − 6

41cos t +

13

41sin t

(f) L−1[

s+1s2+s

]

= L−1[

1s

]

= 1

(3) (a)

L−1

[

1

s (s2 − 3s + 2)

]

= L−1

[

1

2s− 1

s− 1+

1

2 (s− 2)

]

=1

2− et +

1

2e2t.

(b) If a 6= b, then

L−1

[

αs + β

(s2 + a2) (s2 + b2)

]

= αL−1

[

s

(s2 + a2) (s2 + b2)

]

+ βL−1

[

1

(s2 + a2) (s2 + b2)

]

b2 − a2(cos at− cos bt) +

β

b2 − a2

(

sin at

a− sin bt

b

)

.

If a = b, then

L−1

[

αs + β

(s2 + a2) (s2 + b2)

]

= αL−1

[

s

(s2 + a2)2

]

+ βL−1

[

1

(s2 + a2)2

]

=αt sin at

2a+

β(sin at− at cos at)

2a3.

(c) Since q − p2

4 > 0, we may put ω2 = q − p2

4 and complete square to obtain

L−1

[

αs + β

(s2 + ps + q)2

]

= L−1

α (s + p/2) + (β − αp/2)(

(s + p/2)2 + ω2)2

= e−pt/2

[

α cos ωt +β − αp/2

2ω3/2(sinωt− ωt cos ωt)

]

.

178

Page 181: AMA SolutionsManual

CHAPTER 8. LAPLACE AND FOURIER TRANSFORMATIONS

(4) (a)

L[

y′ − 5y]

= L[

8t2 − 2]

L[

y′]

− 5L [y] = 8L[

t2]

− 2L [1]

[sY (s)− y (0)]− 5Y (s) = 82!

s3− 2

1

s

(s− 5) Y (s)− 1 =16− 2s2

s3

Y (s) =s3 − 2s2 + 16

s3 (s− 5)=

91

125 (s− 5)+

34

125s− 16

25s2− 16

5s3

y (t) =91

125L−1

[

1

s− 5

]

+34

125L−1

[

1

s

]

− 16

25L−1

[

1

s2

]

− 8

5L−1

[

2

s3

]

=91

125e5t +

34

125− 16

25t− 8

5t2

(b)

L[

y′ − 5y]

= L[

e3t]

[sY (s)− y (0)]− 5Y (s) =1

s− 3

Y (s) =s− 2

(s− 3) (s− 5)=

3

2 (s− 5)− 1

2 (s− 3)

y (t) =3

2L−1

[

1

s− 5

]

− 1

2L−1

[

1

s− 3

]

=3

2e5t − 1

2e3t

(c)

L[

y′ − 5y]

= L [cos t]

[sY (s)− y (0)]− 5Y (s) =s

s2 + 1

Y (s) =s2 + s + 1

(s2 + 1) (s− 5)=

31

26 (s− 5)− 5s− 1

26 (s2 + 1)

y (t) =31

26L−1

[

1

s− 5

]

− 5

26L−1

[

s

s2 + 1

]

+1

26L−1

[

1

s2 + 1

]

=31

26e5t − 5

26cos t +

1

26sin t

179

Page 182: AMA SolutionsManual

CHAPTER 8. LAPLACE AND FOURIER TRANSFORMATIONS

(d)

L[

y′ − 5y]

= L[

e3t + cos t]

[sY (s)− y (0)]− 5Y (s) =1

s− 3+

s

s2 + 1

Y (s) =22

13 (s− 5)− 1

2 (s− 3)− 5s− 1

26 (s2 + 1)

y (t) =22

13L−1

[

1

s− 5

]

− 1

2L−1

[

1

s− 3

]

− 5

26L−1

[

s

s2 + 1

]

+1

26L−1

[

1

s2 + 1

]

=22

13e5t − 1

2e3t − 5

26cos t +

1

26sin t

(e)

L[

y′′ − y′ − 2y]

= L[

4t2]

[

s2Y (s)− sy (0)− y′ (0)]

− [sY (s)− y (0)]− 2Y (s) =8

s3

(

s2 − s− 2)

Y (s) =8

s3+ 1

Y (s) =7

3 (s + 1)+

2

3 (s− 2)− 3

s+

2

s2− 4

s3

y (t) =7

3L−1

[

1

s + 1

]

+2

3L−1

[

1

s− 2

]

− 3L−1

[

1

s

]

+ 2L−1

[

1

s2

]

− 2L−1

[

2

s3

]

=7

3e−t +

2

3e2t − 3 + 2t− 2t2

(f)

L[

y′′ − y′ − 2y]

= L [sin 2t]

[

s2Y (s)− sy (0)− y′ (0)]

− [sY (s)− y (0)]− 2Y (s) =2

s2 + 4(

s2 − s− 2)

Y (s) =2

s2 + 4+ 1

Y (s) =s− 6

20 (s2 + 4)− 7

15 (s + 1)+

5

12 (s− 2)

y (t) =1

20L−1

[

s

s2 + 4

]

− 3

20L−1

[

2

s2 + 4

]

− 7

15L−1

[

1

s + 1

]

+5

12L−1

[

1

s− 2

]

=1

20cos 2t− 3

20sin 2t− 7

15e−t +

5

12e2t

180

Page 183: AMA SolutionsManual

CHAPTER 8. LAPLACE AND FOURIER TRANSFORMATIONS

(g)

L[

y′′ − y′ − 2y]

= L[

e2t]

[

s2Y (s)− sy (0)− y′ (0)]

− [sY (s)− y (0)]− 2Y (s) =1

s− 2(

s2 − s− 2)

Y (s) =1

s− 2+ 1

Y (s) =2

9 (s− 2)− 2

9 (s + 1)+

1

3 (s− 2)2

y (t) =2

9L−1

[

1

s− 2

]

− 2

9L−1

[

1

s + 1

]

+1

3L−1

[

1

(s− 2)2

]

=2

9e2t − 2

9e−t +

1

3te2t

(h)

L[

y′′ + 4y′ + 8y]

= L[

8t2 + 8t + 18]

[

s2Y (s)− sy (0)− y′ (0)]

+ 4 [sY (s)− y (0)] + 8Y (s) =16

s3+

8

s2+

18

s(

s2 + 4s + 8)

Y (s) =16

s3+

8

s2+

18

s+ 1

Y (s) = − 2s + 7

s2 + 4s + 8+

2

s+

2

s3

y (t) = −2L−1

[

s

(s + 2)2 + 4

]

− 7

2L−1

[

2

(s + 2)2 + 4

]

+ 2L−1

[

1

s

]

+ L−1

[

2

s3

]

= −2e−2t (cos 2t− sin 2t)− 7

2e−2t sin 2t + 2 + t2

= t2 + 2− 3

2e−2t sin 2t− 2e−2t cos 2t

(i)

L[

y′′ + 4y′ + 5y]

= L [2 cos t− 2 sin t]

[

s2Y (s)− sy (0)− y′ (0)]

+ 4 [sY (s)− y (0)] + 5Y (s) =2s

s2 + 1− 2

s2 + 1(

s2 + 4s + 5)

Y (s) =2s

s2 + 1− 2

s2 + 1+ 1

Y (s) =1

2

s

s2 + 1− 1

2

s

s2 + 4s + 5− 1

s2 + 4s + 5

181

Page 184: AMA SolutionsManual

CHAPTER 8. LAPLACE AND FOURIER TRANSFORMATIONS

y (t) =1

2L−1

[

s

s2 + 1

]

− 1

2L−1

[

s

(s + 2)2 + 1

]

− L−1

[

1

(s + 2)2 + 1

]

=1

2cos t− 1

2e−2t (cos t− 2 sin t)− e−2t sin t =

1

2cos t− 1

2e−2t cos t

(j)

L[

y′′ + 2y′ + 3y]

= L[

t2 + sin t]

[

s2Y (s)− sy (0)− y′ (0)]

+ 2 [sY (s)− y (0)] + 3Y (s) =2

s3+

1

s2 + 1(

s2 + 2s + 3)

Y (s) =2

s3+

1

s2 + 1+ 1

Y (s) =19s + 167

108 (s2 + 2s + 3)− s− 1

4 (s2 + 1)+

2

27s− 4

9s2+

2

3s3

y (t) =19

108L−1

[

s

(s + 1)2 + 2

]

+167

108L−1

[

1

(s + 1)2 + 2

]

− 1

4L−1

[

s

s2 + 1

]

+1

4L−1

[

1

s2 + 1

]

+2

27L−1

[

1

s

]

− 4

9L−1

[

1

s2

]

+1

3L−1

[

2

s3

]

=19

108e−t

(

cos√

2t−√

2

2sin√

2t

)

+167

108

√2

2e−t sin

√2t− 1

4cos t

+1

4sin t +

2

27− 4

9t +

1

3t2

=19

108e−t cos

√2t +

37

27

√2

2e−t sin

√2t− 1

4cos t +

1

4sin t +

2

27− 4

9t +

1

3t2

(k)

L[

y′′ − 2y′]

= L[

et sin t]

[

s2Y (s)− sy (0)− y′ (0)]

− 2 [sY (s)− y (0)] =1

(s− 1)2 + 1(

s2 − 2s)

Y (s) =1

(s− 1)2 + 1+ 1

Y (s) =3

4 (s− 2)− 1

2 (s2 − 2s + 2)− 3

4s

y (t) =3

4L−1

[

1

s− 2

]

− 1

2L−1

[

1

(s− 1)2 + 1

]

− 3

4L−1

[

1

s

]

=3

4e2t − 1

2et sin t− 3

4

182

Page 185: AMA SolutionsManual

CHAPTER 8. LAPLACE AND FOURIER TRANSFORMATIONS

(l) Apply Laplace transform to the equation, i.e. L [y′′ − y′ + 2y] = L[(

6t2 + 8t + 7)

et]

.

Then

[

s2Y (s)− sy (0)− y′ (0)]

− [sY (s)− y (0)] + 2Y (s)

= 62

(s− 1)3+ 8

1

(s− 1)2+ 7

1

s− 1

Thus

(

s2 − s + 2)

Y (s) = 62

(s− 1)3+ 8

1

(s− 1)2+ 7

1

s− 1+ 1

Y (s) =1

(s− 1)2+

6

(s− 1)3

y (t) = L−1

[

1

(s− 1)2

]

+ 3L−1

[

2

(s− 1)3

]

= tet + 3t2et.

(5) (a) Taking Laplace Transform on both sides of the differential equations and making use

of the initial conditions, we have

Y (s) =As

(s2 + ω2)(s2 + k2)+

(s2 + ω2)(s2 + k2).

i. If ω = k, then

y (t) = L−1

[

As

(s2 + ω2)2+

(s2 + ω2)2

]

= Atsinωt

2ω+ B

sin ωt− ωt cos ωt

2ω2.

ii. If ω 6= k, then

y (t) = L−1

[

As

(s2 + ω2) (s2 + k2)+

(s2 + ω2) (s2 + k2)

]

=A

k2 − ω2(cos ωt− cos kt) +

k2 − ω2

(

sin kt

k− sinωt

ω

)

.

(b) Since L[

5e−2t · sin 2t]

=10

(s + 2)2 + 4, we obtain

s2Y (s)− s + sY (s)− 1 + Y (s) =10

(s + 2)2 + 4.

183

Page 186: AMA SolutionsManual

CHAPTER 8. LAPLACE AND FOURIER TRANSFORMATIONS

Therefore,

Y (s) =s + 1

s2 + s + 1+

1

s2 + s + 1× 10

(s + 2)2 + 4

=s + 1

s2 + s + 1− 10

37

3s− 4

s2 + s + 1+

10

37

3s + 5

(s + 2)2 + 4

=7

37

s

(s + 1/2)2 + 3/4+

77

37

1

(s + 1/2)2 + 3/4

+30

37

s

(s + 2)2 + 4+

50

37

1

(s + 2)2 + 4,

from which it follows that

y (t) = e−t/2

(

7

37cos

√3

2t− 70

37√

3sin

√3

2t

)

+ e−2t

(

30

37cos 2t− 5

37sin 2t

)

.

(c) Take Laplace Transform to obtain sY (s) + 4Y (s) =5s

s2 + 4. Thus

Y (s) =5s

(s + 4)(s2 + 4).

Since5s

(s + 4)(s2 + 4)= − 1

s + 4+

1 + s

s2 + 4, we conclude that

y(t) = −e−4t +sin 2t

2+ cos 2t.

(6) The circuit equation is given by

2di

dt+ 20i +

1

0.08

∫ t

0i (τ) dτ = 200 e−t.

Denoting L [i (t)] by I (s), we obtain 2sI (s) + 20I (s) +25

2

I (s)

s=

200

s + 1, which implies

I (s) =400s

(s + 1) (4s2 + 40s + 25)

=100s

(s + 1)(

s + 5 + 52

√3) (

s + 5− 52

√3)

=400

11 (s + 1)+

100

33

( √3− 6

s + 5 + 52

√3

)

− 100

33

( √3 + 6

s + 5− 52

√3

)

.

184

Page 187: AMA SolutionsManual

CHAPTER 8. LAPLACE AND FOURIER TRANSFORMATIONS

Taking inverse Laplace Transform, we obtain

i (t) =400

11e−t +

100(√

3− 6)

33e−(5+ 5

2

√3)t − 100

(√3 + 6

)

33e−(5− 5

2

√3)t.

(7) (a)

(f ∗ g) (t) =

∫ t

0eaτdτ =

[

1

aeaτ

]t

0

=eat − 1

a.

(b)

(f ∗ g) (t) =

∫ t

0(t− τ) sinωτdτ = t

∫ t

0sinωτdτ −

∫ t

0τ sin ωτdτ

= t

[

−cos ωt− 1

ω

]t

0

−[

τcos ωτ

ω

]t

0−∫ t

0

(

−cos ωτ

ω

)

dτ =− sinωt + ωt

ω2.

(c)

(f ∗ g) (t) =

∫ t

0τ2ea(t−τ)dτ =

[

−τ2

aea(t−τ)

]t

0

−∫ t

0

(

−2τ

aea(t−τ)

)

=

[

−τ2

aea(t−τ)

]t

0

−[

a2ea(t−τ)

]t

0

+

∫ t

0

2

a2ea(t−τ) dτ

= −2at + 2 + a2t2 − 2eat

a3.

(8) (a) f (t) =

2t if 0 < t ≤ 1

t if t > 1and f ′ (t) =

2 if 0 < t ≤ 1

1 if t > 1.

F (s) = L [f (t)] =

∫ ∞

0f (t) e−stdt

=

∫ 1

02te−stdt +

∫ ∞

1te−stdt =

∫ ∞

0te−stdt +

∫ 1

0te−stdt

=

[

−1

ste−st

]∞

0

+1

slim

x→∞

∫ x

0e−stdt +

[

−1

ste−st

]1

0

+1

s

∫ 1

0e−stdt

=1

slim

x→∞

[

−1

se−st

]x

0

− 1

se−s +

1

s

[

−1

se−st

]1

0

=1

s2− 1

se−s − 1

s2e−s +

1

s2

=2

s2− 1

se−s − 1

s2e−s

185

Page 188: AMA SolutionsManual

CHAPTER 8. LAPLACE AND FOURIER TRANSFORMATIONS

L[

f ′ (t)]

=

∫ ∞

0f ′ (t) e−stdt

=

∫ 1

02e−stdt +

∫ ∞

1e−stdt =

∫ ∞

0e−stdt +

∫ 1

0e−stdt

= limx→∞

[

−1

se−st

]∞

0

+

[

−1

se−st

]1

0

=2

s− 1

se−s

(b) f (t) =

t2 if 0 < t ≤ 1

0 if t > 1, f ′ (t) =

2t if 0 < t < 1

0 if t > 1,

f ′′ (t) =

2 if 0 < t < 1

0 if t > 1.

L[

f ′′ (t)]

=

∫ ∞

0f ′′ (t) e−stdt =

∫ 1

02e−stdt

=

[

−2

se−st

]1

0

= −2

se−s +

2

s=

2

s

(

1− e−s)

(9) (a)

L−1

[

s2

(s2 + 4)2

]

= L−1

[

s

(s2 + 4)× s

(s2 + 4)

]

=

∫ t

0cos 2x cos 2 (t− x) dx

=1

2

∫ t

0[cos (2t) + cos (4x− 2t)] dx

=1

2cos (2t)

∫ t

0dx +

1

2

∫ t

0cos (4x− 2t) dx

=1

4sin 2t +

1

2t cos 2t

(b)

L−1

[

1

(s2 + 1)3

]

= L−1

[

1

(s2 + 1)2× 1

(s2 + 1)

]

=

∫ t

0

sinx− x cos x

2sin (t− x) dx

=1

2

∫ t

0(sin x− x cos x) (sin t cos x− cos t sinx) dx

=1

2sin t

∫ t

0

(

sin x cos x− x cos2 x)

dx +1

2cos t

∫ t

0

(

x sinx cos x− sin2 x)

dx

=3

8sin t− 1

8t2 sin t− 3

8t cos t

186

Page 189: AMA SolutionsManual

CHAPTER 8. LAPLACE AND FOURIER TRANSFORMATIONS

(c)

L−1

[

s

(s2 + 4)3

]

= L−1

[

1

(s2 + 4)2× s

s2 + 4

]

=

∫ t

0

sin 2x− 2x cos 2x

16cos 2 (t− x) dx

=1

16

∫ t

0(sin 2x− 2x cos 2x) (cos 2t cos 2x + sin 2t sin 2x) dx

=1

16cos 2t

∫ t

0

(

sin 2x cos 2x− 2x cos2 2x)

dx

+1

16sin 2t

∫ t

0

(

sin2 2x− 2x sin 2x cos 2x)

dx

=1

64t sin 2t− 1

32t2 cos 2t

(10) (a) We take Laplace Transform of the equation of motion to obtain

Y (s) = αsG(s) + (β +cα

M)G(s) +

1

MF (s)G(s),

where F (s) = L [f(t)] and

G(s) =1

s2 + cM s + k

M

=1

(s + c2M )2 + 4kM−c2

4M2

.

Suppose g(t) = L−1 [G(s)].

i. If c2 < 4kM , define 4kM−c2

4M2 = ω2. Then g(t) = e−c

2Mt · sinωt

ωand

y(t) = αe−c

2Mt(cos ωt− c

2Mωsinωt) + (β +

M)g(t) +

1

M(f ∗ g)(t).

ii. If c2 = 4kM , then g(t) = t · e− c2M

t and therefore

y(t) =(

α− cα

2Mt)

e−c

2Mt + (β +

M)g(t) +

1

M(f ∗ g)(t).

iii. If c2 > 4kM , put 4kM−c2

4M2 = −Ω2. Then g(t) = e−c

2Mt · e

Ωt − e−Ωt

2Ωand therefore

y(t) = αe−c

2Mt

(

eΩt + e−Ωt

2− ck

4M· e

Ωt − e−Ωt

Ω

)

+ (β +cα

M)g(t) +

1

M(f ∗ g)(t).

187

Page 190: AMA SolutionsManual

CHAPTER 8. LAPLACE AND FOURIER TRANSFORMATIONS

(11) Taking Laplace Transform, we obtain

2sI1 + 2sI2 =E0

s

4sI1 +

(

5s + 20 +20

s

)

I2 = 0.

A simple calculation shows that

I2 = −2E0

s(

s + 20 + 201s

) = −2E0

(s2 + 20s + 20)

=−2E0

(

s + 10 + 4√

5) (

s + 10− 4√

5) =

√5E0

20

(

1

s + 10 + 4√

5− 1

s + 10− 4√

5

)

.

Therefore,

i2(t) =

√5 E0

20

(

e−(10+4√

5)t − e−(10−4√

5)t)

.

Similarly, I1 =E0

2s2− I2 implies

i1(t) = L−1

[

E0

2s2

]

− i2(t) =E0 t

2−√

5E0

20

(

e−(10+4√

5)t − e−(10−4√

5)t)

.

(12) (a) L [3 sin 2t] = L

[

y (t) +∫ t0 (t− τ) y (τ) dτ

]

.

Let G (s) = L [g (t− τ)] and g (t) = t, so G (s) = 1s2

3

(

2

s2 + 22

)

= Y (s) + Y (s) G (s)

6

s2 + 4=

(

1 +1

s2

)

Y (s)

Y (s) =6s2

(s2 + 4) (s2 + 1)=

8

s2 + 4− 2

s2 + 1

y (t) = L−1 [Y (s)] = L

−1

[

8

s2 + 4− 2

s2 + 1

]

= 4 sin 2t− 2 sin t

(b) L[

e−t]

= L

[

y (t) + 2∫ t0 cos (t− τ) y (τ) dτ

]

.

188

Page 191: AMA SolutionsManual

CHAPTER 8. LAPLACE AND FOURIER TRANSFORMATIONS

Let G (s) = L [g (t− τ)] and g (t) = cos t, so G (s) = ss2+1

.

1

s + 1= Y (s) + 2Y (s) G (s)

1

s + 1=

(

1 +2s

s2 + 1

)

Y (s)

Y (s) =s2 + 1

(s + 1)3=

1

s + 1− 2

(s + 1)2+

2

(s + 1)3

y (t) = L−1 [Y (s)] = L

−1

[

1

s + 1− 2

(s + 1)2+

2

(s + 1)3

]

= e−t − 2te−t + t2e−t

(13) Since m = 1, c = 2 and k = 15, the spring mass-system after the blow is given by

y′′(t) + 2y′(t) + 5y(t) = δ(t)

with initial conditions y(0) = y′(0) = 0.

Apply Laplace transform to the system, we have s2Y (s) + 2sY (s) + 5Y (s) = 1 or

Y (s) =1

s2 + 2s + 5=

1

(s + 1)2 + 22

Therefore, y(t) = 12e−t sin(2t).

(14) Apply Laplace transform to the equation y(t) = 14e−3t sin(2t) to obtain

Y (s) =1

4

2

(s + 3)2 + 22=

1

2s2 + 12s + 26.

Therefore, m = 2, c = 12, k = 26.

(15)

G(ω) =

∫ ∞

−∞g(x)e−iωxdx =

∫ ∞

−∞f(kx)e−iωxdx

Let kx = t, then kdx = dt

G(ω) =

∫ ∞

−∞f(t)e−iω( t

k) dt

k=

1

k

∫ ∞

−∞f(t)e−i(ω

k)tdt =

1

kF (

w

k)

189

Page 192: AMA SolutionsManual

CHAPTER 8. LAPLACE AND FOURIER TRANSFORMATIONS

(16) Determine the Fourier transform of the function f(x) = e−x2

F (w) =

∫ ∞

−∞f(x)e−iωxdx =

∫ ∞

−∞e−x2

e−iωxdx

By table, if f = e−t2

2σ2 , then F (w) = σ√

2πe−σ2ω2

2 . Set σ2 = 12 , then

F (w) =

∫ ∞

−∞e−x2

e−iωxdx =

1

2

√2πe

−12

ω2

2 =√

πe−14

ω2

(17) Considering delta function, δ(t), with continuous g(t) and t0 ∈ (a, b), then

∫ b

ag(t)δ(t− t0)dt = g(t0)

where δ(t) = limǫ→0 fǫ(t), fǫ(t) =

1ǫ , 0 ≤ t ≤ ǫ

0, otherwise. So,

∫ ba δ(t)dt = 1, δ(t) =

limǫ→0 fǫ(t) = 0 for t 6= 0 and δ(0) = limǫ→0 fǫ(0) =∞. So,

∫ b

ag(t)δ(t− t0)dt =

∫ b

ag(t) lim

ǫ→0fǫ(t− t0)dt = g(t0)

⇒ limǫ→0

∫ b

ag(t)fǫ(t− t0)dt = g(t0)

190

Page 193: AMA SolutionsManual

Chapter 9

Partial Differential Equations

(1) Differentiation yields∂2u1

∂t2= cos x ·

[

−c2 cos ct]

= c2 ∂2u1

∂x2.

Similar calculations for u2. Since u = cos(x± ct) = cos ct cos x∓ sin ct sinx, we conclude

by the previous calculations and by the Principle of superposition that u satisfies the wave

equation.

(2) Write u (x, t) = X (x) T (t) and substitute into the partial differential equation:

X (x) T ′ (t) = c2X ′′ (x)T (t)

Hence, the two differential equations are:

X ′′ (x) + λX (x) = 0, X (0) = 0, X (L) = 0 (SLP)

T ′ (t) = −λc2T (t) (DET)

Follow the same way as in Example 9.2.1,

n0 = 1, λn =(nπ

L

)2, Xn (x) = βn sin

(nπ

Lx)

Solving the DET,

T (t) = ae−λc2t, and hence Tn (t) = ane−(nπc/L)2t

191

Page 194: AMA SolutionsManual

CHAPTER 9. PARTIAL DIFFERENTIAL EQUATIONS

The solution u (x, t) now becomes:

u (x, t) =∞∑

n=1

Bne−(nπc/L)2t sin(nπ

Lx)

By the initial condition,

u (x, 0) = f (x) =∞∑

n=1

Bne−(nπc/L)2(0) sin(nπ

Lx)

=∞∑

n=1

Bn sin(nπ

Lx)

∫ L

0sin2

(nπ

Lx)

dx =L

2and

Bn =

∫ L0 f (x) sin

(

nπL x)

dx∫ L0 sin2

(

nπL x)

dx=

2

L

∫ L

0f (x) sin

(nπ

Lx)

dx

For different function f (x) in part (a) to (f), find the corresponding Bn:

(a) f (x) = 1

Bn =2

L

∫ L

0(1) sin

(nπ

Lx)

dx =2

L

∫ L

0sin(nπ

Lx)

dx =2

L

[−L

nπcos(nπ

Lx)

]L

0

=−2

nπ[cos (nπ)− 1] =

−2

nπ[(−1)n − 1] =

4nπ , n is odd,

0, n is even.

Therefore, the solution is u (x, t) =∑∞

n odd4

nπe−(nπc/L)2t sin(

nπL x)

.

(b) f (x) = x

Bn =2

L

∫ L

0x sin

(nπ

Lx)

dx =2

L

[−L

nπx cos

(nπ

Lx)

]L

0

− 2

L

∫ L

0

−L

nπcos(nπ

Lx)

dx

=−2

nπL cos (nπ) +

2

[

L

nπsin(nπ

Lx)

]L

0

=2L

nπ(−1)n+1 +

2

nπ(0)

Therefore,

u (x, t) =∞∑

n=1

2L (−1)n+1

nπe−(nπc/L)2t sin

(nπ

Lx)

.

192

Page 195: AMA SolutionsManual

CHAPTER 9. PARTIAL DIFFERENTIAL EQUATIONS

(c) f (x) = A + B−AL x

Bn =2

L

∫ L

0

(

A +B −A

Lx

)

sin(nπ

Lx)

dx

= A

[

2

L

∫ L

0sin(nπ

Lx)

dx

]

+B −A

L

[

2

L

∫ L

0x sin

(nπ

Lx)

dx

]

= A2

[

(−1)n+1 + 1]

+B −A

L

2L

nπ(−1)n+1

=2

[

A + B (−1)n+1]

(by (a) and (b))

Therefore, u (x, t) =∑∞

n=1

2[A+B(−1)n+1]nπ e−(nπc/L)2t sin

(

nπL x)

.

(d) f (x) = sin(

3πL x)

Bn =2

L

∫ L

0sin

(

Lx

)

sin(nπ

Lx)

dx =

0, n 6= 3,2L

∫ L0 sin2

(

3πL x)

dx n = 3.

B3 =2

L

∫ L

0sin2

(

Lx

)

dx =1

L

∫ L

0

[

1− cos

(

Lx

)]

dx

=1

L

[

x− L

6πsin

(

Lx

)]L

0

= 1

Therefore,

u (x, t) = e−(3πc/L)2t sin

(

Lx

)

.

(e) f (x) = cos(

3πL x)

Bn =2

L

∫ L

0cos

(

Lx

)

sin(nπ

Lx)

dx

=1

L

∫ L

0

[

sinπ

L(3 + n)x− sin

π

L(3− n)x

]

dx

=1

L

[ −L

π (3 + n)cos

π

L(3 + n)x− −L

π (3− n)cos

π

L(3− n)x

]L

0

=−1

π (3 + n)[cos π (3 + n)− 1]− −1

π (3− n)[cos π (3− n)− 1] (n 6= 3)

=(−1)n+4 + 1

π (3 + n)− (−1)n−2 + 1

π (3− n)

=[(−1)n + 1] (3− n)− [(−1)n + 1] (3 + n)

π (9− n2)

=2n [(−1)n + 1]

π (n2 − 9)=

0, n is odd,4n

π(n2−9), n is even.

193

Page 196: AMA SolutionsManual

CHAPTER 9. PARTIAL DIFFERENTIAL EQUATIONS

Therefore,

u (x, t) =∞∑

n even

4n

π (n2 − 9)e−(nπc/L)2t sin

(nπ

Lx)

.

(f) f (x) =

x, 0 ≤ x ≤ L2 ,

L− x, L2 ≤ x ≤ L.

Bn

=2

L

∫ L2

0x sin

(nπ

Lx)

dx +2

L

∫ L

L2

(L− x) sin(nπ

Lx)

dx

=2

L

∫ L

0x sin

(nπ

Lx)

dx +2

L

∫ L

L2

(L− 2x) sin(nπ

Lx)

dx

=2L

nπ(−1)n+1 − 2

[

(L− 2x) cos(nπ

Lx)]L

L2

+2

∫ L

L2

(−2) cos(nπ

Lx)

dx

=2L

nπ(−1)n+1 +

2L

nπ(−1)n − 4

[

L

nπsin(nπ

Lx)

]L

L2

=4L

n2π2sin(nπ

2

)

=

4L(−1)k

(2k+1)2π2, where n = 2k + 1, k = 0, 1, 2, ...,

0, otherwise.

Therefore,

u (x, t) =∞∑

k=0

4L

(2k + 1)2 π2(−1)k e−[(2k+1)πc/L]2t sin

[

(2k + 1)π

Lx

]

.

(3) Write u (x, t) = v (x, t) + l (x) and substitute into the partial differential equation:

vt (x, t) = c2[

vxx (x, t) + l′′ (x)]

The boundary conditions imply that

u (0, t) = v (0, t) + l (0) = A, u (L, t) = v (L, t) + l (L) = B, 0 ≤ t ≤ ∞.

Furthermore, the initial condition also implies that

u (x, 0) = v (x, 0) + l (x) = f (x) , 0 ≤ x ≤ L.

Note that l (x) = A + B−AL x, l′′ (x) = 0, l (0) = A and l (L) = B.

194

Page 197: AMA SolutionsManual

CHAPTER 9. PARTIAL DIFFERENTIAL EQUATIONS

Then v (x, t) should satisfy the following initial boundary value problem

∂v∂t = c2 ∂2v

∂x2 ,

v (0, t) = 0, v (L, t) = 0, 0 ≤ t <∞,

v (x, 0) = f (x)− l (x) , 0 ≤ x ≤ L.

Solve the initial boundary value problem in question 2:

Write v (x, t) = X (x)T (t) and substitute into the partial differential equation:

X (x) T ′ (t) = c2X ′′ (x)T (t)

Hence, the two differential equations are:

X ′′ (x) + λX (x) = 0, X (0) = 0, X (L) = 0 (SLP)

T ′ (t) = −λc2T (t) (DET)

Follow the same way as in Example 9.2.1,

n0 = 1, λn =(nπ

L

)2, Xn (x) = βn sin

(nπ

Lx)

Solving the DET,T (t) = Ae−λc2t, and hence

Tn (t) = Ane−(nπc/L)2t

The solution v (x, t) now becomes:

v (x, t) =∞∑

n=1

Bne−(nπc/L)2t sin(nπ

Lx)

By the initial condition,

v (x, 0) = f (x)− l (x) =∞∑

n=1

Bn sin(nπ

Lx)

Since∫ L0 sin2

(

nπL x)

dx = L2 ,

Bn =

∫ L0 [f (x)− l (x)] sin

(

nπL x)

dx∫ L0 sin2

(

nπL x)

dx=

2

L

∫ L

0[f (x)− l (x)] sin

(nπ

Lx)

dx

195

Page 198: AMA SolutionsManual

CHAPTER 9. PARTIAL DIFFERENTIAL EQUATIONS

For different function f (x) in part (a) and (b), find the corresponding Bn:

(a) f (x) = sin(

3πL x)

Bn =2

L

∫ L

0

[

sin

(

Lx

)

−A− B −A

Lx

]

sin(nπ

Lx)

dx

=2

L

∫ L

0sin

(

Lx

)

sin(nπ

Lx)

dx− 2

L

∫ L

0

(

A +B −A

Lx

)

sin(nπ

Lx)

dx

By (2) (d),

2

L

∫ L

0sin

(

Lx

)

sin(nπ

Lx)

dx =2

L

∫ L

0sin

(

Lx

)

sin(nπ

Lx)

dx

=

0, n 6= 3,

1, n = 3.

By (2) (c)

2

L

∫ L

0

(

A +B −A

Lx

)

sin(nπ

Lx)

dx =2

[

A + (−1)n+1 B]

Therefore,

v (x, t) = e−( 3πcL )

2t sin

(

Lx

)

−∞∑

n=1

2

[

A + (−1)n+1 B]

e−(nπcL )

2t sin

(nπ

Lx)

and

u (x, t) = e−( 3πcL )

2t sin

(

Lx

)

−∞∑

n=1

2

[

A + (−1)n+1 B]

e−(nπcL )

2t sin

(nπ

Lx)

+

(

A +B −A

Lx

)

.

(b) f (x) = Q (x) =

x, 0 ≤ x ≤ L2 ,

L− x, L2 ≤ x ≤ L.

Bn =2

L

∫ L

0

[

Q (x)−A− B −A

Lx

]

sin(nπ

Lx)

dx

=2

L

∫ L

0Q (x) sin

(nπ

Lx)

dx− 2

L

∫ L

0

[

A +B −A

Lx

]

sin(nπ

Lx)

dx

196

Page 199: AMA SolutionsManual

CHAPTER 9. PARTIAL DIFFERENTIAL EQUATIONS

By (2) (f)

2

L

∫ L

0Q (x) sin

(nπ

Lx)

dx =

4L(−1)k

(2k+1)2π2, n = 2k + 1, k = 0, 1, 2, ...,

0, otherwise.

By (2) (c)

2

L

∫ L

0

(

A +B −A

Lx

)

sin(nπ

Lx)

dx =2

[

A + (−1)n+1 B]

Therefore,

v (x, t)

=

∞∑

k=0

4L (−1)k

(2k + 1)2 π2−

∞∑

k=0

2[

A + (−1)2k+3 B]

(2k + 1)π

e−[(2k+1)πc/L]2t sin

(

(2k + 1) π

Lx

)

and

u (x, t)

=

∞∑

k=0

4L (−1)k

(2k + 1)2 π2−

∞∑

k=0

2[

A + (−1)2k+3 B]

(2k + 1)π

e−[(2k+1)πc/L]2t sin

(

(2k + 1) π

Lx

)

+

(

A +B −A

Lx

)

.

(4) (a) Separation of variables yields

u(x, t) =∞∑

n=1

bn · e−4n2t sin nx,

with

bn =2

π

∫ π

0x(π − x) sinnx dx =

4[1− (−1)n]

πn3.

Thus

u(x, t) =8

π

n odd

e−4n2t

n3sinnx.

(b) The function w(x) = 30 + 70x satisfies w(0) = 30, w(1) = 100 and w′′(x) = 0. It is

now clear that v(x, t) = u(x, t) − w(x) would satisfy the equation vt = c2 vxx, with

boundary conditions v(0, t) = v(1, t) = 0 and initial condition v(x, 0) = 20 − 70x.

197

Page 200: AMA SolutionsManual

CHAPTER 9. PARTIAL DIFFERENTIAL EQUATIONS

We may now solve for v(x, t) to obtain

u(x, t) = (30 + 70x) +∞∑

n=1

(

100× (−1)n + 40

)

e−n2π2c2t sinnπx.

(5) We substitute u(x, t) = φ(t) · w(x, t) into ut = c2uxx − ku to obtain

φ′(t)w + φ(t)wt = φ(t)[

c2wxx − kw]

If w(x, t) satisfies the standard heat equation wt = c2 wxx, then φ′(t) = −k φ(t), or

φ(t) = e−kt. For the given IBVP, we let u(x, t) = e−t · w(x, t) to obtain

wt = wxx for 0 < x < 1, t > 0;

w(0, t) = 0 and w(1, t) = 0 for t ≥ 0;

w(x, 0) = sinπx + 12 sin 3πx for 0 ≤ x ≤ 1

It follows that

w(x, t) = e−π2t · sinπx +1

2e−9π2t · sin 3πx

and therefore

u(x, t) = e−t · w(x, t) = e−(π2+1)t · sinπx +1

2e−(9π2+1)t · sin 3πx.

(6)

2

∫ 1

0sin pnx sin pmx dx =

∫ 1

0[cos(pn − pm)x− cos(pn + pm)x] dx.

For n 6= m, we have

2

∫ 1

0sin pnx · sin pmx dx =

sin(pn − pm)

pn − pm− sin(pn + pm)

pn + pm.

Since pn is a positive root of the equation tan p = − pσ , it follows that sin pn = −

(pn

σ

)

cos pn.

Therefore,

2

∫ 1

0sin pnx · sin pmx dx =

sin pn cos pm − cos pn sin pm

pn − pm− sin pn cos pm + cos pn sin pm

pn + pm= 0.

Similarly, we have

∫ 1

0sin2 pnx dx =

1

2

∫ 1

0[1− cos 2pnx ] dx =

1

2− sin 2pn

4pn=

1

2− sin pn · cos pn

2pn=

1

2+

cos2 pn

2σ.

198

Page 201: AMA SolutionsManual

CHAPTER 9. PARTIAL DIFFERENTIAL EQUATIONS

(7) (a) Solving the S-L Problem, one has λn = (n− 12)2 as eigenvalues and Xn(x) = sin(n−

12)x as the corresponding eigenfunctions . Therefore,

u(x, t) =∞∑

n=1

bn e−c2(n− 12)2 t sin(n− 1

2)x.

Using the initial condition x(π − x) = u(x, 0), one has

x(π − x) =∞∑

n=1

bn sin(n− 1

2)x,

where

bn =

∫ π0 x(π − x) sin(n− 1

2)x dx∫ π0 sin2(n− 1

2)x dx=

8

π

[

4 + (−1)n (2n− 1)π

(2n− 1)3

]

Therefore,

u(x, t) =8

π

∞∑

n=1

[

4 + (−1)n (2n− 1)π

(2n− 1)3

]

e−c2(n− 12)2 t sin(n− 1

2)x.

(b) We have u(x, t) = a0+∑∞

n=1 an e−16n2t cos nx, where a0+∑∞

n=1 an cos nx = u(x, 0) =

2xπ if 0 ≤ x ≤ π

22π (π − x) if π

2 ≤ x ≤ π

The arbitrary constants a0 and an’s may now be calculated by Euler’s Formula. The

result is

u(x, t) =1

2+

8

π2

n even

[

cos nπ2 − 1

n2

]

e−16n2t cos nx.

(c) We substitute

u(x, t) =∞∑

n=1

Tn(t) sin nπx,

where Tn(t)’s are to be determined, into the p.d.e. to obtain T ′′n (t)+n2π2c2 Tn(t) = 0

for n 6= 3 and T ′′3 (t) = −9π2c2 T3(t)+1. Initial conditions on u(x, t) imply T ′

n(0) = 0

for every n = 1, 2, 3, . . ., T1(0) = 3, T4(0) = −2, and Tn(0) = 0 whenever n 6= 1, 4.

Solving the ODE, we obtain Tn(t) = 0 when n 6= 1, 3, 4; T1(t) = 3 cos πct, T3(t) =1

9π2c2[1− cos 3πct] and T4(t) = −2 cos 4πct. Therefore,

u(x, t) = 3 cos πct · sinπx + 1

[

1− cos 3πct

9π2c2

]

sin 3πx− 2 cos 4πct · sin 4πx.

199

Page 202: AMA SolutionsManual

CHAPTER 9. PARTIAL DIFFERENTIAL EQUATIONS

(d) We substitute

u(x, t) =∞∑

n=1

Tn(t) sin pnx

into the equation to obtain T ′1(t) = −p2

1 T1(t) + 1 and T ′n(t) = −p2

n Tn(t) for n > 1,

with Tn(0) = 0 for n = 1, 2, 3, . . .. Here, pn’s are positive roots of the equation

tan p = −p. Thus Tn(t) = 0 whenever n > 1 and T1(t) =1− e−p2

1 t

p21

. Hence

u(x, t) =

(

1− e−p21 t

p21

)

sin p1x.

(e) We put u(x, t) = v(x, t) + x · cos t into the equation to obtain

∂v

∂t= c2 ∂2v

∂x2+ x · sin t,

with boundary conditions v(0, t) = 0 and v(1, t) = 0 and initial condition v(x, 0) = 0

for 0 ≤ x ≤ 1.

Substituting v(x, t) =∑∞

n=1 Tn(t) sinnπx into the equation and bearing in mind

that

x · sin t =∞∑

n=1

2(−1)n+1 sin t

nπsin nπx,

we obtain

T ′n(t) + c2n2π2Tn(t) =

2(−1)n+1 sin t

nπ.

Solving for the equation, with initial condition Tn(0) = 0, we have

Tn(t) =2 (−1)n+1

(

e−c2n2π2t

c4n4π4 + 1− cos t

c4n4π4 + 1+

c2n2π2 sin t

c4n4π4 + 1

)

Hence

u(x, t) = x · cos t +2

π

∞∑

n=1

(−1)n+1

(

e−c2n2π2t − cos t + c2n2π2 sin t

n(c4n4π4 + 1)

)

sinnπx.

(8) Write u (x, t) = X (x) T (t) and substitute into the partial differential equation:

X (x)T ′′ (t) = c2X ′′ (x)T (t)

200

Page 203: AMA SolutionsManual

CHAPTER 9. PARTIAL DIFFERENTIAL EQUATIONS

Hence, the two differential equations are:

T ′′ (t) + λc2T (t) = 0

X ′′ (x) + λX (x) = 0, X (0) = 0, X (L) = 0 (SLP)

Follow the same way as in Example 9.2.1,

n0 = 1, λn =(nπ

L

)2, Xn (x) = βn sin

(nπ

Lx)

Solving T ′′ (t) + λc2T (t) = 0 with λn =(

nπL

)2,

Tn (t) = an cos(nπc

Lt)

+ bn sin(nπc

Lt)

The solution u (x, t) now becomes:

u (x, t) =∞∑

n=1

[

An cos(nπc

Lt)

+ Bn sin(nπc

Lt)]

sin(nπ

Lx)

By the initial condition,

u (x, 0) = f (x) =∞∑

n=1

An sin(nπ

Lx)

ut (x, t) = g (x) =nπc

L

∞∑

n=1

[

−An sin(nπc

Lt)

+ Bn cos(nπc

Lt)]

sin(nπ

Lx)

ut (x, 0) = g (x) =nπc

L

∞∑

n=1

Bn sin(nπ

Lx)

Since∫ L0 sin2

(

nπL x)

dx = L2 ,

An =

∫ L0 f (x) sin

(

nπL x)

dx∫ L0 sin2

(

nπL x)

dx=

2

L

∫ L

0f (x) sin

(nπ

Lx)

dx and

Bn =L

nπc

∫ L0 g (x) sin

(

nπL x)

dx∫ L0 sin2

(

nπL x)

dx=

2

nπc

∫ L

0g (x) sin

(nπ

Lx)

dx

For different function f (x) in part (a) to (e), find the corresponding An and Bn:

201

Page 204: AMA SolutionsManual

CHAPTER 9. PARTIAL DIFFERENTIAL EQUATIONS

(a) f (x) ≡ sin(

3πL x)

and g (x) ≡ 0. By (2) (d)

An =2

L

∫ L

0sin

(

Lx

)

sin(nπ

Lx)

dx =

0, n 6= 3,

1, n = 3.

Bn =2

nπc

∫ L

0(0) sin

(nπ

Lx)

dx = 0

Therefore, the solution is u (x, t) = cos(

3πcL t)

sin(

3πL x)

.

(b) f (x) ≡ 0 and g (x) ≡ sin(

3πL x)

. By (2) (d)

An =2

L

∫ L

0(0) sin

(nπ

Lx)

dx = 0

Bn =2

nπc

∫ L

0sin

(

Lx

)

sin(nπ

Lx)

dx =

0, n 6= 3,L

3πc , n = 3.

Therefore, u (x, t) = L3πc sin

(

3πcL t)

sin(

3πL x)

.

(c) f (x) ≡ sin(

3πL x)

and g (x) ≡ sin(

3πL x)

. By (2) (d)

An =2

L

∫ L

0sin

(

Lx

)

sin(nπ

Lx)

dx =

0, n 6= 3,

1, n = 3.

Bn =2

nπc

∫ L

0sin

(

Lx

)

sin(nπ

Lx)

dx =

0, n 6= 3,L

3πc , n = 3.

Therefore, u (x, t) =[

cos(

3πcL t)

+ L3πc sin

(

3πcL t)]

sin(

3πL x)

.

(d) f (x) ≡ sin(

3πL x)

and g (x) ≡ sin(

4πL x)

. By (2) (d)

An =2

L

∫ L

0sin

(

Lx

)

sin(nπ

Lx)

dx =

0, n 6= 3,

1, n = 3.

Bn =2

nπc

∫ L

0sin

(

Lx

)

sin(nπ

Lx)

dx =

0, n 6= 4,2

4πc

∫ L0 sin2

(

4πL x)

dx, n = 4.

B4 =1

4πc

∫ L

0

[

1− cos

(

Lx

)]

dx =1

4πc

[

x− L

8πsin

(

Lx

)]L

0

=L

4πc

Therefore, u (x, t) = cos(

3πcL t)

sin(

3πL x)

+ L4πc sin

(

4πcL t)

sin(

4πL x)

.

202

Page 205: AMA SolutionsManual

CHAPTER 9. PARTIAL DIFFERENTIAL EQUATIONS

(e) f (x) = ǫQ (x) =

ǫx, 0 ≤ x ≤ L2 ,

ǫ (L− x) , L2 ≤ x ≤ L.

and g (x) = 0. By (2) (f)

An =2

L

∫ L

0ǫQ (x) sin

(nπ

Lx)

dx =

4ǫL(−1)k

(2k+1)2π2, where n = 2k + 1, k = 0, 1, 2, ...,

0, otherwise.

Bn =2

nπc

∫ L

0(0) sin

(nπ

Lx)

dx = 0

Therefore, u (x, t) =∑∞

k=0

4ǫL(−1)k

(2k+1)2π2cos[

(2k+1)πcL t

]

sin[

(2k+1)πL x

]

.

(9) Write u (x, t) = X (x) T (t) and substitute into the partial differential equation:

X (x)T ′′ (t) = c2X ′′ (x)T (t)

Hence, the two differential equations are:

T ′′ (t) + λc2T (t) = 0

X ′′ (x) + λX (x) = 0, X ′ (0) = 0, X ′ (L) = 0 (SLP)

Follow the same way as in Example 9.2.1,

n0 = 0, λn =(nπ

L

)2, Xn (x) = αn cos

(nπ

Lx)

Solving T ′′ (t) + λc2T (t) = 0 with λn =(

nπL

)2,

Tn (t) = an cos(nπc

Lt)

+ bn sin(nπc

Lt)

The solution u (x, t) now becomes:

u (x, t) =

∞∑

n=0

[

An cos(nπc

Lt)

+ Bn sin(nπc

Lt)]

cos(nπ

Lx)

203

Page 206: AMA SolutionsManual

CHAPTER 9. PARTIAL DIFFERENTIAL EQUATIONS

By the initial condition,

u (x, 0) = A0 +∞∑

n=1

An cos(nπ

Lx)

ut (x, t) = B0 +nπc

L

∞∑

n=1

[

−An sin(nπc

Lt)

+ Bn cos(nπc

Lt)]

cos(nπ

Lx)

ut (x, 0) = B0 +nπc

L

∞∑

n=1

Bn cos(nπ

Lx)

Since∫ L

0cos2

(nπ

Lx)

dx =

L, n = 0,L2 , n ≥ 1.

We have

A0 =

∫ L0 f (x) dx

L=

1

L

∫ L

0f (x) dx and

An =

∫ L0 f (x) cos

(

nπL x)

dx∫ L0 cos2

(

nπL x)

dx=

2

L

∫ L

0f (x) cos

(nπ

Lx)

dx and

B0 =

∫ L0 g (x) dx

L=

1

L

∫ L

0g (x) dx and

Bn =L

nπc

∫ L0 g (x) cos

(

nπL x)

dx∫ L0 cos2

(

nπL x)

dx=

2

nπc

∫ L

0g (x) cos

(nπ

Lx)

dx.

Thus,

ut (x, t) = B0 +nπc

L

∞∑

n=1

[

−An sin(nπc

Lt)

+ Bn cos(nπc

Lt)]

cos(nπ

Lx)

and

u (x, t) = A0 + B0t +∞∑

n=1

[

An cos(nπc

Lt)

+ Bn sin(nπc

Lt)]

cos(nπ

Lx)

.

(10) Separation of variables and the superposition principle give

u(x, t) =∞∑

n=1

[αn cos nπt + βn sinnπt] sinnπx

Initial conditions are

u(x, 0) =

0.02x if 0 ≤ x ≤ 12

0.02(1− x) if 12 ≤ x ≤ 1

and ut(x, 0) = 0

204

Page 207: AMA SolutionsManual

CHAPTER 9. PARTIAL DIFFERENTIAL EQUATIONS

Therefore, we have

βn = 0 and αn = 2

[

∫ 1/2

00.02x · sinnπx dx +

∫ 1

1/20.02(1− x) · sinnπx dx

]

.

This implies (see Ex. 3 of Revision with L = 1 and K = 0.01)

u(x, t) =0.08

π2

n odd

(−1)n−1

2

n2cos nπt · sin nπx.

(11) We substitute u(x, t) =∑∞

n=1 Tn(t) sinnx into the wave equation

utt = uxx + sin 100x · sin 100t

to obtain ordinary differential equations

T ′′n (t) + n2Tn(t) = 0 when n 6= 100 and T ′′

100(t) + 10000 · T100(t) = sin 100t,

with initial conditions

T1(0) = 2, T ′1(0) = 0, T4(0) = −3, T ′

4(0) = 0 and Tn(0) = T ′n(0) = 0 for n 6= 1, 4

Solving these ODE, we obtain

T1(t) = 2 cos t,

T4(t) = −3 cos 4t,

T100(t) = L−1

100

(s2 + 1002)2

=sin 100t− 100t · cos 100t

20000.

Therefore,

u(x, t) = 2 cos t · sinx− 3 cos 4t · sin 4x +

[

sin 100t− 100t · cos 100t

20000

]

sin 100x

(12) Differentiating E(t) with respect to t, one obtains

E′(t) =

∫ L

0[ρ ut(x, t) · utt(x, t) + T ux(x, t) · uxx(x, t)] dx

=

∫ L

0

[

ρ ut(x, t) · c2uxx(x, t) + T ux(x, t) · uxt(x, t)]

dx

= T

[∫ L

0ut(x, t) · uxx(x, t) dx +

∫ L

0ux(x, t) · uxt(x, t) dx

]

205

Page 208: AMA SolutionsManual

CHAPTER 9. PARTIAL DIFFERENTIAL EQUATIONS

Integrating the second integral by parts, we have

∫ L

0ux(x, t) · uxt(x, t) dx

= [ux(L, t) · ut(L, t)− ux(0, t) · ut(0, t)]−∫ L

0uxx(x, t) · ut(x, t) dx

= −∫ L

0uxx(x, t) · ut(x, t) dx because ut(0, t) = ut(L, t) = 0

Therefore, E′(t) = 0, and the total energy is conserved.

(13) Let u(x, y) = X(x)Y (y).

Solving the following two ODEs:

1) X ′′(x) + λX(x) = 0 where 0 < x < 2;

2) Y ′′(y)− λY (y) = 0 where 0 < y < 1.

So, with the given boundary conditions,

λn =n2π2

4, Xn = sin

nπx

2, Yn = β[e

nπ2

y − e−nπ2

y], n = 1, 2, 3 . . . .

Therefore,

un(x, y) = βn[enπ2

y − e−nπ2

y] sinnπx

2, n = 1, 2, 3....

By Principle of Superposition,

u(x, y) =∞∑

n=1

βn[enπ2

y − e−nπ2

y] sinnπx

2

For boundary condition: u(x, 1) = x(2− x), 0 < x < 2,

βn =2

2[enπ2 − e−

nπ2 ]

∫ 2

0x(2− x) sin

nπx

2dx, n = 1, 2, 3....

=1

[enπ2 − e−

nπ2 ]

−8(−2 + 2 cos nπ + nπ sinnπ)

n3π3

=1

[enπ2 − e−

nπ2 ]

16(1− (−1)n)

n3π3

206

Page 209: AMA SolutionsManual

CHAPTER 9. PARTIAL DIFFERENTIAL EQUATIONS

Finally,

u(x, y) =∞∑

n=1

[

1

[enπ2 − e−

nπ2 ]

16(1− (−1)n)

n3π3

]

[enπ2

y − e−nπ2

y] sinnπx

2

=32

π3

n=odd

1

n3

(

enπ2

y − e−nπ2

y

enπ2 − e−

nπ2

)

sinnπx

2

(14) Set Solution of uxx + uyy = 0 as u(x, y) = X(x)Y (y). Then

Y ′′X + X ′′Y = 0Y ′′

Y= −X ′′

X= −λ

Solving the following two ODEs:

(1) Y ′′(x) + λY (x) = 0 where 0 < y < 1;

(2) X ′′(y)− λX(y) = 0 where 0 < x < 1.

So, with the given boundary conditions,

λn = n2π2, Yn = sinnπy, X(x) = αenπx + βe−nπx

Since u(1, y) = 0,

X(1) = αenπ + βe−nπ = 0⇒ β = −αe2nπ

Therefore,

Xn = αn[enπx − e2nπe−nπx] = αn[enπx − enπ(2−x)], n = 1, 2, 3, . . .

and

un(x, y) = αn[enπx − enπ(2−x)] sinnπy, n = 1, 2, 3 . . . .

By Principle of Superposition,

u(x, y) =∞∑

n=1

αn[enπx − enπ(2−x)] sinnπy, n = 1, 2, 3....

From boundary condition, u(0, y) = 3 sin πy − 4 sin 6πy for 0 ≤ y ≤ 1, we have

α1 =3

(1− e2π), α6 = − 4

(1− e12π)and αn = 0, n 6= 1, 6

207

Page 210: AMA SolutionsManual

CHAPTER 9. PARTIAL DIFFERENTIAL EQUATIONS

Hence

u(x, y) =3

(1− e2π)[eπx − eπ(2−x)] sinπy − 4

(1− e12π)[e6πx − e6π(2−x)] sin 6πy.

(15) Using polar coordinates, we substitute u(r, θ) = R(r)·S(θ) into the equations and proceed

as in the lecture notes. Since S(θ + 2π) = S(θ), we have

S(θ) = an cos nθ + bn sin nθ when n ≥ 1 and S(θ) = a0 when n = 0,

R(r) = cn rn + dn r−n when n ≥ 1 and R(r) = c0 ln r + d0 when n = 0

Consequently, the Principle of Superposition implies that

u(r, θ) = (A0 ln r + B0) +∞∑

n=1

rn [An cos nθ + Bn sinnθ] + r−n [Cn cos nθ + Dn sinnθ] ,

where A0, B0, An, Bn, Cn and Dn’s are arbitrary constants.

(a) To ensure that u(r, θ) is finite as r → 0, we must choose A0 = Cn = Dn = 0.

Furthermore, the boundary condition implies that

B0 +∞∑

n=1

[An cos nθ + Bn sinnθ] =

1 for 0 ≤ θ ≤ π;

0 for π < θ < 2π

Therefore,

u(r, θ) =1

2+∑

n odd

[

2

]

rn sinnθ.

(b) The requirement for limr→∞ u(r, θ) to be finite implies that

A0 = An = Bn = 0

Therefore,

u(r, θ) = B0 +∞∑

n=1

r−n[Cn cos nθ + Dn sin nθ].

Since u(1, θ) = 1 + cos 2θ − 3 sin 4θ, we may compare coefficients to obtain B0 = 1,

C2 = 1, D4 = −3 and all other coefficients = 0. Therefore,

u(r, θ) = 1 +cos 2θ

r2− 3 sin 4θ

r4.

208